format , 4.1 MB - Recreaţii Matematice

Transcription

format , 4.1 MB - Recreaţii Matematice
Anul XII, Nr. 1
Ianuarie – Iunie 2010
RECREAŢII
MATEMATICE
REVISTĂ DE MATEMATICĂ PENTRU ELEVI ŞI PROFESORI
Seminarul Matematic “A. Myller”
(1910 – 2010)
e iπ = −1
Asociaţia “Recreaţii Matematice”
IAŞI - 2010
Semnificaţia formulei de pe copertă:
iπ
Într-o formă concisă, formula e = −1 leagă cele patru ramuri fundamentale
ale matematicii:
ARITMETICA
GEOMETRIA
ALGEBRA
ANALIZA MATEMATICĂ
reprezentată
reprezentată
reprezentată
reprezentată
de
de
de
de
1
π
i
e
Redacţia revistei :
Petru ASAFTEI, Dumitru BĂTINEŢU-GIURGIU (Bucureşti), Temistocle BÎRSAN, Dan
BRÂNZEI, Alexandru CĂRĂUŞU, Constantin CHIRILĂ, Eugenia COHAL, Adrian
CORDUNEANU, Mihai CRĂCIUN (Paşcani), Paraschiva GALIA, Paul GEORGESCU,
Mihai HAIVAS, Gheorghe IUREA, Lucian-Georges LĂDUNCĂ, Mircea LUPAN, Gabriel
MÎRŞANU, Alexandru NEGRESCU (student, Iaşi), Gabriel POPA, Dan POPESCU
(Suceava), Florin POPOVICI (Braşov), Maria RACU, Neculai ROMAN (Mirceşti), Ioan
SĂCĂLEANU (Hârlău), Ioan ŞERDEAN (Orăştie), Dan TIBA (Bucureşti), Marian TETIVA
(Bârlad), Lucian TUŢESCU (Craiova), Adrian ZANOSCHI, Titu ZVONARU (Comăneşti)
COPYRIGHT © 2010, ASOCIAŢIA “RECREAŢII MATEMATICE”
Toate drepturile aparţin Asociaţiei “Recreaţii Matematice”. Reproducerea integrală sau
parţială a textului sau a ilustraţiilor din această revistă este posibilă numai cu acordul prealabil
scris al acesteia. Se consideră că autorii materialelor trimise redacţiei revistei sunt, în mod
implicit, de acord cu publicarea lor, îşi asumă responsabilitatea conţinutului lor şi cedează
Asociaţiei “Recreaţii Matematice” dreptul de proprietate intelectuală asupra acestora.
TIPĂRITĂ LA S.C. BLUE SIM & CO S.R.L.
Bd. Carol I, nr. 3-5
Tel. 0788 498933
E-mail: [email protected]
ISSN 1582 - 1765
Anul XII, Nr. 1
Ianuarie – Iunie 2010
RECREAŢII
MATEMATICE
REVISTĂ DE MATEMATICĂ PENTRU ELEVI ŞI PROFESORI
e iπ = −1
Revistă cu apariţie semestrială
EDITURA “RECREAŢII MATEMATICE”
IAŞI - 2010
Centenarul Seminarului Matematic ”A. Myller”
Au trecut 100 de ani de când, ı̂n toamna 1910, Alexandru Myller, proaspăt
profesor de geometrie analitică la Universitatea din Iaşi, a avut iniţiativa creării unei
biblioteci de matematică la această universitate. Biblioteca a fost gândită, la ı̂nceput,
pentru uz personal. Destul de curând, ı̂nsă, s-a permis accesul la publicaţii şi altor
cadre didactice, biblioteca devenind astfel de uz comun. Prin aceasta, se ı̂mbunătăţea
climatul favorabil ridicării calitative a activităţii de cercetare ştiinţifică ı̂n universităţile româneşti, creat de legea ı̂nvăţământului, care l-a avut ca principal autor pe
matematicianul şi astronomul Spiru Haret, pe atunci ministru al Instrucţiunii Publice.
În Iaşi, terenul fusese pregătit de activitatea vechilor profesori de matematică de la
Universitatea din Iaşi: Şt. Emilian, N. Culianu, I. Melik, M. Tzony, C. Climescu,
A. Mănescu, I.D. Rallet, V. Costin. Dintre aceştia, N. Culianu, C. Climescu şi
I. Melik, ı̂mpreună cu câţiva profesori de liceu, contribuiseră la apariţia la Iaşi, pe o
perioadă de 6 ani (1883-1888), a revistei Recreaţii Ştiinţifice, precursoare a Gazetei
Matematice. Au existat dificultăţi legate de eterna problemă a absenţei fondurilor
şi de mentalitatea unor persoane aflate ı̂n posturi de conducere ale ţării şi Iaşilor,
care considerau că nu este necesară o bibliotecă de uz comun; profesorii interesaţi de
anumite cărţi şi reviste puteau să şi le procure pe cheltuială proprie şi să-şi constituie
biblioteci personale.
A fost nevoie de concepţia clară şi tenacitatea tânărului profesor A. Myller,
pentru ca o astfel de instituţie să ı̂nceapă să funcţioneze, sub denumirea de Seminarul
Matematic, având ca model biblioteca de acelaşi tip de la Göttingen, Germania.
1
Mai ı̂ntâi cu un număr restrâns de publicaţii, s-a dezvoltat treptat, ajungând ca ı̂ntre
cele două războaie mondiale să fie cea mai bună bibliotecă de matematici din sudestul Europei. Pe lângă această bibliotecă, şi-au desfăşurat, mai mult sau mai puţin
sporadic, activitatea ştiinţifică şi de cercetare mai mulţi tineri talentaţi, care aveau
să devină nume cunoscute ı̂n matematica românească: C. Popovici, D. Pompeiu,
V. Vâlcovici, S. Stoilow, S. Sanielevici, Gh. Vrănceanu, Gr. Moisil, T. Popoviciu,
O. Mayer ş.a.
Obţinerea fondurilor pentru achiziţionarea de publicaţii ştiinţifice era o preocupare permanentă a lui A. Myller, care a condus biblioteca de la ı̂nfiinţare până ı̂n
anul 1947. Aceste fonduri erau obţinute prin demersuri insistente pe lângă forurile
conducătoare ale ı̂nvăţământului superior şi ale culturii româneşti, prin convingerea
unor persoane particulare să facă donaţii pentru bibliotecă şi prin contribuţii personale. Mai ales ı̂n perioada dintre cele două războaie mondiale, A. Myller pleca
ı̂n mod regulat ı̂n ţările din vestul Europei, unde colinda prin librării şi anticariate,
făcând achiziţii masive de cărţi şi de colecţii de reviste, ajungând să cheltuiască şi din
banii săi. El insista mereu pe lângă colegii săi să contribuie la ı̂mbogăţirea bibliotecii
prin publicaţii şi este cunoscut că era tot timpul nemulţumit, spunând că membrii
Seminarului Matematic nu fac suficient de mult pentru ı̂mbogăţirea patrimoniului
acestuia. A. Myller a scris un manual de geometrie analitică (foarte bun!) pentru
liceu, iar drepturile de autor au fost folosite ı̂n ı̂ntregime pentru cumpărarea de cărţi
şi reviste pentru Seminarul Matematic. Mulţi ani, această carte a fost folosită pentru
premierea studenţilor fruntaşi de la Facultatea de Matematică. Unul din succesele
importante ale lui A. Myller a fost convingerea industriaşului N. Malaxa să facă o
donaţie pentru Seminarul Matematic. Prin aceste eforturi, biblioteca Seminarului
Matematic s-a ı̂mbogăţit cu colecţiile celor mai prestigioase reviste de matematică
apărute ı̂n lume, cu ultimele cărţi publicate la diverse edituri de specialitate din Europa şi din America, precum şi cu colecţii complete ale unora din cele mai vechi reviste
de matematică (Acta Eruditorum, Crelle′ s Journal, Journal de l′ École Polytechnique
de Paris etc.) şi cu cărţi vechi de matematică din perioada Renaşterii şi de mai târziu.
O altă direcţie ı̂n care a acţionat stăruitor A. Myller, a fost menţinerea unei
discipline stricte ı̂n Seminarul Matematic. Accesul la publicaţii era liber, fiecare
membru putea să ı̂mprumute, fără să apeleze la un bibliotecar (care, de fapt, nici nu
exista), dar era obligat să respecte câteva reguli simple ı̂nsă stricte. Orice publicaţie
ı̂mprumutată era trecută ı̂ntr-un registru de ı̂mprumuturi şi se restituia la anumite
date, publicaţiile trebuiau păstrate cu grijă pentru a se ı̂mpiedica deteriorarea sau
pierderea lor, nimeni nu avea voie să plece din Iaşi fără să restituie mai ı̂ntâi toate
publicaţiile ı̂mprumutate. Unele abateri, rare, erau sancţionate fără rezerve de prof.
A. Myller prin ridicarea dreptului de acces ı̂n bibliotecă pentru o perioadă determinată
sau chiar pentru totdeauna. Sancţiunile se aplicau la orice beneficiar găsit ı̂n culpă,
indiferent de vechimea şi prestigiul acestuia. Acest spirit de disciplină instaurat la
Seminarul Matematic a făcut ca din bibliotecă să se piardă extrem de puţine publicaţii.
În 1944, Universitatea din Iaşi a fost evacuată, ı̂mpreună cu biblioteca Seminarului
Matematic, la Alba Iulia. Mutarea a fost organizată aşa de bine, ı̂ncât la ı̂ntoarcere
ı̂n 1945, nu a lipsit decât un singur volum din fondul de publicaţii al bibliotecii.
După război, au trebuit ı̂nfruntate mari greutăţi rezultate din distrugerile cauzate
2
de luptele purtate pe teritoriul ţării (inclusiv la Iaşi), izolarea ı̂n care a intrat ţara
noastră, ca urmare a constituirii blocului statelor comuniste, şi lipsa cronică a fondurilor pentru achiziţionarea publicaţiilor. Chiar ı̂n aceste condiţii, ı̂n biblioteca Seminarului Matematic au continuat să sosească numeroase volume de reviste, obţinute
prin schimb cu revista Analele Ştiinţifice ale Universităţii ”Al.I. Cuza” Iaşi, Matematică, cărţi trimise de diverse edituri pentru recenzare ı̂n aceeaşi revistă şi, ı̂n mică
măsură, reviste obţinute prin abonament şi cărţi achiziţionate din fondurile oficiale,
mai ales din fosta Uniune Sovietică.
Trebuie menţionate aici şi numeroasele donaţii generoase ale unor membri ai Seminarului Matematic, constând din cărţi şi reviste cumpărate cu ocazia deplasărilor
ı̂n străinătate.
În acest moment, există un schimb cu circa 200 de reviste de specialitate. În
ultimul timp, prin dezvoltarea internetulului, se pot accesa liber diverse cărţi şi reviste
din lume, precum şi articole separate apărute ca preprinturi pe diverse site-uri specializate (e.g. arXiv). Mai menţionăm şi unele iniţiative la nivel de ţară care ne oferă acces
pe bază de abonament la diverse baze de date: Science Direct (unde se pot accesa,
ı̂n principal, publicaţii de la editura Elsevier), SpringerLink (cu acces la pulicaţiile de
la editurile Springer şi Birkhäuser), Thomson ISI (Web of Science, Journal Citations
Report şi Derwent Inovation Index), CSA Research Pack. Mai menţionez că, prin
eforturi financiare deosebite ale facultăţii, se asigură accesul on-line la Mathematical
Reviews şi Zentralblatt MATH.
Biblioteca Seminarului Matematic nu este importantă doar ca sursă de informare
pentru cercetătorii ı̂n domeniul matematicii sau din alte domenii ı̂nrudite. În sălile
ei domneşte o atmosferă calmă şi sobră, care ı̂ndeamnă pe cel care intră ı̂n bibliotecă
către meditaţie şi cercetare. În ultimii ani, la mesele din aceste săli s-au documentat
şi au făcut descoperiri matematice importante numeroşi cercetători care au devenit
cunoscuţi ı̂n matematica mondială. Unele din rezultatele lor au apărut ı̂n reviste de
specialitate de prestigiu din ţară şi din străinătate sau au fost ı̂ncorporate ı̂n monogafii
matematice scrise de autori din Iaşi sau din alte centre ştiinţifice din lume.
Acum, la 100 de ani de la ı̂nfiinţare, Seminarul Matematic din Iaşi, care poartă
numele fondatorului său, A. Myller, ı̂şi trăieşte o nouă tinereţe.
Prof. dr. Vasile OPROIU
Directorul Seminarului Matematic ”A. Myller”
3
Seminarul Matematic din Iaşi - 100 de ani
de ı̂nvăţământ matematic românesc
În acest an, un secol de existenţă a Seminarului Matematic se suprapune ı̂n
mod fericit cu 150 de ani de la crearea la Iaşi ı̂n 1860 a primei universităţi din ţară.
Învăţământul matematic ieşean avea o oarecare tradiţie ı̂ncă ı̂nainte de inaugurarea Universităţii, dar un adevărat centru de cercetare matematică a fost iniţiat
de profesorul Alexandru Myller ı̂n 1910, când tocmai primise postul de titular la
catedra de geometrie analitică a Facultăţii de Ştiinţe din Iaşi.
În acelaşi an, profesorul Myller a făcut demersuri către ministrul instrucţiunii
publice de atunci - Spiru Haret - ı̂n vederea obţinerii de fonduri pentru dezvoltarea
unei biblioteci de specialitate. Iniţial s-a primit o sumă cu care s-au achiziţionat
câteva reviste matematice mai importante, iar mai târziu s-a obţinut chiar o alocaţie
bugetară fixă.
Astfel, actul de naştere al Bibliotecii Seminarului Matematic (iniţial
Biblioteca Seminarului de Geometrie analitică) putem spune că a fost semnat la
18 octombrie 1910, când Alexandru Myller a ı̂nregistrat oficial primele o sută de
volume din ”Crelle′ s Journal für de reine und angewandte Mathematik”.
La ı̂nceput, spaţiul destinat bibliotecii era doar o fostă sală de curs administrată numai de profesorul Myller, ı̂nsă ı̂n timp spaţiul s-a extins, iar personalul de
administraţie a crescut prin alăturarea unor membri ai Seminarului.
Profesorul Myller concepe Biblioteca Seminarului Matematic după modelul sălii
de lectură a Seminarului Matematic de la Göttingen unde a studiat, susţinând şi
un doctorat sub ı̂ndrumarea celebrului matematician David Hilbert. Cu un corp
profesoral de calitate şi cu o bibliotecă de valoare, s-a format la Iaşi o adevărată
şcoală de matematică cunoscută sub numele de Seminarul Matematic din Iaşi (S.M.I.),
având ca bază pentru cercetare o bibliotecă de specialitate realizată cu entuziasmul
şi sacrificiul profesorilor şi care ı̂n perioada interbelică era considerată cea mai mare
bibliotecă din estul Europei.
După evacuările la Alba Iulia, ı̂n timpul celui de-Al II-lea Război Mondial,
biblioteca a revenit la Universitate fără pierderi. A urmat o perioada foarte grea
de refacere a ı̂nvăţământului universitar. La 14 octombrie 1944 Alexandru Myller
este numit rector al Universităţii, iar ı̂n noiembrie 1945 se retrage din această funcţie,
rămânând profesor activ până la ieşirea la pensie din noiembrie 1947.
În 1949, ı̂n anul următor Reformei ı̂nvăţământului, când toate bibliotecile facultăţilor au devenit filiale ale Bibliotecii Universitare din Iaşi, care s-a transformat
ı̂n Biblioteca Centrală Universitară, şi Biblioteca Seminarului Matematic a devenit la
rândul său filială, iar ı̂n 1954 a primit numele iniţiatorului său, Alexandru Myller.
Amintim că, deşi Biblioteca Seminarului Matematic a devenit subordonată Bibliotecii Centrale Universitare, la conducerea sa, din partea Facultăţii de Matematică, au
urmat după profesorul Myller următorii directori: prof. Ilie Popa ı̂n perioada 19471952; prof. Adolf Haimovici ı̂n anii 1952-1992; prof. Gheorghe Bantaş ı̂n anii
1994-2006, iar din anul 2006 prof. Vasile Oproiu.
Dacă până ı̂n 1952 toate activităţile erau făcute de profesorul Myller şi o parte
din membrii S.M.I., după aceea a fost numit şi un bibliotecar din partea B.C.U., ı̂n
4
persoana d-rei Anca Jacotă, licenţiată ı̂n limba franceză.
Membrii Seminarului, ı̂n prezent aproape 150, sunt ı̂n special de la Facultatea de
Matematică a Universităţii ”Al.I. Cuza”, de la catedra de matematică a Universităţii
Tehnice ”Gh. Asachi”, dar şi de la alte facultăţi şi instituţii de cercetare din ţară
sau străinătate. Ca utilizatori ai bibliotecii mai sunt, la recomandarea membrilor
Seminarului Matematic, şi beneficiari externi, majoritatea studenţi ı̂n ultimii ani ai
Facultăţii de Matematică sau doctoranzi.
Cu o aşezare alfabetică a majorităţii fondului de carte, Biblioteca Seminarului
Matematic are peste 81.000 de volume aflate aproape ı̂n totalitate ı̂ntr-o bază de date
informatizată, iar ı̂ntre documentele bibliotecii se află, pe lângă un fond de carte
curentă, dicţionare şi enciclopedii generale şi de specialitate, reviste de referate, şi
un fond de carte veche şi manuscrise cuprinzând aproximativ 300 de volume apărute
ı̂nainte de 1850. Dintre acestea din urmă amintim:
• Apollonii Pergaei Conicorum libri quator, 1566;
• Gemma Frisius, R. - Arithmeticae Practicae methodus facilus, Lipsiae, 1575;
• Clavius Bambergensis, Christophorus - Geometria practica, Mainz, 1606;
• La Hire, Philippe de - Nouveaux éléments des section coniques, Paris, 1679;
• Ozanam, Jacques - Dictionnaire mathématique, Amsterdam, 1691;
• Abrégé des mathématiques pour l′ usage de Sa Majesté Imperiale de toutes les
Roussies, St. Petersbourg, 1728;
• Simson, Robert - Sectionum conicarum libri V, Edinburg, 1735;
• Saverien, Alexandre - Dictionnaire universel de mathématique et de physique,
Paris, 1753;
• Garnier, J.G. - Réciproques de la géometrié, Paris, 1810;
• Stoiheia arithmetikes, Iaşi, 1818 (carte românească veche).
De asemenea, ı̂n bibliotecă există operele complete ı̂n original ale marilor matematicieni şi fizicieni cum ar fi: Euclid, Galileo Galilei, Johannes Kepler, Pierre
Fermat, Christian Huygens, Isaac Newton, Jacob Bernoulli, Leonhard
Euler, Joseph Louis Lagrange, Pierre-Simon Laplace, Karl Friedrich Gauss,
Augustin Louis Cauchy, Niels Henrick Abel, Henri Poincaré etc.
Alături de colecţiile de cărţi se află şi peste 700 de titluri de periodice de specialitate, iar dintre cele cu apariţie mai veche amintim: Acta Eruditorum (1682); Journal
de l′ École polytechnique (1797); Journal für die reine und angewandte Mathematik
(1829); Compte Rendus de l′ Académie des Sciences (1835); Bulletin de l′ Académie
Royale des Sciences (1836); Proceedings of the Royal Society of London (1862); Mathematische Annalen (1869); American Journal of Mathematics Pure and Applied (1878);
Acta Mathematica (1882); Recreaţii Ştiinţifice (1883); Bulletin of American Mathematical Society (1895); Transaction of American Mathematical Society (1900);
Annales Scientifiques de l′ Université de Jassy (1900) etc.
În acest loc ı̂ncărcat de istorie, prin care au trecut şi s-au format atâţia oameni
de seamă, avem datoria faţă de ı̂naintaşi, cât şi faţă de cei care vor veni, să păstrăm
mereu, ı̂mbinând tradiţia cu modernitatea, acest tezaur de cultură.
Andrei PATRAŞ
Bibliotecar şef al Seminarului Matematic
5
Amintiri de la Seminarul Matematic
Am intrat ca student la Facultatea de Matematică de la Universitatea din Iaşi
ı̂n septembrie 1959. După ce am făcut cunoştinţă cu personalul de la secretariat,
unde, ca şef de grupă, trebuia să predau săptămânal prezenţa studenţilor la cursuri şi
seminarii, am ajuns şi la biblioteca facultăţii, situată deasupra decanatului facultăţii
de matematică. Îmi amintesc de o sală luminoasă şi primitoare, ı̂n care se putea studia
ı̂n condiţii foarte bune. Exista şi o bibliotecară, d-ra Timofte, care stătea ı̂ntr-o săliţă
vecină, situată deasupra cabinetului decanului, prin care se trecea pentru a se intra
ı̂n sala de lectură, după ce se semna ı̂ntr-o condică care atesta numărul de vizitatori.
Studenţii din anii mari aveau o altă sală de lectură alături, deasupra amfiteatrului II.5.
Aveau cheie şi puteau să intre când doreau (aici nu exista o bibliotecară), găseau o
sală de lectură care ı̂i ı̂mbia la studiu, găseau o mică bibliotecă cu cărţi dintre cele mai
folosite la facultatea de matematică. Multă vreme am privit această sală de lectură
ca pe un loc misterios, ı̂n care s-ar fi petrecut lucruri deosebite. Nu am mai ajuns
să intru şi eu cu drepturi depline ı̂n această sală de lectură, pentru că s-a desfiinţat.
Erau unii studenţi care stăteau mult ı̂n aceste două săli de lectură, ba chiar ı̂şi fixaseră
nişte locuri ale lor. Se studia cu o anumită ı̂ndârjire, mi se părea că mulţi doreau
să-şi depăşească condiţia de copii din familii nevoiaşe şi căutau să ajungă la situaţii
mai bune. Mai târziu, după ce am terminat facultatea, s-a mai ı̂nfiinţat o sală de
lectură la parter, pe coridorul unde se află acum administratorul facultaţii, ba chiar
şi pe coridorul paralel, din faţa sălii metodice. Apoi, biblioteca pentru studenţi s-a
mutat ı̂n corpul B ı̂n locul bibliotecii facultaţii de ştiinţe economice, care, la rândul
ei, s-a mutat ı̂n corpul D, nou construit, lângă biserica ”Patruzeci de Sfinţi”.
Am ajuns să intru, de câteva ori, ı̂n biblioteca Seminarului Matematic pentru a
consulta nişte materiale necesare pentru lucrul la cercurile ştiinţifice studenţeşti. În
acest fel am cunoscut pe bibliotecara A. Jacotă, care a activat parcă o veşnicie la
această bibliotecă. Îmi amintesc că pe coridoarele de la parter şi de la etajul II era
destul de multă gălăgie, ı̂n special ı̂n pauze. Prin contrast, pe coridorul de la etajul I,
unde era intrarea la Seminarul Matematic era o atmosfera mai liniştită care, ı̂ntr-un
fel, prevestea atmosfera de lucru, de reculegere, ca ı̂ntr-o biserică, ce o regăseam de
fiecare dată ı̂n interior. Intrarea era prin săliţa ce desparte sala de carţi de cea de
colecţii de reviste. În sala de cărţi, aflată ı̂n dreapta, se adunau şi lucrau profesorii,
poate şi conferenţiarii mai ı̂n vârstă, ı̂n sala de colecţii de reviste se adunau şi lucrau
conferenţiarii şi lectorii mai ı̂n vârstă, iar ı̂n sala de modele, acolo unde acum se află
amfiteatrul ”A. Myller” (ı̂n care se ajungea, atât din sala de colecţii reviste printr-o
trecere mică ı̂nzestrată cu o perdea, după care se coborau câteva trepte sau, direct,
de pe coridorul de la Catedra de geometrie, pe unde se intra cu ajutorul unei chei),
se adunau şi lucrau asistenţii şi lectorii mai tineri. Fiecare avea locul lui, la mese cu
patru locuri şi atmosfera era realmente una de lucru.
Cam prin 1962, facultatea a primit noi spaţii de ı̂nvăţământ, devenite libere prin
mutarea facultăţilor de biologie şi de geografie ı̂n corpul B, abia dat ı̂n folosinţă. În
acest fel, fiecare catedră (Algebră, Analiză matematică, Geometrie, Ecuaţii şi matematici speciale, Mecanică) a primit câte un sediu, la fel şi profesorii şi conferenţiarii.
Cursurile şi seminariile se desfăşurau acum şi ı̂n amfiteatrul I.3 şi ı̂n sălile 1.4 şi 1.5.
6
Profesorii Gh. Gheorghiev, A. Haimovici, I. Popa, M. Haimovici, I. Creangă aveau
cabinetele lor. Existau cabinete pentru conferenţiari la analiză, geometrie, algebră şi
mecanică, sala de mecanică era folosită ca sediu de catedră, mai era un laborator de
informatică ı̂n actualul sediu al Catedrei de geometrie. În acest fel apăruse o degajare
a spaţiului la Seminarul Matematic, unde vizitatorii puteau să citească ı̂n tihnă la
una din mesele existente.
Atmosfera de studiu şi bogăţia de cărţi existente ı̂n biblioteci m-au incitat să
cumpăr şi eu cărţi de la librării sau din anticariate. Trebuie spus că situaţia generală
ı̂n România de atunci nu prea permitea ca tinerii să dea bani pe cărţi. Era destulă
sărăcie şi erau multe lipsuri care au fost ı̂ndreptate ceva mai târziu. Am ı̂nceput să
merg prin librării şi prin anticariate şi am avut destule ocazii să cumpăr cărţi utile.
Dintre titlurile ce mi le amintesc aş putea enumera: două din cele trei volume ale
tratatului de analiză matematică al lui Miron Nicolescu (primul volum, pe care ı̂l
găseam cel mai interesant şi util, ı̂n care se trata convergenţa seriilor, era de negăsit),
cele trei volume ale tratatului de analiză matematică al lui Fichtengholtz, tradus din
limba rusă, cartea de mecanică teoretică a lui Plăcinţeanu şi altele. Ştiu că la BCU
cartea lui Plăcinţeanu ca şi cea a lui Vâlcovici aveau regim special, nu se ı̂mprumutau
acasă. Mai târziu, am ı̂nceput să cumpăr şi cărţi ı̂n limba rusă. În timpul şcolii,
ı̂ncepând cu clasa a IV-a, am avut lecţii de rusă. Apoi, ı̂n timpul facultăţii, am urmat
cursurile de limba rusă concentrată pe diverşi termeni matematici, astfel că puteam
ı̂nţelege cărţi şi articole scrise ı̂n limba rusă. Nu am studiat deloc limba engleză şi
acest aspect a fost un handicap pentru mine pentru că a trebuit s-o ı̂nvăţ singur şi
să mă descurc cum pot la diversele conferinţe la care am participat şi unde a trebuit
să şi vorbesc. Îmi permit să evoc protocolul prin care se achiziţionau cărţile ı̂n limba
rusă. Se trecea, de regulă săptămânal, pe la librăria cu cărţi ruseşti, care se afla
atunci la etajul librăriei Junimea din Piaţa Unirii. (Anterior, această librărie fusese
la Fundaţie, ı̂n clădirea librăriei Maxim Gorki, situată pe locul unde se află acum
Casa de Cultură a Studenţilor. Mai târziu, am aflat că acolo fusese Jockey Clubul,
un loc unde se ı̂ntâlnea protipendada Iaşilor. Pe lângă loc de ı̂ntâlnire şi cultivare
a relaţiilor sociale, acesta mai era şi un loc unde se practicau jocuri de noroc. Am
aflat că aici au pierdut destui bani mai multe personalităţi ale Iaşului interbelic ; spre
exemplu, Cezar Petrescu.) Apoi, secţia de cărţi ı̂n limba rusă s-a mutat la Casa Cărţii.
Săptămânal, soseau nişte blancuri (broşuri reclamă) cu titluri de cărţi ce urmau să
apară. Căutam la secţia de matematică şi ne semnam ı̂n dreptul cărţilor ce doream
să le cumpărăm. La unele titluri era aglomeraţie mare de semnături, la altele niciuna;
totuşi soseau şi din cele la care nu erau semnături. Peste câteva luni, când soseau
cărţile, unele titluri erau rezervate celor semnaţi pe blancuri şi se puteau cumpăra
cu uşurinţă. Atunci când soseau mai puţine cărţi, nu se mai făcea această rezervare.
Ştiu că, o bună perioadă de timp, sistemul a funcţionat. Apoi, au ı̂nceput să apară
sincope. Îmi amintesc că nu am mai ajuns să cumpăr cartea de geometrie diferenţială
a lui S. Sternberg şi a fost mare supărarea mea. Oricum, reţin perioada ca un moment
foarte bun din viaţa mea, eram mulţumit şi de ce cumpăram şi de rezultatele ce le
obţineam atunci.
După ce am terminat facultatea, ı̂n 1964, am căpătat drept de acces liber ı̂n biblioteca Seminarului Matematic. Aceasta ı̂nsemna că puteam să intru la orice oră
7
doream, să consult şi să ı̂mprumut cărţile şi revistele existente ı̂n bibliotecă. Când
ı̂mprumutam, trebuia să le notez ı̂ntr-un registru anume făcut, unde aveam o rubrică
a mea. Cum spuneam şi mai sus, erau mulţi tineri care studiau cu mult sârg, prezenţi
aproape la orice oră din zi şi orice oră rezonabilă din noapte. Spre exemplu, colegul
meu V. Barbu era prezent tot timpul la locul său de muncă ce se afla lângă cabinetul
prof. Gheorghiev. De asemenea, făcea vizite foarte dese la bibliotecă unde consulta
ı̂n special revistele de pe panoul cu noutăţi. Am văzut ı̂n registrul de ı̂mprumuturi
că obişnuia să ı̂mprumute reviste sovietice pe care le ı̂nsemna succint : YMH -pentru
Uspekhi Matematiceskih Nauk sau ∆AH - pentru Doklady Akademii Nauk. Eu, care
eram ı̂ncadrat la Academie, veneam des pe la prof. Gheorghiev, pe la D. Papuc şi
R. Miron, care deveniseră conferenţiari, şi, bineı̂nţeles, pe la biblioteca Seminarului Matematic, unde zăboveam destul de mult. Acolo ı̂l ı̂ntâlneam deseori pe prof.
C. Corduneanu, care venea să vadă noutăţile; din când ı̂n când mă aborda şi mă
ı̂ntreba cum merge treaba cu cercetarea ştiinţifică. A fost singurul care se interesa de
ce fac tinerii ce lucrau ı̂n direcţii de cercetare diferite de a lui. Organizarea bibliotecii,
prin aranjarea cărţilor alfabetic după autori, ı̂mi permitea să descopăr numeroase
cărţi de care nu ştiam, atunci când căutam o carte recomandată. Stăteam mult şi
le răsfoiam, mai mult, pe unele le citeam şi cultura mea matematică se ı̂mbogăţea
considerabil.
Am putut să apreciez şi eficienţa bibliotecarei A. Jacotă care ţinea biblioteca ı̂n
ordine. Era tot timpul amabilă, un pic cam severă ı̂n legătură cu curăţenia ı̂n bibliotecă şi o adevărată enciclopedie ı̂n materie de biblioteconomie. Mai târziu, am
aflat că ı̂n BCU era considerată o adevărată legendă, datorita acestor vaste cunoştinţe
despre cărţi, dar şi datorită faptului că a refuzat cu ı̂ncăpăţânare să se mute ı̂n sediul
BCU (printr-o promovare) sau să primească alţi biliotecari ca ajutoare la Seminarul
Matematic. Oricum, singură ţinea ı̂n ordine vasta colecţie de reviste şi de cărţi. Aşa
cum spuneam, d-ra Jacotă era foarte exigentă cu curăţenia ı̂n sălile de lectură ale
bibliotecii. Atunci când ploua, era foarte atentă cu vizitatorii, ı̂n special cu cei tineri,
care trebuiau să se şteargă foarte bine pe picioare la intrare, pentru ca să nu ducă
apă ı̂n săli. Pe cei mai neglijenţi ı̂i mustra cu asprime şi, de regulă, a doua oară
nu se mai greşea. Trebuie să menţionez că, ı̂n sălile bibliotecii era un parchet foarte
frumos şi foarte bine ı̂ntreţinut. Cel puţin o dată pe an, se făcea curăţenie generală şi
parchetul era spălat cu petrosin, ceruit şi lustruit. Era o adevărată plăcere să priveşti
acel parchet după o astfel de curăţenie generală. Acum, acel parchet a fost scos şi
ı̂n locul lui s-a pus ceva care seamănă cu linoleumul. Oricum, deja s-a vălurit şi nu
arată deloc bine pe la colţuri. Tot ı̂n legătură cu cadrul plăcut din sălile de lectură,
trebuie să menţionez existenţa unor ciubere cu plante exotice, care erau foarte bine
ı̂ngrijite. Erau nişte femei de serviciu, foarte devotate sălilor de lectură de la Seminarul Matematic, care aveau grija să le ude, să le şteargă frunzele şi să le facă să
arate bine. Menţionez că tot ele se ocupau şi de expedierea numerelor din Anale ı̂n
operaţia de schimb cu alte reviste. Atunci când erau tipărite diverse fascicule din
Anale, nu numai de la secţia matematică, acestea se adunau pe mese ı̂n sala de cărţi,
se ı̂mpachetau ı̂n hârtie, se legau cu sfoară şi se puneau adresele la care urmau să
fie expediate. Unele pachete erau mai consistente, altele conţineau doar o fasciculă,
după cum era convenţia de schimb a revistei noastre cu alte publicaţii. La ı̂nceput,
8
adresele erau scrise de mână pe pachetele ı̂n discuţie; mai târziu au ı̂nceput să fie
dactilografiate şi lipite pe pachete. Sistemul funcţiona foarte bine şi se ı̂nregistrau
foarte puţine plângeri ı̂n legătură cu expedierea lor. Cheltuielile erau acoperite, ı̂n
marea lor majoritate, din cotizaţiile membrilor Seminarului, ı̂ncasate lunar.
Noi, care veneam de două-trei ori pe săptămână la Seminarul Matematic (unii
veneau zilnic şi studiau la mesele existente), ne opream cu plăcere la panoul de noutăţi,
unde răsfoiam diversele fascicule de reviste proaspăt primite. Era o adevărată plăcere
să descoperi un articol interesant sau să găseşti vreo citare utilă. Este adevărat că,
ı̂n vremea aceea, nu eram aşa de interesaţi de citări ; era plăcut să te ştii citat, dar
nu făceam un obiectiv din aceasta, aşa că nu prea era o vânătoare după citări. Nici
despre reviste cotate ISI nu era vorba deloc (de fapt, nu se inventase ı̂ncă sistemul
respectiv), ştiam că existau nişte reviste bune, ı̂n special cele americane, englezeşti,
franţuzeşti şi japoneze, dar, de cele mai multe ori, noi ne publicam lucrările ı̂n reviste
româneşti: Analele de la Iaşi, Revue Roumaine de Mathématiques a Academiei, de la
Bucureşti şi altele câteva. În ultimul timp, nici nu prea puteam să trimitem lucrări
ı̂n străinătate, având ı̂n vedere că pentru expediere exista un adevărat protocol ce
necesita timp, nervi şi bani şi nu erai ı̂ntotdeauna sigur că plicul respectiv va pleca.
Scene hazlii se petreceau ı̂n perioada reviziilor anuale la bibliotecă. În operaţia de
revizuire, care se desfăşura pe o perioadă de 10 zile-2 săptămâni, trebuiau să participe
toate persoanele tinere ce aveau acces la bibliotecă. Acestea, grupate ı̂n perechi,
revizuiau două sau mai multe coloane de publicaţii din rafturile bibliotecii. În acel
moment era o hărmălaie deosebită ı̂n bibliotecă din cauza comunicării ı̂ntre partenerul
aflat pe scară, care verifica existenţa cărţilor ı̂n raft, şi cel aflat la baza scării, care
verifica existenţa cărţilor ı̂n carnetele cu fişele cărţilor. Cu această ocazie se semnalau
diverse lipsuri. După ce se termina această operaţie, se trecea la recuperarea lipsurilor.
Se mai căuta ı̂ncă o dată, se mai căuta şi ı̂n alte locuri. Se căuta şi ı̂n registrele de
ı̂mprumuturi, pentru a se vedea dacă nu cumva vreun cititor nu a restituit publicaţiile;
dacă se ı̂ntâmpla acest lucru, era posibil ca cititorul respectiv să-şi piardă dreptul
de a mai ı̂mprumuta cărţi. După câteva zile de vânzoleală, majoritatea lipsurilor
se recuperau. Mai mult, uneori se recuperau şi publicaţii găsite lipsă la reviziile
anterioare. Până la urmă, se ajungea la o situaţie acceptabilă, doar 3-5 publicaţii
lipsă, şi se aştepta cu sufletul la gură terminarea reviziei şi acordarea dreptului de
a ı̂mprumuta publicaţii. În momentul când directorul bibliotecii acorda acest drept,
se pornea o adevărată cursă (o asemănam uneori cu cursele din filmele western către
terenurile neocupate, ı̂n cucerirea vestulului ı̂n America) pentru găsirea publicaţiilor
dorite a fi ı̂mprumutate. Se mergea ı̂n grabă până la raftul vizat, se căuta o scară
şi apoi publicaţia la care trebuia să se ajungă ı̂naintea altor colegi interesaţi. Aceste
publicaţii erau recomandate de către profesorii de la facultate şi erau folosite pentru
referate la doctorat sau pentru pregătirea tezelor de doctorat sau ı̂n documentarea
pentru o temă de cercetare ştiinţifică. După ce găseau publicaţiile ce ı̂i interesau,
cititorii aşteptau să le ı̂nregistreze ca publicaţii ı̂mprumutate şi plecau fericiţi cu ele
la cabinete sau acasă. Erau şi colegi care pierdeau cursa, ı̂n sensul că publicaţiile
căutate de ei fuseseră luate deja de alţii, şi trebuiau să se roage de aceştia să-i lase să
le consulte.
Era destul de greu ı̂n perioada aceea, având ı̂n vedere că nu existau aparate de
9
copiere. Îmi amintesc că, ı̂n anumite cazuri, am copiat pur şi simplu de mână câteva
articole ce mă interesau. Pentru altele, făceam fotocopii, adică le fotografiam pur
şi simplu, developam filmul şi făceam copiile pe hârtie fotografică de mărimea unui
carnet (jumătate de caiet). Erau vremuri grele, dar dispuneam de energia şi inventivitatea necesare pentru a ne descurca. La un moment dat au apărut aparate de copiat
(xeroxuri), dar erau puţine, se defectau uşor şi se cam aflau sub controlul securităţii,
având ı̂n vedere posibilitatea de a se multiplica materiale considerate duşmănoase
pentru regim. Copiile erau de proastă calitate, dar puteau fi utilizate.
În perioada ı̂n care am fost bursier la Universitatea din Napoli, Italia, ı̂n 1972, am
beneficiat de o bibliotecă organizată la fel ca şi cea din Iaşi: cu publicaţiile ordonate
alfabetic şi cu acces liber la raft pentru cititori. Director al bibliotecii era profesorul
Carlo Miranda, care a făcut o specializare la Universitatea din Göttingen, unde fusese
şi A. Myller, şi a organizat biblioteca după modelul de acolo. La un moment dat,
m-a ı̂ntrebat despre D. Mangeron cu care studiase ı̂mpreună. Biblioteca de la Napoli
dispunea de mai multe fonduri. Îmi amintesc că avea un aparat de fotocopiat la care
se puteau face liber diverse copii şi că hârtia era de un tip special, cu proprietaţi ce
aminteau de hârtia fotografică. Mai mult, erau comandate diverse cărţi ı̂n mai multe
exemplare. Spre exemplu, cărţile din colecţia Lecture Notes de la Springer se găseau
ı̂ntr-un loc dedicat acestei serii, dar şi ı̂n raft la autorul respectiv. Acelaşi lucru se
petrecea cu cărţile din colecţiile Grundlehren sau Ergebnisse ş.a. de la Springer sau cu
cărţile de la Academic Press, Marcel Dekker etc. Nu mai vorbesc de faptul că puteam
să comandăm cărţile pe care le doream, găsite prin broşurile de reclamă. Acestea
soseau ı̂n câteva luni. Am regăsit aceeaşi organizare eficientă şi la bibliotecile de la
Universitatea din Freiburg sau de la Centrul de Cercetare de la Oberwolfach.
La biblioteca Seminarului Matematic, majoritatea cărţilor ajungeau prin operaţia
de recenzare a lor ı̂n revista Analele Şt. Univ. ”Al.I. Cuza” Iaşi . Anumiţi colegi
urmăreau atent apariţia diverselor cărţi şi trimiteau prompt la editurile ce le publicau
cereri pentru un exemplar care să fie recenzat. După recenzie, cartea rămânea ı̂n
fondul bibliotecii. Dacă se ı̂ntârzia, era posibil ca fondul de reclamă al editurii să
se epuizeze şi atunci cărţile nu mai ajungeau la noi. Îmi amintesc că ı̂ntr-o anumită
perioadă, de această operaţie se ocupa colegul nostru J. Weinstein care era foarte
informat şi foarte prompt ı̂n comandarea cărţilor pentru recenzii. În anumiţi ani se
ajungea până la 300 de cărţi venite pentru recenzii. După ce soseau, cărţile erau
repartizate unor cititori competenţi ı̂n domeniile respective şi aceştia le recenzau. A
fost o perioadă extrem de bună pentru biblioteca noastră, când editurile occidentale
erau foarte generoase cu fondurile de reclamă. Mai târziu, când editurile nu mai
dispuneau de fonduri de reclamă aşa de mari, cărţile nu mai ajungeau la noi ı̂n număr
atât de mare.
Altă modalitate de procurare a cărţilor era achiziţia prin intermediul BCU din
Iaşi. Se lansau diverse comenzi de cărţi, care nu erau primite la recenzii, şi se aştepta
ca BCU să dispună de banii (valuta) pentru a le achiziţiona. Această operaţie dura
destul de mult, uneori şi un an. Îmi amintesc că, ı̂ntr-o anumită perioadă, profesorii
cu notorietate aveau dreptul să achiziţioneze câte o carte din occident, pe an. Ei
cedau acest drept bibliotecii şi se obţineau astfel un număr de cărţi ı̂n plus. Achiziţii
se mai făceau şi ı̂n cadrul Filialei Iaşi a Academiei Române. Ştiu că exista o a10
numită coordonare astfel că achiziţiile făcute la BCU şi la Filială erau ı̂ntotdeauna
complementare.
Majoritatea revistelor erau primite prin schimb cu revista noastră, Analele Şt.
Univ. ”Al.I. Cuza”, Matematică. Mai existau şi abonamente la unele reviste cu care
nu se putea face schimb. Acestea erau puţine şi nesigure - ı̂n anii ı̂n care se operau
restricţii la fondurile valutare anumite abonamente se ı̂ntrerupeau. Astfel, multe
colecţii au fost descompletate. Pentru unele dintre ele s-au făcut eforturi deosebite
de completare prin fotocopiere. În cazul când ı̂ntreruperea era foarte mare, nu s-a
mai putut face nimic. Alte posibilităţi de completare a fondului de publicaţii au
apărut datorită generozităţii unor colegi care, fiind membri ı̂n colectivele de redacţie
ale unor reviste internaţionale, au donat Seminarului Matematic numerele primite de
la redacţiile acestora; aş menţiona aici pe V. Barbu şi C. Corduneanu, dar mai există
şi alţii.
Acum este momentul să-mi exprim o nelinişte şi, parţial, nemulţumire ı̂n legătură
cu orientarea activităţii Seminarului Matematic. În ultimul timp s-a renunţat la
abonamentele clasice preferându-se accesul on-line. Pe de o parte acest fapt constituie
un avantaj: se obţine acces la mai multe reviste, accesăm informaţia foarte rapid, fără
a mai căuta volumul ı̂n bibliotecă, informaţiile noi ajung mai repede la cititori. Spre
exemplu, prin accesul on-line la Springer Link se pot consulta circa 190 reviste de
matematică (numărul total de reviste din diverse domenii ce pot fi accesate este mult
mai mare, circa 1600 reviste). Tot aşa, prin accesul la Science Direct se pot consulta
circa 1800 reviste publicate, ı̂n principal de la editura Elsevier. Pe de altă parte, ı̂n
bibliotecă nu mai rămâne nimic! În cazul ı̂n care accesul on-line ı̂ncetează, pierdem
toate informaţiile la care aveam anterior acces. Cred că trebuie reflectat la aceste
aspecte şi trebuie găsită o modalitate de a păstra informaţiile ı̂n bibliotecă.
Prof. dr. Vasile OPROIU
11
Rigla şi compasul
Gabriel POPA1
Abstract. The two instruments accepted by the ancient Greeks for performing geometric constructions, if separately used, are not equally powerful. The compasses alone can accomplish all
the constructions able to be performed by means of the rule and the compasses together (Mohr Mascheroni), while the rule alone cannot do it (Hilbert). These results are presented in this Note,
with some clearing up brought to the proof of reference [1].
Keywords: circle, cone, rule, compasses.
MSC 2000: 51M15.
1. În problemele de construcţii geometrice este permisă, ı̂n general, utilizarea a
două instrumente: rigla şi compasul. Aceste instrumente sunt considerate ca fiind
ideale; ele trasează dreptele şi cercurile exact, grosimea liniei de creion şi orice alte
aproximări nefiind luate ı̂n considerare.
Rigla este presupusă ca fiind infinită, fără gradaţii pe ea. Ea poate fi folosită
pentru a trasa dreapta ce trece prin două puncte date (ı̂n sensul determinării oricărui
punct al acesteia). Nu o putem utiliza pentru a măsura distanţe ı̂ntre puncte.
Date O, P două puncte ı̂n plan, compasul poate fi utilizat pentru a trasa cercul
de centru O şi care trece prin P (ı̂n sensul determinării oricărui punct al acestuia).
Compasul este considerat ca fiind nerigid: odată ce l-am ridicat de pe hârtie, el se
ı̂nchide, altfel spus nu putem ”transporta” distanţa cuprinsă ı̂ntre vârfurile sale.
În orice problemă de construcţii geometrice, se porneşte de la o mulţime dată S de
puncte ale planului. Putem obţine puncte noi cu ajutorul riglei şi compasului aşa cum
am văzut anterior, precum şi prin următoarele trei operaţii, numite fundamentale:
• determinarea punctului de intersecţie a două drepte;
• determinarea punctelor de intersecţie a unei drepte cu un cerc;
• determinarea punctelor de intersecţie a două cercuri.
Definiţie. Spunem că o problemă de construcţie este rezolvabilă cu rigla şi compasul dacă o putem reduce la o succesiune finită de operaţii alese dintre cele trei
operaţii fundamentale.
Scopul acestui demers este prezentarea posibilităţilor de folosire a acestor două
instrumente. Rezultatele principale sunt date de teoremele 2, 4 şi 5 de mai jos.
2. Ne propunem mai ı̂ntâi să arătăm că putem ı̂nlocui compasul nerigid cu un
compas rigid (care, ı̂n plus faţă de cel nerigid, poate ”transporta” lungimea unui
segment, deci nu se ı̂nchide automat după utilizare). Este adevarată următoarea
Teoremă. Toate construcţiile care pot fi realizate cu rigla şi compasul rigid pot fi
realizate cu rigla şi compasul, ı̂n sensul precizat la 1.
Demonstraţie. Este suficient să dăm un procedeu de construcţie a unui segment
congruent cu un segment dat şi având un capăt fixat, folosind doar rigla şi compasul
nerigid (altfel spus, să arătăm cum se poate transporta un segment). Pentru aceasta,
1 Profesor,
Colegiul Naţional, Iaşi
12
fie [AB] un segment dat şi [M M ′ o semidreaptă dată; dorim să găsim unicul punct
N ∈ [M M ′ pentru care [M N ] ≡ [AB].
Cercurile de centre A şi M şi care trec prin M , respectiv prin A, se intersectează
ı̂n două puncte; fie X unul dintre ele. Avem că △AXM este echilateral. Trasăm
cercul de centru A care trece prin B; acesta intersectează semidreapta [AX ı̂ntr-un
punct C. Deosebim două situaţii:
a) C este ı̂ntre A şi X. Fie cercul de centru X care trece prin C şi fie P punctul
de intersecţie dintre acesta şi segmentul [XM ]. Există un asemenea punct, ı̂ntrucât
XP = XC = XA − AC < XA = XM . Desenăm cercul de centru M şi care
trece prin P ; acesta intersectează semidreapta [M M ′ ı̂ntr-un punct N şi avem că
M N = M P = M X − P X = AX − CX = AC = AB, deci N este punctul căutat.
b) X este ı̂ntre A şi C. Construcţia curge la fel, ı̂nsă punctul P nu se va mai afla
pe segmentul [M X], ci pe semidreapta opusă lui [XM .
Observaţie. În cele ce urmează, vom folosi exprimări de genul: ”fie cercul de
centru O şi rază AB”, unde atât A cât şi B sunt diferite de O; aceste construcţii sunt
permise de teorema precedentă.
3. Dorim să arătăm ı̂n continuare că un compas rigid poate realiza singur toate
construcţiile posibil a fi efectuate cu rigla şi compasul. Calea urmată este, ı̂n linii
mari, cea prezentată ı̂n [1], unele afirmaţii directe de acolo fiind justificate mai riguros.
Demonstraţia clasică, folosind inversiunea, poate fi găsită, spre exemplu, ı̂n [2], pp.2629.
Începem prin a indica algoritmi pentru trei construcţii importante.
(i) Construcţia simetricului unui punct dat faţă de alt punct dat.
Presupunem date două puncte A şi B şi
fie a = d(A, B). Desenăm cercul (C1 ) de
centru A şi care trece prin B, apoi cercul
(C2 ) de centru B şi care trece prin A.
Razele celor două cercuri sunt ambele a,
iar distanţa centrelor este, de asemenea,
a. Deoarece a < a + a, conform teoremei
celor două cercuri, rezultă că (C1 ) şi (C2 )
au ı̂n comun două puncte P şi Q, aflate
de o parte şi de alta a dreptei AB. În
13
÷ ) = m(AQ)
÷ = 60◦
plus, cum △P AB şi △QAB sunt echilaterale, avem că m(AP
(arcele sunt gândite ı̂n cercul (C2 )).
Construim acum cercul (C3 ) de centru Q, care trece prin P . Cum raza lui (C3 )
este P Q < 2AB, urmează că (C3 ) şi (C2 ) au ı̂n comun două puncte; fie A′ al doilea
dintre ele. Deoarece ı̂n cercul (C2 ) coardele [P Q] şi [QA′ ] sunt congruente, avem că
÷ şi QA
ø′ sunt egale. Atunci:
şi arcele QP
ú′ ) = m(AQ)
÷ + m(QA
ø′ ) = m(AQ)
÷ + m(P
÷
m(AQA
Q) = 60◦ + 120◦ = 180◦ ,
deci punctele A şi A′ sunt diametral opuse ı̂n cercul (C2 ), altfel spus A′ este simetricul
lui A faţă de B pe care ı̂l căutam.
(ii) Construcţia mijlocului unui segment dat. Fie A, B două puncte; aflăm ca
mai sus simetricul A′ al lui A faţă de B.
Trasăm cercurile (C1 ) şi (C2 ), de centre A,
respectiv A′ şi care trec prin B, respectiv A.
Dacă a = AB, razele celor două cercuri sunt a
şi 2a, iar distanţa centrelor este 2a. Sunt verificate ipotezele teoremei celor două cercuri şi
fie atunci {P, Q} = (C1 ) ∩ (C2 ). Trasăm acum
cercurile (C3 ) şi (C4 ), de centre P , respectiv Q
şi care trec prin A. Deoarece distanţa centrelor
este P Q < 2AB, urmează că (C3 ) şi (C4 ) au
ı̂n comun două puncte; fie M al doilea dintre
ele. Vom arăta că M este mijlocul căutat al
segmentului [AB].
Se observă uşor că patrulaterul P AQM este romb, deci P Q⊥AM. Pe de altă parte,
÷
A este mijlocul arcului P
Q ı̂n cercul (C2 ), deci P Q⊥AA′ . De aici, punctele A, M, A′ şi
B sunt toate coliniare. Triunghiurile A′ AP şi P AM sunt isoscele: A′ A = A′ P = 2a ca
raze ı̂n (C2 ), P A = P M = a ca raze ı̂n (C3 ) şi au un unghi, ∠P AM, comun. Urmează
că ele sunt asemenea, raportul de asemănare fiind 2 : 1. Atunci P A = 2AM , deci
1
1
AM = AP = a.
2
2
(iii) Construcţia piciorului perpendicularei coborâtă dintr-un punct P pe o dreaptă
AB. Fie A, B două puncte ı̂n plan, iar P un punct necoliniar cu ele. Trasăm cercurile (C1 ) şi (C2 ), de centre A,
respectiv B şi care trec prin P . Fie Q al doilea punct de
intersecţie al acestor cercuri; este clar că Q este simetricul lui P faţă de dreapta AB. Atunci mijlocul M al segmentului [P Q], care poate fi determinat ca ı̂n construcţia
precedentă, este piciorul perpendicularei din P pe [AB].
4. Teoremă (Mohr–Mascheroni). Orice construcţie geometrică realizabilă cu rigla şi compasul se
poate efectua folosind doar compasul rigid.
Demonstraţie. Vom considera că o dreaptă este determinată prin două puncte
ale sale; pentru a afla un alt punct al dreptei, trebuie să indicăm un procedeu de
construcţie a lui folosind compasul. Pentru a demonstra teorema, trebuie să arătăm
14
cum pot fi realizate cele trei operaţii fundamentale. Evident, putem limita discuţia
la primele două operaţii.
(i) Aflarea punctelor de intersecţie dintre un cerc şi o dreaptă. Presupunem că
aceste puncte există şi dorim să le determinăm ca intersecţii de cercuri. În cazul ı̂n
care, pe parcursul construcţiei, vom avea cercuri fără puncte comune, ı̂nseamnă că
dreapta considerată este exterioară cercului iniţial. Deosebim două situaţii:
a) Dreapta nu trece prin centrul cercului. Fie (C) un cerc dat de centru O,
iar A, B două puncte astfel ı̂ncât O ∈
/ AB.
Aflăm simetricul O′ al punctului O faţă de
dreapta AB, ca ı̂n construcţia precedentă.
Trasăm apoi cercul (C ′ ), de centru O′ şi
având aceeaşi rază ca şi cercul (C). Cum
AB este axă de simetrie a figurii obţinute,
urmează că AB ∩ (C) = (C) ∩ (C ′ ), de
unde construcţia punctelor de intersecţie dintre AB şi (C).
b) Dreapta conţine centrul cercului. Fie
(C) un cerc dat de centru O şi rază R, iar
A un punct ı̂n plan. Dorim să determinăm
punctele comune pentru (C) şi OA. Fie M ∈ (C) oarecare. Conform a), putem
determina N – al doilea punct de intersecţie a lui (C) cu AM . Cu vârful compasului ı̂n
M , apoi ı̂n N şi păstrând aceeaşi deschidere, determinăm un punct O′ pe mediatoarea
segmentului [M N ] şi construim un cerc (C1 ) de centru O′ , care să aibă raza mai mare
decât R.
Fie [P Q] o coardă a lui (C1 ) de lungime 2R, posibil de determinat
conform 3.(i). Aflăm B – punct de intersecţie
al dreptei P Q cu cercul (C2 ) de centru O′ şi
care trece prin A, folosind a). Ca la 3.(ii),
fie O′′ mijlocul segmentului [P Q], iar (C3 )
cercul de centru O′′ care trece prin P . Intersectăm acest cerc cu cercul de centru B
şi rază AN ; fie S unul dintre punctele de
intersecţie. Determinăm acum X, Y pe (C),
prin intersecţii de cercuri, astfel ı̂ncât [N X] ≡
[SP ], [N Y ] ≡ [SQ]. Vom arăta că X, Y sunt
punctele căutate.
Deoarece cercurile (C) şi (C3 ) sunt congruente iar [N X] ≡ [SP ], [N Y ] ≡ [SQ], urmează
că △N XY ≡ △SP Q, de unde [XY ] ≡ [P Q].
Însă [P Q] este diametru ı̂n (C3 ), deci [XY ] va fi
diametru ı̂n (C), adică X, O, Y vor fi coliniare.
Rămâne să demonstrăm că A ∈ XY .
Punctele A şi B sunt situate pe cercul (C2 ), concentric cu (C1 ) şi atunci ele vor
avea aceeaşi putere faţă de (C1 ), adică AM · AN = BP · BQ. Dacă {T } = BS ∩ (C3 ),
obţinem că BP · BQ = BT · BS, de unde AM · AN = BT · BS. Cum [AN ] ≡ [BS],
15
rezultă că [AM ] ≡ [BT ], deci [M N ] ≡ [T S]. Însă [M N ] şi [T S] sunt coarde ı̂n cercuri
ø
ö şi apoi XM
ø = T
÷
egale, deci M
N = ST
P , adică ∠XN M ≡ ∠P ST . Urmează că
△XN A ≡ △P SB şi de aici ∠AXN ≡ ∠BP S. Pe de altă parte, ∠N XY ≡ ∠SP Q,
deci m(∠AXN ) + m(∠N XY ) = m(∠BP S) + m(∠SP Q) = 180◦ , i.e. A ∈ XY , adică
ceea ce doream să dovedim.
(ii) Aflarea punctului de intersecţie a două drepte. Fie AB şi A′ B ′ două drepte, ı̂n
sensul că avem date perechile de puncte (A, B) şi (A′ , B ′ ). Folosind 3.(iii), construim
piciorul L al perpendicularei din B ′ pe AB, apoi piciorul N al perpendicularei din L pe
A′ B ′ . Dacă N = B ′ , atunci AB ∩ A′ B ′ ̸= ∅. Dacă nu putem determina N , atunci AB
şi A′ B ′ sunt drepte perpendiculare, concurente ı̂n L.
Presupunem determinate L ̸=
N şi fie P punctul comun celor
două drepte. P este bine determinat de lungimea l a segmentului B ′ P , ı̂ntrucât odată cunoscută aceasta, intersectăm cercul
de centru B ′ şi rază l cu drepta
A′ B ′ . Aplicând teorema catetei
ı̂n △LB ′ P , obţinem că
(1)
B ′ L2 = B ′ N · B ′ P = B ′ N · l.
Determinăm simetricul B ′′ al lui B ′ faţă de L şi construim un cerc având centrul
pe mediatoarea segmentului [B ′ B ′′ ], de rază suficient de mare. Prin intersecţii de
cercuri, fixăm D pe acest cerc astfel ı̂ncât [DL] ≡ [B ′ N ], apoi fie E punctul ı̂n care
DL taie cercul. Din puterea punctului L,
(2)
B ′ L2 = B ′ L · LB ′′ = LD · LE = B ′ N · LE.
Comparând (1) şi (2), rezultă că LE = l, ceea ce ı̂ncheie demonstraţia.
5. În final, vom arăta că rigla este un instrument mai puţin puternic decât compasul, ı̂n sensul că rigla singură nu poate realiza toate construcţiile geometrice posibil
a fi efectuate cu rigla şi compasul, ı̂n timp ce compasul singur poate realiza toate
aceste construcţii. Avem nevoie de următorul rezultat, a cărui demonstraţie poate fi
găsită, de exemplu, ı̂n [5], pp. 235-238:
Lemă. Fie un con oblic de vârf V , având drept bază ı̂n planul (P ) cercul (C).
Fie [AB] diametrul bazei pentru care (V AB)⊥(P ), iar (P ′ ) un plan perpendicular
pe (V AB), care ı̂l intersectează după dreapta (A′ B ′ ), cu A′ ∈ V A, B ′ ∈ V B. Dacă
∠V A′ B ′ ≡ ∠V BA, atunci (P ′ ) intersectează conul după un cerc.
Putem atunci demonstra următoarea
Teoremă (Hilbert). Nu orice construcţie geometrică realizabilă cu rigla şi compasul poate fi efectuată folosind numai rigla.
Demonstraţie. Dat un cerc ı̂n plan, putem să-i aflăm centrul folosind rigla
şi compasul (trasăm mediatoarele a două laturi ale unui triunghi ı̂nscris ı̂n cerc şi
16
considerăm intersecţia acestora); vom arăta că această construcţie nu poate fi realizată
numai cu rigla. Să presupunem prin absurd că există un anumit mod de a găsi centrul
unui cerc folosind numai rigla. O transformare geometrică prin care cercul dat este
dus ı̂ntr-un cerc, iar orice dreaptă este transportată ı̂ntr-o dreaptă, ar face ca ı̂n figura
transformată a construcţiei presupuse, imaginile dreptelor care iniţial se intersectau
ı̂n centrul cercului dat, să se intersecteze ı̂n centrul cercului nou obţinut. Vom arăta
ı̂nsă ca o anumită proiecţie conică duce dreptele ı̂n drepte, cercul dat ı̂ntr-un cerc, ı̂nsă
nu face să se corespundă şi centrele celor două cercuri; obţinem astfel o contradicţie
care va ı̂ncheia demonstraţia.
Fie (C) un cerc de centru O ı̂n planul (P ), iar V un punct astfel ı̂ncât V O să nu fie
perpendiculară pe (P ). Fie (P ′ ) un plan ca ı̂n ipoteza lemei şi considerăm proiecţia
conică a planului (P ) pe planul (P ′ ). Este suficient să mai arătăm că proiecţia lui
O nu este mijlocul O′ al segmentului [A′ B ′ ]. Să presupunem că V A > V B; dacă
Õ
V U este bisectoarea unghiului AV
B, rezultă că AU > U B, deoarece bisectoarea
determină pe latura pe care cade segmente proporţionale cu laturile unghiului din
care pleacă. Pe de altă parte, din V A > V B rezultă că m(∠V BA) > m(∠V AB),
deci m(∠V A′ B ′ ) > m(V B ′ A′ ), de unde V B ′ > V A′ . Cum V U ′ este bisectoare ı̂n
△V A′ B ′ , unde {U ′ } = V U ∩ A′ B ′ , deducem că U ′ B ′ > U ′ A′ . În concluzie, punctele
O şi O′ , mijloacele segmentelor [AB] şi respectiv [A′ B ′ ], sunt separate de dreapta V U
şi deci ele nu pot coincide.
Notăm, ı̂n ı̂ncheiere, că dacă pe foaia pe care se realizează construcţia este desenat
un cerc oarecare, ı̂mpreună cu centrul său, atunci putem efectua numai cu rigla (şi
folosindu-ne de cercul dat) toate construcţiile realizabile cu rigla şi compasul (teorema
Poncelet-Steiner, demonstrată, de exemplu, ı̂n [3], pp. 98-99).
Bibliografie
1. N. Hungerbühler - A Short Elementary Proof of the Mohr-Mascheroni Theorem,
A.M.M. 101 (1994), pp.784-787.
2. H. Lebesgue - Leçons sur les constructiones géométriques, Gauthier-Villars, 1950.
3. G.E. Martin - Geometric constructions, Springer-Verlag, 1998.
4. E. Moise - Geometrie elementară dintr-un punct de vedere superior, E.D.P., 1980.
5. M.H. Rademacher, O. Toeplitz - Despre numere şi figuri, Ed. Ştiinţifică, 1968.
Vizitaţi pagina web a revistei:
http://www.recreatiimatematice.ro
17
O inegalitate ponderată cu medii
Gheorghe CIORESCU, Adrian SANDOVICI
1
Abstract. A refinement of the inequality of the means, ma ≥ mg , is given by inequalities (2)
and (5), with the condition p ≥ (n − 1)q.
Keywords: arithmetic mean, geometric mean, harmonic mean, Sturm′ s method.
MSC 2000: 97D99.
Considerăm n ∈ N, n ≥ 2, şi numerele strict pozitive ai , 1 ≤ i ≤ n. Notăm cu
ma , mg , mh mediile aritmetică, geometrică şi respectiv armonică ale acestor numere.
Scopul acestei note este de a demonstra o inegalitate de tipul
p · ma + q · mh ≥ (p + q) · mg ,
(1)
cu p şi q numere reale strict pozitive. Observăm că (1) poate fi privită ca o rafinare
a inegalităţii ma ≥ mg .
Propoziţia 1. Are loc inegalitatea
(n − 1)ma + mh ≥ n · mg ,
(2)
cu egalitate dacă şi numai dacă a1 = a2 = . . . = an .
Demonstraţie. Avem
(n − 1)ma + mh = n ·
È
ma + . . . + ma + mh
n
≥ n mn−1
· mh .
a
n
Ca urmare, este suficient să arătăm că are loc inegalitatea
È
n
(3)
Să notăm xi = ai /
n
X
mn−1
· mh ≥ mg sau mn−1
· mh ≥ mng .
a
a
n
X
ak , 1 ≤ i ≤ n, şi să observăm că xi ∈ (0, 1), 1 ≤ i ≤ n, şi
k=1
xi = 1. După calcule elementare, inegalitatea (3) se rescrie sub forma
i=1
(4)
Sn (x1 , x2 , . . . , xn ) =
X
1≤j1 <...<jn−1 ≤n
n−1
Y
!
xjk
≤
k=1
1
nn−2
.
Vom demonstra această inegalitate folosind metoda lui Sturm. Presupunem că x1
şi x2 sunt astfel ı̂ncât x1 < x2 . Considerăm numerele x′1 = x1 + ε şi x′2 = x2 − ε aşa
ca x1 < x′1 < x′2 < x2 şi x′k = xk , 3 ≤ k ≤ n. Este clar că
n
X
i=1
1 Profesori,
Colegiul Naţional ”Petru Rareş”, Piatra Neamţ
18
x′i = 1 şi x′i ∈ (0, 1),
1 ≤ i ≤ n. Vom avea
„
X
n−3
Y
1≤j1 <...<jn−3 ≤n
k=1
Sn (x′1 , x′2 , . . . , x′n ) = x′1 x′2
„
!Ž
x′jk
X
n−3
Y
1≤j1 <...<jn−3 ≤n
k=1
= (x1 x2 + ε(x2 − x1 − ε))
„
+ (x′1 + x′2 )
+ (x1 + x2 )
X
n−3
Y
1≤j1 <...<jn−3 ≤n
k=1
= Sn (x1 , x2 , . . . , xn ) + ε(x2 − x1 − ε)
x′j =
j=3
!Ž
xjk
n
Y
!Ž
xjk
n
Y
xj =
j=3
>
> Sn (x1 , x2 , . . . , xn ).
Rezultă că suma Sn (x1 , x2 , . . . , xn ) ı̂şi atinge maximul atunci când numerele xi sunt
1
egale, adică pentru x1 = x2 = . . . = xn = . De aici, se obţine inegalitatea (4) şi, ca
n
urmare, (2) este dovedită.
În sfârşit, are loc egalitatea ı̂n (2) dacă şi numai dacă mediile ma , mg şi mh sunt
egale, adică dacă şi numai dacă numerele ai , 1 ≤ i ≤ n, sunt egale.
Observaţie. În membrul ı̂ntâi al inegalităţii (2) nu putem lua mai mulţi termeni
egali cu mh , adică nu este adevărată inegalitatea (n − k)ma + k · mh ≥ n · mg pentru
k ≥ 2. Un exemplu ı̂n acest sens este următorul: n = 3, a1 = 1, a2 = 3 şi a3 = 9.
O formă mai generală a inegalităţii (2) este dată de
Propoziţia 2. Are loc inegalitatea
(5)
p · ma + q · mh ≥ (p + q)mg ,
unde p, q sunt numere reale strict pozitive ce verifică condiţia p ≥ (n − 1)q. În (5) are
loc egalitate dacă şi numai dacă numerele ai (1 ≤ i ≤ n) sunt egale.
Demonstraţie. Ţinând cont de (2), avem
p · ma + q · mh = [p − (n − 1)q]ma + q[(n − 1)ma + mh ] ≥
[p − (n − 1)q]mg + qn · mg = (p + q)mg ,
de unde (5). Ultima afirmaţie din enunţ se stabileşte cu uşurinţă.
Vom ı̂ncheia cu câteva aplicaţii directe ale rezultatului de mai sus.
Aplicaţia 1 (Problema 26013, G.M.-7-8/2008). Să se arate că pentru orice numere reale a, b, c > 0 are loc inegalitatea
a+b+c+
3
1
a
+
1
b
+
1
c
√
3
≥ 4 abc.
Soluţie. Pentru n = 3, p = 3 şi q = 1 luaţi ı̂n Propoziţia 2, obţinem rezultatul
problemei.
19
Aplicaţia 2. Fie a, b, p, q ∈ R∗+ cu p ≥ 2q. Rezolvaţi ı̂n mulţimea R∗+ inecuaţia
(6)
p(x2 + x + 1) +
9qx2
≤ 3(p + q)x.
+x+1
x2
Soluţie. Pentru n = 3, a = x2 , b = x şi c = 1, relaţia (5) devine
(7)
p(x2 + x + 1) +
9qx2
≥ 3(p + q)x.
x2 + x + 1
Deci, ı̂n relaţiile (6) şi (7) vom avea egalitate. Conform Propoziţiei 2, numerele x2 , x
şi 1 sunt egale. În consecinţă, x = 1 este unica soluţie a inecuaţiei (6).
Aplicaţia 3. Rezolvaţi ı̂n R∗+ ecuaţia
(8)
√
2010x
2009
(2009 + x) +
= 2010 2010 x.
2010
2009x + 1
Soluţie. Luând ı̂n (2) n = 2010, x1 = x2 = . . . = x2009 = 1 şi x2010 = x, avem
√
2009
2010x
(2009 + x) +
≥ 2010 2010 x.
2010
2009x + 1
Cum (8) cere egalitate ı̂n relaţia precedentă, rezultă că x1 = x2 = . . . = x2010 ; deci,
x = 1.
Diofant din Alexandria (sec. III d.Hr.)
Despre viaţa lui Diofant nu se cunoaşte aproape nimic; nici data şi nici locul
naşterii. Se consideră că a trăit, cel mai probabil, ı̂n jurul anului 250 d.Hr. Şi-a
desfăşurat activitatea la Alexandria şi a scris un tratat ı̂n 13 volume, Aritmetica,
care poate fi comparat ca importanţă cu Elementele lui Euclid (tot ı̂n 13 volume).
Numai şase dintre aceste volume nu au fost pierdute şi au devenit sursă de inspiraţie
pentru matematicienii Renaşterii. Pe marginea cărţii a II-a a lui Diofant, matematicianul francez Pierre Fermat a notat celebra sa teoremă, Marea Teoremă a lui
Fermat.
Durata vieţii lui Diofant se poate afla rezolvând o problemă a sa, care a fost, se
pare, gravată pe piatra lui funerară.
Dumnezeu i-a ı̂ngăduit să fie copil o şesime din viaţa sa şi, adăugând la aceasta a
douăsprezecea parte, i-a acoperit obrazul cu puf gingaş, i-a ı̂mpărtăşit lumina sfântă a
căsniciei după a şaptea parte a vieţii, iar după cinci ani de căsătorie i-a oferit un fiu.
Dar vai! nefericit copilul născut târziu; după ce a atins o jumătate din ı̂ntreaga viaţă
a tatălui, copilul a fost răpit de soarta necruţătoare. După ce şi-a alinat suferinţa,
adâncindu-se ı̂n ştiinţa numerelor vreme de patru ani, şi-a dat sufletul.
Întrebare. Câţi ani a trăit Diofant?
N.B. Răspunsul se găseşte la pagina 24.
20
Inegalitatea lui Jensen pentru funcţii J-convexe
ı̂n raport cu medii cvasiaritmetice
Florin POPOVICI 1
Abstract. In this Note an elementary proof is given for Jensen′ s inequality related to a (M, N )J-convex function (Definition 3), in the case when M and N are quasi-arithmetic means (Definition
2).
Keywords: J-convex function, quasi-arithmetic mean, (M, N )-J-convex function.
MSC 2000: 52A40.
Înlocuind ı̂n definiţia funcţiilor J-convexe, cele două medii aritmetice cu două
medii oarecare M şi N G. Aumann ([1], pag. 4), ı̂n anul 1933, extinde noţiunea de
funcţie J-convexă prin noţiunea de funcţie J-convexă ı̂n raport cu perechea ordonată
de medii (M, N ). Inegalitatea lui Jensen, adaptată pentru funcţiile J-convexe ı̂n
raport cu perechi ordonate de medii (M, N ) are loc pentru o clasă largă de medii, care
include mediile cvasiaritmetice. Demonstraţia de mai jos adaptează raţionamentul
prezentat de noi ı̂n [3]; credem că este nouă. În particular, din inegalitatea lui Jensen
astfel generalizată, se obţin diferite inegalităţi clasice.
Definiţia 1. Fie I ⊂ R un interval. Un şir de funcţii M = (Mn )n≥2 se numeşte
medie pe I dacă pentru orice n ∈ N, n ≥ 2, funcţia Mn : I n → I satisface condiţia
(1)
min{xi |i = 1, n} ≤ Mn (x1 , . . . , xn ) ≤ max{xi |i = 1, n}, ∀xi ∈ (0, ∞), i = 1, n;
numărul Mn (x1 , . . . , xn ) se numeşte media numerelor x1 , . . . , xn .
Definiţia 2. Fie I, J ⊂ R două intervale. Fie φ : I → J o funcţie bijectivă strict
monotonă. Considerăm şirul de funcţii M = (Mn )n≥2 , Mn : I n → I, ∀n ≥ 2, definit
prin
(2)
Mn (x1 , . . . , xn ) = φ−1

‹
φ(x1 ) + . . . + φ(xn )
, ∀x1 , . . . , xn ∈ I.
n
Evident, M este o medie pe I. Media M se numeşte medie cvasiaritmetică.
Observaţii. 1) În cazul particular ı̂n care I = J şi φ = 1I , (2) este media
x1 + . . . + xn
aritmetică A = (An )n≥2 , unde An (x1 , . . . , xn ) =
, ∀x1 , . . . , xn ∈ I.
n
1
În cazul particular ı̂n care I = J = (0, ∞) şi φ(x) = , ∀x ∈ (0, ∞), (2) este media
x
n
armonică H = (Hn )n≥2 , unde Hn (x1 , . . . , xn ) = 1
1 , ∀x1 , . . . , xn ∈ (0, ∞).
x1 + . . . + xn
În cazul particular ı̂n care I = (0, ∞), J = R şi φ(x) = ln x, ∀x ∈ (0, ∞), (2) este
√
media geometrică G = (Gn )n≥2 , unde G(x1 , . . . , xn ) = n x1 · . . . · xn , ∀x1 , . . . , xn ∈
(0, ∞).
2) Dacă M = (Mn )n≥2 este o medie cvasiaritmetică pe I, atunci media M este
strict crescătoare, adică pentru orice n ∈ N, n ≥ 2, funcţia Mn este strict crescătoare
ı̂n raport cu fiecare din variabilele x1 , . . . , xn .
1 Profesor
dr., Colegiul Naţional de Informatică ”Gr. Moisil”, Braşov
21
Definiţia 3. Fie I1 , I2 ⊂ R două intervale date. Fie M o medie pe I1 şi fie N o
medie pe I2 . O funcţie f : I1 → I2 se numeşte convexă ı̂n raport cu perechea ordonată
de medii (M, N ) (pe scurt, f este (M, N ) − J-convexă), dacă
f (M2 (x, y)) ≤ N2 (f (x), f (y)), ∀x, y ∈ I1 .
(3)
Observaţii. 1) În cazul particular ı̂n care M este media aritmetică pe I1 şi N
este media aritmetică pe I2 , (3) devine
f
x + y
2
≤
f (x) + f (y)
, ∀x, y ∈ I1 ;
2
deci funcţiile (A, A) − J-convexe sunt funcţiile J-convexe.
2) Dacă M şi N sunt medii cvasiaritmetice, atunci condiţia (3) devine

(4)
f φ−1

φ(x) + φ(y)
2
‹‹
≤ ψ −1

‹
ψ(f (x)) + ψ(f (y))
, ∀x, y ∈ I1 .
2
Teorema 1. Fie I1 , I2 , J1 , J2 ⊂ R patru intervale date. Fie φ : I1 → J1 şi
ψ : I2 → J2 două bijecţii strict crescătoare. Fie M = (Mn )n≥2 media cvasiaritmetică
determinată de funcţia φ, şi fie N = (Nn )n≥2 media cvasiaritmetică determinată de
funcţia ψ. Dacă f : I1 → I2 este o funcţie (M, N ) − J-convexă, atunci, pentru orice
n ∈ N, n ≥ 2, şi orice x1 , . . . , xn are loc inegalitatea lui Jensen generalizată
f (Mn (x1 , . . . , xn )) ≤ Nn (f (x1 ), . . . , f (xn )).
(5)
Demonstraţie. Stabilim (5) prin inducţie. Pentru n = 2, (5) are loc conform
ipotezei. Fie n ∈ N, n ≥ 2, o valoare pentru care are loc (5). Fie a, b ∈ I1 . Notăm
c = Mn+1 (a, . . . , a, b) şi d = Mn+1 (a, b, . . . , b). Avem
| {z }
| {z }
n
c=φ
=φ
n
−1

−1
nφ(a) + φ(b)
n+1
‹
=φ
−1
(n2 − 1)φ(a) + φ(a) + nφ(b)
n(n + 1)
))
(n − 1)φ(a) + φ(φ−1 ( φ(a)+nφ(b)
n+1
=
!
= Mn (a, . . . , a, d),
| {z }
n
n−1
deci c = Mn (a, . . . , a, d). În mod analog, obţinem d = Mn (c, b, . . . , b). Rezultă că
| {z }
| {z }
n−1
n−1
avem c = Mn (a, . . . , a, Mn (c, b, . . . , b)).
| {z }
| {z }
n−1
n−1
Ţinând cont de monotonia mediei N şi de ipoteza inductivă, obţinem
f (c)=f (Mn (a, . . . , a, Mn (c, b, . . . , b))≤Nn (f (a), . . . , f (a), Nn (f (c), f (b), . . . , f (b)))) =
| {z }
| {z }
n−1
n−1
‚
ψ
−1
|
{z
|
}
n−1
ψ(f (c))+(n−1)ψ(f (b))
n
(n − 1)ψ(f (a)) +
n
22
{z
n−1
Œ
.
}
De aici, obţinem succesiv
ψ(f (c)) + (n − 1)ψ(f (b))
⇐⇒
n
⇐⇒ (n + 1)ψ(f (c)) ≤ nψ(f (a)) + ψ(f (b)) ⇐⇒
nψ(f (c)) ≤ (n − 1)ψ(f (a)) +

‹
nψ(f (a)) + ψ(f (b))
⇐⇒
n+1
f (Mn+1 (a, . . . , a, b)) ≤ Nn+1 (f (a), . . . , f (a), f (b)).
f (c) ≤ ψ −1
(6)
| {z }
|
n
{z
}
n
Pentru orice x1 , . . . , xn+1 ∈ I1 avem
‚
−1
Mn+1 (x1 , . . . , xn+1 ) = φ
= φ−1

n)
n φ(x1 )+...+φ(x
+ φ(xn+1 )
n
n+1
Œ
=
‹
nφ(Mn (x1 , . . . , xn )) + φ(xn+1 )
,
n+1
deci
(7)
Mn+1 (x1 , . . . , xn+1 ) = Mn+1 (Mn (x1 , . . . , xn ), . . . , Mn (x1 , . . . , xn ), xn+1 ).
|
{z
}
n
În mod analog, pentru orice y1 , . . . , yn+1 ∈ I2 avem
(8)
Nn+1 (y1 , . . . , yn+1 ) = Nn+1 (Nn (y1 , . . . , yn ), . . . , Nn (y1 , . . . , yn ), yn+1 ).
{z
|
}
n
Ţinând cont de (6), (7) şi (8), de ipoteza inductivă şi de monotonia mediei N ,
rezultă că pentru orice x1 , . . . , xn+1 ∈ I1 avem
(6)
(7)
f (Mn+1 (x1 , . . . , xn+1 )) = f (Mn+1 (Mn (x1 , . . . , xn ), . . . , Mn (x1 , . . . , xn )), xn+1 ) ≤
|
{z
}
n
Nn+1 (f (Mn (x1 , . . . , xn )), . . . , f (Mn (x1 , . . . , xn )), f (xn+1 )) ≤
(8)
≤ Nn+1 (Nn (f (x1 ), . . . , f (xn )), . . . , Nn (f (x1 ), . . . , f (xn )), f (xn+1 )) =
= Nn+1 (f (x1 ), . . . , f (xn+1 )),
deci f (Mn+1 (x1 , . . . , xn+1 )) ≤ Nn+1 (f (x1 ), . . . , f (xn+1 )). Conform principiului inducţiei
matematice rezultă că (5) are loc pentru orice n ∈ N, n ≥ 2.
Observaţii. 1) Evident, inegalitatea (5) generalizează inegalitatea lui Jensen
pentru funcţii J-convexe.
√
2) Deoarece
xy ≤ 12 (x + y), ∀x, y ∈ (0, ∞), rezultă că funcţia f = 1(0,∞) este
(G, A) − J-convexă; conform Teoremei 1, obţinem inegalitatea mediilor
√
n
x1 · . . . · xn ≤
x1 + . . . + xn
, ∀x1 , . . . , xn ∈ (0, ∞), ∀n ≥ 2.
n
23
3) Considerăm funcţia f : (0, ∞) → (0, ∞), definită prin f (x) = 1+x, ∀x ∈ (0, ∞).
Avem
È
È
√
√
f ( xy) = 1 + xy ≤ (1 + x)(1 + y) = f (x)f (y), ∀x, y ∈ (0, ∞),
deci funcţia f este (G, G) − J-convexă; rezultă că are loc inegalitatea lui Huygens
1+
√
n
x1 · . . . · xn ≤
È
n
(1 + x1 ) · . . . · (1 + xn ), ∀x1 , . . . , xn ∈ (0, ∞), ∀n ≥ 2.
4) În [2], inegalitatea lui Jensen generalizată este stabilită pentru funcţii (M, N )−
J-convexe, corespunzător unor clase largi de medii, care includ mediile cvasiaritmetice.
5) În [3], prin aplicarea directă a metodei din demonstraţia Teoremei 1, am prezentat demonstraţii simple pentru inegalitatea mediilor şi pentru inegalitatea lui Huygens.
6) Inegalitatea (5) poate fi stabilită şi pe baza raţionamentului lui Cauchy pentru
dovedirea inegalităţii ı̂ntre mediile aritmetică şi geometrică. Propunem acest exerciţiu
cititorului.
Bibliografie
1.
C.P. Niculescu, L.E. Persson - Convex Functions and Their Applications, A
Contemporary Approach, CMS Books in Mathematics, vol. 23, Springer-Verlag, New
York, 2006.
2. C.P. Niculescu, F. Popovici - Inegalitatea lui Jensen pentru funcţii (M, N ) − Jconvexe ı̂n condiţii generale, va apare.
3. F. Popovici - Asupra inegalităţii Jensen, Recreaţii Matematice, 1/2009, 12-14.
Răspuns la ı̂ntrebarea de la pag. 20.
x
Notând cu x durata vieţii lui Diofant, din problemă rezultă următoarele:
–
6
x
x
perioada copilăriei;
– adolescenţa;
– perioada de dinainte de căsătorie; 5 ani
12
7
x
mai târziu i s-a născut fiul;
– durata vieţii fiului; 4 ani au mai trecut până la
2
moartea sa. Ca urmare, pentru aflarea necunoscutei x trebuie să rezolvăm ecuaţia
x=
Cum ecuaţia se scrie
x
x
x
x
+
+ + 5 + + 4.
6 12 7
2
3
x = 9, rezultă că Diofant a trăit 84 de ani.
28
24
Inegalitatea H ≤ G ≤ A revizitată
Vasile CHIRIAC 1 , Bogdan CHIRIAC 2
Abstract. In this Note, a new proof of the inequality H ≤ G ≤ A is given, together with a
couple of applications of this inequality.
Keywords: arithmetic mean, geometric mean, harmonic mean.
MSC 2000: 97D99.
Fie ai > 0, i = 1, n numere reale; notăm cu A, G, H mediile aritmetică, geometrică
şi respectiv armonică, adică
a1 + a2 + . . . + an
n
√
, G = n a1 · a2 · . . . · an , H =
A=
.
1
1
1
n
+
+ ... +
a1
a2
an
Matematicianul englez Colin Maclaurin (1698-1746), ı̂n lucrarea sa Algebra
apărută postum, ı̂n 1748, a arătat relaţia H ≤ G ≤ A ([2], p.30; [1] p.43, 44).
Se cunosc multe demonstraţii pentru această inegalitate. Ne propunem să dăm o
nouă demonstraţie.
Lemă. Pentru orice x, y > 0 reale şi n ≥ 2 natural, are loc inegalitatea:
xn + y n ≥ xn−1 · y + y n−1 · x
(1)
Demonstraţie. Cum x − y şi xn−1 − y n−1 au acelaşi semn, oricare ar fi x, y > 0,
putem scrie (x − y) · xn−1 − y n−1 ≥ 0, de unde rezultă (1).
Teoremă. Oricare ar fi numerele reale xi > 0 cu i = 1, n şi n ≥ 2, avem:
xn1 + xn2 + . . . + xnn ≥ n · x1 x2 · . . . · xn
(2)
Demonstraţie. Procedăm prin inducţie matematică. Pentru n = 2 se obţine
binecunoscuta inegalitate x21 +x22 ≥ 2x1 x2 . Presupunem că inegalitatea este adevărată
pentru n−1 numere şi o vom demonstra pentru n. Putem scrie următoarele inegalităţi:
· x1
· xn + xn−1
· x2 + x2n−1 · x1
...
xn1 + xnn ≥ xn−1
xn1 + xn2 ≥ xn−1
n
1
1
n−1
n−1
n−1
n
n
n
n
x2 + x3 ≥ x2 · x3 + x3 · x2
...
x2 + xn ≥ x2 · xn + xn−1
· x2
n
............................................................................................
n−1
xnn−1 + xnn ≥ xn−1
· xn−1
n−1 · xn + xn
Adunând membru cu membru şi grupând convenabil găsim:
+ xŠn−1
+ . . . + x€n−1
)+
(n − 1) (xn1 + xn2 €+ . . . + xnn ) ≥ x1 (xn−1
n
2
3
Š
n−1
n−1
n−1
+x2 x1 + x3 + . . . + xn
+ . . . + xn xn−1
+ xn−1
+ . . . + xn−1
1
2
n−1 .
Ţinând seamă de presupunerea făcută, obţinem (n − 1) (xn1 + xn2 + . . . + xnn ) ≥
≥ x1 · (n − 1) x2 x3 · . . . · xn + x2 · (n − 1) x1 x3 · . . . · xn + . . . + xn−1 · (n − 1) x1 x2 · . . . ·
xn−2 · xn + xn · (n − 1) x1 x2 · . . . · xn−1 ,
de unde se deduce inegalitatea de demonstrat pentru n numere.
Consecinţă. Dacă ai > 0, i = 1, n şi n ≥ 2, atunci are loc relaţia H ≤ G ≤ A.
1 Profesor,
2 Student,
Liceul ”V. Alecsandri”, Bacău
Facultatea de matematică, Univ. ”Al. I. Cuza”, Iaşi
25
√
√
√
Demonstraţie. În (2), luând x1 = n a1 , x2 = n a2 , . . . , xn = n an obţinem
1
1
1
A ≥ G, iar pentru x1 = √
, x2 = √
, . . . , xn = √
obţinem G ≥ H.
n a
n a
n a
1
2
n
Aplicaţie. Fie ai > 0, i = 1, n şi n ≥ 4. Să se arate că:
a1 a2 + a2 a3 + a3 a1
a2 a3 + a3 a4 + a4 a2
an−1 an + an a1 + a1 an−1
+
+ ... +
+
a31 + a32 + a33
a32 + a33 + a34
a3n−1 + a3n + a31
+
an a1 + a1 a2 + a2 an
1
1
1
≤
+
+ ... +
.
a3n + a31 + a32
a1
a2
an
1
1
≤
, oricare ar
3
3
+ aj + ak
3a a a
i j k
ai aj + aj ak + ak ai
1
1
1
1
fi i, j, k = 1, n, i ̸= j ̸= k =
̸ i. Deci
≤ ·
+
+
şi,
a3i + a3j + a3k
3
ak
ai
aj
sumând, deducem inegalitatea dorită.
Soluţie. Din a3i + a3j + a3k ≥ 3ai aj ak , avem
a3i
Lăsăm ı̂n seama cititorului să demonstreze inegalităţile următoare :
1) Fie a > 0, b > 0 cu proprietatea că a + b = 1. Să se arate că
r
r
6
6
1
32 + 5 +
a
32 +
1
≥ 4.
b5
2) Oricare ar fi numerele reale strict pozitive a1 , a2 , . . . , an , n ≥ 3, avem
a2 + a3
an−1 + an
an + a1
1
a1 + a2
1
1
+ 2
+ ... + 2
+ 2
≤
+
+ ... +
.
a21 + a22
a2 + a23
an−1 + a2n
an + a21
a1
a2
an
3) Fie ai > 0, i = 1, n, astfel ı̂ncât a1 + a2 + . . . + an = 1. Atunci
n 
X
i=1
1
ai +
ai
‹2
n2 + 1
≥
n
2
.
4) Să se arate că ı̂n orice triunghi ABC au loc inegalităţile:
‹2

A
B
C
A
B
C 3
i) a · sin + b · sin + c · sin ≥ 3p tg · tg · tg
;
2
2
2
2
2
2
A
B
C
A
B
C
ii) a · tg + b · tg + c · tg ≥ 6p · tg · tg · tg .
2
2
2
2
2
2
Bibliografie
1. V. Chiriac - Matematică. Fundamentele Algebrei, Editura Sigma, 2007.
2. N. Mihăileanu - Istoria Matematicii, vol. 2, Ed. Şt. şi Enciclop., 1981.
3. - Gazeta Matematică, seriile A, B, 1969-2009.
26
O extensiune a şirului Fibonacci
Petru MINUŢ1 , Cristina SIMIRAD 2
Abstract. A sequence (vn )n∈N , defined by v0 = 0, v1 = 1 and vn+2 = avn+1 + vn , where
a ∈ N∗ , is considered. Properties of this sequence are revealed , some of them being similar to those
of Fibonacci′ s sequence.
Keywords: Fibonacci′ s sequence, matrix, characteristic equation, Binet′ s formula.
MSC 2000: 11B39.
Şirul Fibonacci este şirul (Fn )n∈N determinat de recurenţa:
F0 = 0, F1 = 1, Fn+2 = Fn+1 + Fn , n ∈ N.
(1)
El poate fi definit şi prin egalităţile matriceale:

(2)
1 1
1 0
‹n

‹
Fn+1
=
Fn
Fn
, n ∈ N (cu convenţia F−1 = 1).
Fn−1

‹0

‹
1 1
1 0
=
, ceea ce este adevărat.
1 0
0 1
Apoi, dacă (2) este adevărată pentru n, vom avea

‹n+1 
‹
‹ 
‹ 
‹
1 1
1 1
Fn+1
Fn
Fn+1 + Fn Fn + Fn−1
Fn+2 Fn+1
=
=
=
,
1 0
1 0
Fn
Fn−1
Fn+1
Fn
Fn+1
Fn
deci (2) este adevărată şi pentru n + 1.
Într-adevăr, pentru n = 0 relaţia (2) devine

Ne‹punem problema găsirii şirurilor (vn )n∈N definite cu ajutorul unei matrici A =
a b
, a, b, c, d ∈ N, prin
c d

(3)
a b
c d
‹n

=

Deoarece pentru n = 0 avem
vn+1
vn
a b
c d
v2
1
‹0

=

‹
, n ∈ N.
vn−1
caz, v1 = 1, v0 = 0 şi v−1 = 1, iar egalitatea

‹
vn
‹
1 0
, relaţia (3) ne va da, ı̂n acest
0 1
a b
c d
‹1

=
v2
v1
v1
v0
‹

se scrie
‹
a b
=
c d
1
şi conduce la v2 = a, b = c = 1 şi d = 0. Prin urmare, A este de forma
0

‹
a 1
A=
şi (3) se scrie
1 0

(4)
1 Prof.
‹n
a 1
1 0

v
= n+1
vn
univ., Univ. ”Al.I. Cuza”, Iaşi
Şcoala nr. 10 ”Gh. Brătianu”, Iaşi
2 Profesoară,
27
vn
vn−1
‹
, ∀n ∈ N.
Din faptul că

vn+2
vn+1
‹

‹
vn+1
a 1
= An+1 =
vn
1 0
vn+1
vn
vn
‹
vn−1

=
avn+1 + vn
vn+1
avn + vn−1
vn
‹
rezultă imediat că şirul (vn )n∈N , ce are v0 = 0 şi v1 = 1, satisface relaţia
vn+2 = avn+1 + vn , n ∈ N.
(5)
Numim (vn )n , dat de (5) şi v0 = 0, v1 = 1, şir generalizat al lui Fibonacci. Vom vedea
mai jos că multe proprietăţi ale şirului (Fn )n∈N al lui Fibonacci rămân valabile şi ı̂n
acest caz şi cu aceleaşi demonstraţii ([2], [3]).
Ecuaţia caracteristică ataşată şirului (vn )n∈N este
x2 − ax − 1 = 0,
√
√
1
1
(a + a2 + 4) şi x2 = (a − a2 + 4).
2
2
Să ı̂nlăturăm cazul a = 0, care este banal. Observăm că a2 + 4 nu poate fi pătrat
perfect: pentru a = 1 avem a2 + 4 = 5, iar pentru a ≥ 2 avem a2 < a2 + 4 < (a + 1)2 .
Adoptăm notaţia ϕ = x1 , deci x2 = ϕ (conujugatul lui ϕ). Se ştie ([1], [3]) că termenul
general al şirului (5) este de forma
cu rădăcinile x1 =
n
vn = Aϕn + Bϕ , n ∈ N.
Pentru n = 0 şi n = 1 obţinem sistemul de ecuaţii: A+B = v0 = 0 şi Aϕ+Bϕ = v1 = 1
1
din care deducem A = −B = √
. Înlocuind aceste constante, deducem formula
2
a +4
de tip Binet
vn = √
(6)
1
a2
n
+4
(ϕn − ϕ ), n ∈ N.
Mai ı̂ntâi , vom enumera câteva proprietăţi elementare ale şirului (vn )n∈N :
n
X
1
◦
1
vk = (vn+1 + vn − 1).
a
k=1
Într-adevăr, ţinând seama de (5), avem avk = vk+1 −vk−1 , k = 1, n. Sumând membru
cu membru aceste egalităţi, vom obţine pe cea dorită.
n
X
1
2◦
v2k−1 = v2n .
a
k=1
Avem: av2k−1 = v2k − v2k−2 , k = 1, n. Sumăm membru cu membru.
n
X
1
3◦
v2k = (v2n+1 − 1).
a
k=1
La fel, pornind de la av2k = v2k+1 − v2k−1 , k = 1, n.
2n
X
1
◦
4
(−1)k−1 vk = (v2n − v2n+1 + 1).
a
k=1
28
Într-adevăr,
2n
X
(−1)k−1 vk =
k=1
n
X
v2k−1 −
k=1
n
X
vk =
k=1
1
1
1
v2n − (v2n+1 − 1) = (v2n −
a
a
a
v2n+1 + 1).
5◦
n
X
vk2 =
k=1
1
vn · vn+1 .
a
Observăm că vk vk+1 − vk−1 vk = vk (vk+1 − vk−1 ) = avk2 şi sumăm pentru k = 1, n.
1 2
2
6◦ . vm+n = vm−1 vn + vm vn+1 ; ı̂n particular, v2n = (vn+1
− vn−1
).
a
m
Se poate arăta prin inducţie după n sau ı̂n felul următor: egalitatea A · An = Am+n ,
cu An dat de (4), devine:

vm+1
vm
vm
vm−1
‹
vn+1
vn
vn
vn−1
‹

v
= m+n+1
vm+n
vm+n
vm+n−1
‹
.
Se efectuează produsul matricelor şi se scrie apoi egalitatea elementelor situate pe
linia a doua şi coloana ı̂ntâi.
∞
P
Se numeşte funcţie generatoare a unui şir (un )n∈N funcţia F dată de F (z) =
un z n .
n=0
Teorema 1. Funcţia generatoare a şirului (vn )n∈N este
(7)
F (x) =
x
.
1 − ax − x2
Demonstraţie. Avem:
F (x) = v0 + v1 x + v2 x2 + . . . + vn+2 xn+2 + . . .
−axF (x) = −av0 x − av1 x2 − . . . − avn+1 xn+2 + . . .
−x2 F (x) =
−v0 x2 − . . . −
vn xn+2 + . . .
Sumând, se obţine (1 −ax− x2 )F (x) = v0 + (v1 −av0 )x sau, deoarece v0 = 0 şi v1 = 1,
(1 − ax − x2 )F (x) = x, de unde rezultă (7).
Observaţie. Conform teoremei precedente, şirul (vn )n∈N este şirul coeficienţilor
câtului ı̂mpărţirii polinomului x la 1 − ax − x2 .
Următoarele patru teoreme indică proprietăţi de divizibilitate ale şirului (5).
Teorema 2. Dacă d/n, atunci vd /vn .
Demonstraţie. Fie n = dm. Vom avea:
1
1
n
dm
(ϕn − ϕ ) = √
(ϕdm − ϕ ) =
a2 + 4
a2 + 4
1
d
d
d(m−1)
=√
(ϕd − ϕ )(ϕd(m−1) + ϕd(m−2) ϕ + . . . + ϕ
) = vd M,
2
a +4
vn = √
unde M este un polinom simetric ı̂n ϕ şi ϕ, rădăcinile ecuaţiei x2 −ax−1 = 0. Conform
teoremei fundamentale a polinoamelor simetrice, M va fi un polinom cu coeficienţi
ı̂ntregi ı̂n coeficienţii acestei ecuaţii. Pin urmare, M este un ı̂ntreg şi vd |vn .
29
Teorema poate fi demonstrată şi pe baza relaţiei 6◦ şi procedând prin inducţie
după d.
Teorema 3. Dacă n este număr compus, n ̸= 4, atunci vn este număr compus.
Demonstraţie. v4 = a3 +2a este prim pentru a = 1. Dacă n = n1 n2 , 1 < n1 ≤ n2 ,
cel puţin n2 ≥ 2, deci vn2 > 1, vn2 < vn şi vn2 |vn .
Teorema 5. (vn+1 , vn ) = 1, ∀n ∈ N.
Demonstraţie. (vn+1 , vn ) = (avn + vn−1 , vn ) = (vn , vn−1 ) = . . . = (v1 , v0 ) = 1.
Teorema 6. (vm , vn ) = v(m,n) , ∀m, n ∈ N.
Demonstraţie. Se face pe baza algoritmului lui Euclid şi cu utilizarea proprietăţii
6◦ şi Teoremei 5. Pentru detalii se pot vedea [2], [3].
Încheiem cu o proprietate de aproximare:
Teorema 7. vn este numărul ı̂ntreg cel mai apropiat de termenul de rang n al
1
progresiei geometrice cu termenul de rangul zero egal cu √
şi raţia ϕ.
a2 + 4
Demonstraţie. Într-adevăr, avem:
n
ϕn n
n
vn − √ ϕ
= √ 1
(ϕ − ϕ ) − √
=
a2 + 4 a2 + 4
a2 + 4 =√
√
( a2 + 4 − a)n
2n
1
|ϕ|n
√
=
< √
< .
2
a2 + 4
2n a2 + 4
2n a2 + 4
Bibliografie
1. A. Markuşevici – Şiruri recurente, Editura Tehnică, Bucureşti, 1954.
2. P. Minuţ, C. Simirad – Numere prime. Numere prime speciale, Editura Matrix
Rom, Bucureşti, 2005.
3. N.N. Vorobiev – Numerele lui Fibonacci, Editura Tehnică, Bucureşti, 1953.
Vizitaţi pagina web a revistei:
http://www.recreatiimatematice.ro
30
Generalizarea unei identităţi şi aplicaţii
Lucian TUŢESCU 1
Abstract. In this Note, the identity (1) is generalized as (2). Two applications are also given.
Keywords: identity, polynomial, divisibility, composed number.
MSC 2000: 13M10.
Identitatea pe care o vom generaliza şi utiliza ı̂n aplicaţii este
a3 + b3 + c3 − 3abc = (a + b + c)(a2 + b2 + c2 − ab − bc − ca), ∀a, b, c ∈ R.
(1)
Pentru a o demonstra considerăm
P (x) = (x − a)(x − b)(x − c) = x3 − (a + b + c)x2 + (ab + bc + ca)x − abc;
avem
P (a) = a3 − (a + b + c)a2 + (ab + bc + ca)a − abc = 0,
P (b) = b3 − (a + b + c)b2 + (ab + bc + ca)b − abc = 0,
P (c) = c3 − (a + b + c)c2 + (ab + bc + ca)c − abc = 0.
Adunând relaţiile de mai sus membru cu membru, obţinem
a3 + b3 + c3 − (a + b + c)(a2 + b2 + c2 ) + (a + b + c)(ab + bc + ca) − 3abc = 0,
de unde deducem identitatea (1).
Vom folosi această identitate binecunocută ı̂n rezolvarea câtorva probleme.
Propoziţie. Arătaţi că are loc următoarea identitate:
x3 + y 3 + z 3 + t3 − 3xyz − 3xyt − 3xzt − 3yzt =
(2)
= (x + y + z + t)(x2 + y 2 + z 2 + t2 − xy − xz − xt − yz − yt − zt), ∀x, y, z, t ∈ R.
Într-adevăr, avem egalităţile:
x3 + y 3 + z 3 − 3xyz = (x + y + z + t − t)(x2 + y 2 + z 2 − xy − yz − zx),
x3 + y 3 + t3 − 3xyt = (x + y + z + t − z)(x2 + y 2 + z 2 − xy − yt − tx),
x3 + z 3 + t3 − 3xzt = (x + y + z + t − y)(x2 + z 2 + t2 − xz − zt − tx),
y 3 + z 3 + t3 − 3yzt = (x + y + z + t − x)(y 2 + z 2 + t2 − yz − zt − ty),
care, adunate membru cu membru, conduc la:
3(x3 + y 3 + z 3 + t3 ) − 3xyz − 3xyt − 3xzt − 3yzt =
= (x + y + z + t)(3x2 + 3y 2 + 3z 2 + 3t2 )−
−(x + y + z + t)(2xy + 2xz + 2xt + 2yz + 2yt + 2zt)−
−t(x2 + y 2 + z 2 + t2 − t2 ) − z(x2 + y 2 + z 2 + t2 − z 2 )−
−y(x2 + y 2 + z 2 + t2 − y 2 ) − x(x2 + y 3 + z 2 + t2 − x2 )+
+3(xyz + xyt + xzt + yzt),
1 Profesor,
Colegiul Naţional ”Fraţii Buzeşti”, Craiova
31
de unde, ı̂n urma unor calcule simple, obţinem (2).
Observaţie. Identitatea (1) se obţine din (2) luând t = 0. Dacă ı̂n (2) luăm
z = t = 0, vom obţine x3 + y 3 = (x + y)(x2 + y 2 − xy).
Aplicaţia 1. Fie x, y, z ∈ N∗ şi nu toate egale. Arătaţi că dacă x + y + z divide
x + y 3 + z 3 , atunci numărul x + y + z este compus.
Presupunând că x+y +z este număr prim şi ţinând seama de ipoteză şi identitatea
(1), rezultă că x+y+z divide produsul 3xyz, de unde deducem că x+y+z|3, x+y+z|x,
x + y + z|y sau x + y + z|z. Cum, ı̂nsă, x + y + z ≥ 1 + 1 + 2 = 4 şi x + y + z > x,
x + y + z > y, x + y + z > z, se ajunge la o contradicţie. Aşadar, x + y + z este număr
compus.
3
Aplicaţia 2. Fie x, y, z ∈ Z astfel ı̂ncât (x − y)2 + (y − z)2 + (z − x)2 = xyz.
Arătaţi că x3 + y 3 + z 3 se divide cu x + y + z + 6.
xyz
Relaţia din enunţul problemei se mai scrie x2 + y 2 + z 2 − xy − yz − zx =
.
2
Combinând aceasta cu identitatea (1), obţinem
x3 + y 3 + z 3 =
xyz
(x + y + z + 6).
2
Datorită acestei relaţii putem afirma că nu toate numerele x, y, z sunt impare (ı̂n caz
contrar, ar rezulta că şi x + y + z + 6 este impar, ceea ce-i imposibil). Dacă măcar
unul dintre x, y şi z este par, aceeaşi relaţie ne arată că x + y + z + 6|x3 + y 3 + z 3 .
1. Cu numerele 1, 3, 4, 6, luate ı̂n ordinea pe care o doriţi şi punând ı̂ntre ele ı̂n
mod convenabil semnele celor patru operaţii aritmetice şi, eventual, paranteze, obţineţi
ca rezultat fiecare dintre numerele: 21, 22, 23, 24 şi 25.
2. În tabelul de mai jos, obţineţi rezultatul 7 punând semne convenabile de operaţii
matematice sau paranteze:
1
2
3
4
5
6
7
8
9
1
2
3
4
5
6
7
8
9
1
2
3
4
5
6
7
8
9
1
2
3
4
5
6
7
8
9
N.B. Puteţi găsi răspunsurile la pag. 36.
32
=
=
=
=
=
=
=
=
=
7
7
7
7
7
7
7
7
7
Problema G128 – comentarii
Marian Tetiva 1
Abstract. This Note offers a discussion on the conditions ensuring the validity of inequality
G128 that was proposed in Recreaţii Matematice No. 2/2007 and ”solved” in No. 2/2008 of the
same journal.
Keywords: Cauchy-Schwarz inequality.
MSC 2000: 97B99.
Dl Dumitru Barac din Sibiu observă, ı̂ntr-o scrisoare trimisă redacţiei, că ı̂n
soluţia Problemei G128 [1] s-a strecurat o greşeală. Şi are perfectă dreptate, sensul
inegalităţii Cauchy-Schwarz folosite acolo ı̂n forma
x2
y2
z2
+ 2
+ 2
≤
+ (t − 1)xy 2y + (t − 1)yz
2z + (t − 1)zx
(x + y + z)2
≤
2
2
2
2(x + y + z ) + (t − 1)(xy + yz + zx)
2x2
trebuie să fie (corect) sensul contrar - ceea ce anulează demonstraţia publicată.
Problema cerea să se arate că are loc inegalitatea
(1)
a2
a
b
c
3
+ 2
+ 2
≤
+t b +t c +t
t+1
pentru orice t ∈ [1, 5] şi orice numere pozitive a, b şi c cu produsul abc = 1. Iată ce
am reuşit să demonstrăm:
√
√
√Inegalitatea
√ √(1) are√ loc pentru orice t ∈ [3 − 2 2, 3 + 2 2] şi orice a, b, c ∈
[( 2 − 1) t, ( 2 + 1) t] pentru care abc = 1.
Demonstraţia utilizează o intercalare (să-i spunem cu medii) uzuală ı̂n asemenea
inegalităţi; anume, arătăm că, ı̂n ipotezele anunţate, au loc inegalităţile
√
a
b
c
2 ab
c
3
+ 2
+ 2
≤
+ 2
≤
2
a +t b +t c +t
ab + t c + t
t+1
(care implică inegalitatea din G128). Prima inegalitate este succesiv echivalentă cu
√
(a + b)(ab + t)
2 ab
a+b
(a2 + t)(b2 + t)
√
≤
⇔
≤
⇔
(a2 + t)(b2 + t)
ab + t
(ab + t)2
2 ab
√
√
√
√ √
t(a − b)2
( a − b)2
√
≤
⇔
⇔ (ab + t)2 ≤ 2 ab( a + b)2 t.
2
(ab + t)
2 ab
√
√
Ultima inegalitate se mai scrie a2 b2 + t2 ≤ 2 ab(a + b + ab)t şi rezultă folosind tot
o intercalare
√
√
a2 b2 + t2 ≤ 6abt ≤ 2 ab(a + b + ab)t.
1 Profesor,
Colegiul Naţional ”Gheorghe Roşca Codreanu”, Bârlad
33
√
√
Aici avem a2 b2 + t2 ≤ 6abt deoarece√ab ∈ [(3 − 2 √
2)t, (3 + 2 2)t] (conform ipotezei),
iar a doua inegalitate rezultă din 3 ab ≤ a + b + ab.
Pentru
√
2 ab
c
3
+
≤
ab + t c2 + t
t+1
√
să notăm x = c (⇔ c = x2 ) şi să ı̂nlocuim ab cu 1/c = 1/x2 . Avem de demonstrat
că
2x
3
x2
f (x) =
≤
+ 4
2
1 + tx
x +t
1+t
√
√ √
√
pentru x2 = c ∈ [( 2 − 1) t, ( 2 + 1) t].
√
√
Să remarcăm că aceste inegalităţi implică x ∈ [(3−2 2)t, (3+2 2)t]. Într-adevăr,
avem
√
√
√
x2 = c ≤ ( 2 + 1) t ≤ ( 2 + 1)4 t2
şi
√
√
√
x2 = c ≥ ( 2 − 1) t ≥ ( 2 − 1)4 t2 ,
√
√
√
folosind şi 2 − 1 ≤ t ≤ 2 + 1. Dar atunci vom avea x2 −√6tx + t2 ≤ 0√şi vom putea
arăta că funcţia f are un maxim ı̂n 1 pe intervalul [(3 − 2 2)t, (3 + 2 2)t]. Pentru
asta calculăm
1 − tx2
tx − x5
′
f (x) = 2
+ 4
,
(1 + tx2 )2
(x + t)2
ceea ce va conduce la următoarea expresie pentru (1 + tx2 )2 (x4 + t)2 f ′ (x)/2:
(1 − tx2 )(x8 + 2tx4 + t2 ) + (tx − x5 )(1 + 2tx2 + t2 x4 ) =
= (1 − x3 )(t + x)[t(x6 + 1) + 4tx3 − x4 − t2 x2 ]
(nu există altă modalitate de a ajunge aici decât aceea ı̂n care calculăm; doar calculăm!). Deoarece paranteza pătrată este nenegativă:
t(x6 + 1) + 4tx3 − x4 − t2 x2 ≥ 2tx3 + 4tx3 − x4 − t2 x2 = x2 (6tx − x2 − t2 ) ≥ 0
√
(conform celor observate puţin √
mai sus) rezultă că f creşte de la (3 − 2 2)t la 1 şi
descreşte pe intervalul [1, (3 + 2 2)t], deci are un maxim
√ ı̂n 1, aşa cum
√ am anunţat.
Inegalitatea f (x) ≤ f (1) (pentru x cuprins ı̂ntre (3 − 2 2)t şi (3 + 2 2)t) este cea
care ne trebuie pentru a ı̂ncheia demonstraţia. √
√
(Merită un studiu separat cazurile t = 3 − 2 2 şi t = 3 + 2 2, când 6t − 1 −
t2 = 0, deci paranteza pătrată de mai sus se anulează şi ea pentru x = 1; se va
vedea că
√ inegalitatea
√ rămı̂ne valabilă, 1 fiind acum una din extremităţile intervalului
[(3 − 2 2t, (3 + 2 2)t].)
Bibliografie
1. T. Zvonaru, B. Ioniţă - Soluţia problemei G128, Recreaţii Matematice, 2/2008, p.
161.
34
O problemă de numărare
Răzvan CEUCĂ1
Abstract. Problem C.O:5077 of Gazeta Matematică, No. 12/2009 has required to count up the
trapezia whose vertices lie among the vertices of a regular polygon with 2010 sides. This problem is
generalized to the case of a regular polygon with n sides.
Keywords: regular polygon, trapezium.
MSC 2000: 05A05, 97B99.
În Gazeta Matematică 12/2009, apare problema
C.O.: 5077. Se consideră poligonul regulat cu 2010 laturi A1 A2 . . . A2010 . Câte
trapeze Ai Aj Ak Al au vârfurile printre cele ale poligonului?
Gabriel Popa şi Paul Georgescu
În nota de faţă, ne propunem să rezolvăm această problemă ı̂n cazul general,
considerând A1 A2 . . . An poligon regulat cu n laturi.
Fie C cercul circumscris poligonului. Orice trapez Ai Aj Ak Al , fiind ı̂nscris ı̂n cercul
C, este isoscel şi deci admite o axă de simetrie. Deosebim situaţiile:
I. n impar. În acest caz, axa de simetrie a unui trapez dintre cele căutate
este diametru ı̂n C, care conţine un vârf al poligonului şi este mediatoare a laturii
care se opune acestui vârf. Fie n = 2m + 1; numărăm ı̂ntâi trapezele care admit
axa de simetrie A1 Mm+1 , unde Mm+1 este mijlocul segmentului Am+1 Am+2 . Dacă
Ai Aj Ak Al este un astfel de trapez, cu 2 ≤ i < j ≤ m + 1, atunci perechea (i, j) poate
m(m − 1)
m(m − 1)
2
=
fi aleasă ı̂n Cm
moduri, prin urmare există
asemenea trapeze.
2
2
Cum axa de simetrie poate fi aleasă ı̂n n moduri, numărul trapezelor este, ı̂n acest
m(m − 1)
n(n − 1)(n − 3)
caz, n ·
=
.
2
8
II. n par. Atunci, un trapez dintre cele considerate admite drept axă de simetrie: 1) fie un diametru Ap Ap+m , p ∈ {1, 2, . . . , m}, unde n = 2m; 2) fie o mediatoare Mp Mp+m , comună perechii de laturi paralele Ap Ap+1 şi Ap+m Ap+m+1 , cu
p ∈ {1, 2, . . . , m}, unde A2m+1 = A1 . Apar ı̂nsă două dificultăţi suplimentare: contează dacă numărul punctelor rămase de-o parte a axei de simetrie este par sau impar,
iar o parte dintre perechile de puncte care se formează nu furnizează trapeze, ci dreptunghiuri. În aceste condiţii, deosebim situaţiile:
II1 . n = 4q. Fie Ai Aj Ak Al un trapez cu axa de simetrie A1 A2q+1 , unde 2 ≤ i <
(2q − 1)(2q − 2)
2
= (2q − 1)(q − 1)
j ≤ 2q. Perechea (i, j) poate fi aleasă ı̂n C2q−1
=
2
moduri. Dintre acestea furnizează dreptunghiuri q − 1 perechi, anume cele de forma
(i, 2q + 2 − i), i ∈ {2, 3, . . . , q}. Găsim astfel (2q − 1)(q − 1) − (q − 1) = 2(q − 1)2
trapeze cu axa de simetrie A1 A2q+1 . Cum axele de simetrie de tipul 1) sunt ı̂n număr
de 2q, avem 4q(q − 1)2 trapeze de acest tip.
1 Elev,
Colegiul Naţional Iaşi
35
Fie acum Ai Aj Ak Al un trapez cu axa de simetrie M1 M2q+1 , unde 2 ≤ i < j ≤ 2q+
2
1. Perechea (i, j) poate fi aleasă ı̂n C2q
= q(2q −1) moduri. Dintre acestea, furnizează
dreptunghiuri q perechi, anume cele de forma (i, 2q +3−i), unde i ∈ {2, 3, . . . , 2q +1}.
Găsim astfel q(2q − 1) − q = 2q(q − 1) trapeze cu axa de simetrie M1 M2q+1 . Cum
axele de simetrie de tipul 2) sunt ı̂n număr de 2q, obţinem 4q 2 (q − 1) trapeze de acest
tip.
Numărul total al trapezelor este, ı̂n acest caz, 4q(q − 1)2 + 4q 2 (q − 1) = 4q(q − 1)·
n(n − 2)(n − 4)
·(2q − 1) =
.
8
II2 . n = 4q + 2. Cu o analiză asemănătoare, găsim 2q(q − 1)(2q + 1) trapeze cu
axa de simetrie de tipul 1) şi ı̂ncă 2q 2 (2q +1) trazepe cu axa de simetrie de tipul 2). În
n(n − 2)(n − 4)
total, vom avea 2q(q − 1)(2q + 1) + 2q 2 (2q + 1) = 2q(2q + 1)(2q − 1) =
8
trazepe, deci acelaşi rezultat ca ı̂n subcazul II1 .
În concluzie, numărul trapezelor Ai Aj Ak Al cu vârfurile printre cele ale poligonului
n(n − 1)(n − 3)
n(n − 2)(n − 4)
este
, dacă n impar, respectiv
, dacă n par.
8
8
1. Răspuns la ”recreaţia” 1 de la pag. 32:
6·4:1−3
6 · 4 − (3 − 1)
4 · (6 − 1) + 3
6 : (1 − 3 : 4)
3 · (1 + 6) + 4
=
=
=
=
=
21
22
23
24
25.
2. Răspuns la ”recreaţia” 2 de la pag. 32:
1 + (1 + 1 + 1)!
(2 · 2)!! − 2 : 2
3+3+3:√
3
4+4:4+ 4
5 + (5 +
√5) : 5
6:6+ 6·6
7√+ (7 − 7) · 7
8 · 8 −√8 : 8√
9:9+ 9+ 9
=
=
=
=
=
=
=
=
=
7
7
7
7
7
7
7
7
7
(se convine că n! = 1 · 2 · 3 · . . . · n şi (2n)!! = 2 · 4 · 6 · . . . · 2n).
36
Un sous-ensemble particulier de matrices carrées
Adrien REISNER1
Abstract. Let a, b be two strictly positive integer numbers and let A denote the set of matrices
of the form (1), with x, y, z, t ∈ Q. It is shown that A is a Q-vector space of dimension 4 and it is
also a subring of the ring M2 (R, +, ×). Theorem 4 and Corollary 5 state conditions for a nonzero
element in A to be invertible.
Keywords: Q-vector space, subring, invertible element.
MSC 2000: 11C08, 15A03.
a et b étant deux entiers strictement positifs, on considère l′ ensemble A des matrices carrées A(x, y, z, t) d′ ordre 2 de la forme suivante:
√
√ √
x + y √a
z b + t ab
√
√
(1)
A(x, y, z, t) =
z b − t ab
x−y a
où x, y, z et t appartiennent au corps des rationnels Q. On se propose d′ établir
quelques propriétés liées à la structure algébrique de cet ensemble A (voir [1], ch. XI,
page 453 et [2]).
Théorème 1. Muni de l ′ addition des matrices et de la multiplication des matrices
par un scalaire rationnel l ′ ensemble A est un Q - espace vectoriel de dimension 4.
Démonstration. L′ ensemble A est un sous - ensemble de M2 (R). L′ addition
des matrices est une loi de composition interne de A. La matrice nulle appartient à
A. Enfin pour toute matrice A ∈ A, la matrice −A ∈ A. Par suite (A, +) est un
groupe abélien.
√
√ √
λx
λz b + λt ab
√ + λy √a
√
∀λ ∈ Q, ∀A ∈ A :
∈ A.
λx − λy a
λz b − λt ab
L′ ensemble A pourvu de l′ addition et de la multiplication par les éléments de Q est
donc un espace vectoriel sur Q. Etant donné les quatre matrices suivantes:
√ √ 
‹
√
‹
1 0
a
0
0
b
0
ab
√
√ ,K = √
I=
,J =
,L =
0 1
0 − a
b 0
− ab
0
on a pour toute matrice A ∈ A : A(x, y, z, t) = xI + yJ + zK + tL. Donc la famille
{I, J, K, L} est génératrice. Cette
libre puisque
√
√
√
√
√ famille est √
xI +yJ +zK +tL = 0 ⇒ x+y a = 0, x−y a = 0, z b+t ab = 0, z b−t ab = 0
⇒ x = y = z = t = 0.
Finalement, les quatre matrices I, J, K et L constituent une base du Q - espace
vectoriel A qui est donc de dimension 4.
1 Centre
de Calcul E.N.S.T., Paris; e-mail: [email protected]
37
Remarque. detA(x, y, z, t) = x2 − ay 2 − bz 2 + abt2 .
Théorème 2. (A, +, ×) est un sous - anneau de l ′ anneau (M2 (R), +, ×).
Démonstration. La table de multiplication des matrices I, J, K et L est la
suivante:
facteur de droite −→
I
J
K
L
I I
J
K
L
J J
aI
L
aK
K K
-L
bI -bJ
L L -aK bJ -abI
A ⊂ M2 (R) qui est un anneau unitaire. (A, +) est un sous - groupe de (M2 (R), +).
Enfin, la multiplication est une loi de composition interne pour A compte tenu de
la table de multiplication précédente. On en déduit finalement que (A, +, ×) est un
sous-anneau unitaire de l′ anneau (M2 (R), +, ×).
Les deux théorèmes précédents montrent que l′ ensemble A muni des opérations
habituelles est une Q - algèbre de dimension 4 - [1], VI.4 page 215.
Théorème 3. Pour toute matrice A ∈ A l ′ application fA de A dans A définie
par ∀X ∈ A, fA (X) = XA est linéaire. De plus, det fA = (detA)2 .
Démonstration. Pour A ∈ A, fA est une application de A dans A puisque X ∈ A
et A ∈ A ⇒ XA ∈ A. D′ autre part: ∀X1 ∈ A, X2 ∈ A fA (X1 + X2 ) = (X1 + X2 )A =
X1 A + X2 A = fA (X1 ) + fA (X2 ); ∀X ∈ A, ∀λ ∈ Q, fA (λX) = (λX)A = λ(XA) =
λfA (X), i.e. fA ∈ LQ (A). L′ espace vectoriel A étant muni de la base (I, J, K, L) voir théorème 1 - soit A = x0 I + y0 J + z0 K + t0 L. Déterminons en utilisant la table
de multiplication des matrices I, J, K et L la matrice de l′ endomorphisme fA dans
cette base (I, J, K, L):
fA (I) = x0 I + y0 J + z0 K + t0 L = A
fA (J) = ay0 I + x0 J + at0 K + z0 L
fA (K) = bz0 I − bt0 J + x0 K − y0 L
fA (L) = −abt0 I + bz0 J − ay0 K + x0 L,
d′ où la matrice de fA ∈ LQ (A) dans la base considérée:
†
M at(fA , (I, J, K, L)) =
x0
y0
z0
t0
ay0
x0
at0
z0
bz0
−bt0
x0
−y0
−abt0
bz0
−ay0
x0

et un calcul élémentaire montre alors que det(M at(fA , (I, J, K, L))=[detA(x0 , y0 , z0 , t0 )]2
compte tenu de la remarque précédente.
Théorème 4. Dans le sous-anneau A tout élément non nul est inversible si et
seulement si l ′ égalité u2 = av 2 + bw2 avec u, v et w entiers n′ est possible que pour
u = v = w = 0.
Démonstration. Supposons que
(2)
{u2 = av 2 + bw2 , (u, v, w) ∈ Z3 } ⇒ u = v = w = 0.
38
L′ hypothèse (2) étant vérifiée, montrons d′ abord le lemme suivant:
Lemme 5. On a l ′ implication:
{u′2 = av ′ + bw′ , (u′ , v ′ , w′ ) ∈ Q3 } ⇒ u′ = v ′ = w′ = 0.
2
2
Démonstration. α étant le P.P.C.M. des dénominateurs de u′ , v ′ et w′ il vient:
u′ = av ′ + bw′ ⇒ (u′ α)2 = a(v ′ α)2 + b(w′ α)2 , u′ α ∈ Z, v ′ α ∈ Z, w′ α ∈ Z.
2
2
2
L′ hypothèse (2) montre alors que: u′ α = v ′ α = w′ α = 0, d′ où u′ = v ′ = w′ = 0.
Démontrons alors que toute matrice de A−{0} est inversible. Supposant la matrice
A = xI + yJ + zK + tL ∈ A non inversible, montrons que A = 0. Il vient - voir
remarque -: detA = 0 soit x2 − ay 2 = b(z 2 − at2 ), ce qui implique:
(z 2 − at2 )(x2 − ay 2 ) = b(z 2 − at2 )2 ⇒ z 2 x2 + a2 t2 y 2 − a(t2 x2 + z 2 y 2 ) = b(z 2 − at2 )2
⇒ z 2 x2 + 2axyzt + a2 t2 y 2 − a(t2 x2 + 2xyzt + z 2 y 2 ) = b(z 2 − at2 )2
⇒ (zx + aty)2 − a(tx + zy)2 = b(z 2 − at2 )2
soit finalement:
(zx + aty)2 = a(tx + zy)2 + b(z 2 − at2 )2 ,
où
(zx + aty, tx + zy, z 2 − at2 ) ∈ Q3 .
L′ hypothèse (2) étant vérifiée il vient alors en particulier, compte tenu du lemme
précédent: z 2 − at2 = 0. En utilisant une nouvelle fois le lemme avec u′ = z, v ′ = t et
w′ = 0 on obtient: z = t = 0. Par suite x2 −ay 2 = 0 et, d′ après le même raisonnement:
x = y = 0, i.e. A = 0.
On suppose que tout élément A ∈ A − {0} est inversible. Soit (u, v, w) ∈ Z3 tels
que u2 = av 2 + bw2 et considérons la matrice A = uI + vJ + wK ∈ A. Si A ̸= 0, elle
est inversible - hypothèse -, donc son déterminant n′ est pas nul: u2 − av 2 − bw2 ̸= 0,
ce qui est absurde. On en déduit que A = 0, d′ où u = v = w = 0. Le théorème est
ainsi démontré.
Corollaire 6. Si b est un nombre premier et si pour tout n entier n2 - a n ′ est
pas divisible par b, alors tout élément non nul du sous - anneau A est inversible dans
A.
Démonstration. On se propose de démontrer l′ implication suivante:
(3)
{b premier et ∀n, n2 − a non divisible par b} ⇒
⇒ {u2 = av 2 + bw2 où (u, v, w) ∈ Z3 ⇒ u = v = w = 0}
ce qui démontrera le corollaire compte tenu du théorème précédent.
b étant premier et ∀n, n2 - a n′ étant pas divisible par b, soit (u, v, w) ∈ Z3 vérifiant
u2 = av 2 + bw2 . On peut supposer u, v, w premiers entre eux dans leur ensemble, car
2
2
2
sinon, δ étant leur P.G.C.D. on aurait: u = δu′ , v = δv ′ , w = δw′ et u′ = av ′ + bw′ .
39
Si b divise v alors b divise u2 ; comme b est premier, b divise alors u et b2 divise
u − av 2 . Donc b2 divise bw2 ; comme b est premier, b divise w : u, v, w ne seraient
pas premiers entre eux dans leur ensemble, ce qui est impossible.
Si b ne divise pas v, on a:
2
u2 = av 2 + bw2 ⇒ ∀λ ∈ Z, u2 + 2λbu + λ2 b2 − av 2 = b(w2 + 2λu + λ2 b) ⇒
(u + λb)2 − av 2 = b(w2 + 2λu + λ2 b).
On peut toujours choisir l′ entier λ de façon que v divise u + λb. En effet, b et v
sont premiers entre eux, donc ∃(α, β) ∈ Z2 tels que bα + vβ = 1, ce qui entraı̂ne
bαu + vβu = u; donc v divise u − b(αu). Il suffit de prendre λ = −αu pour que
u + λb = nv, d′ où n2 v 2 − av 2 = (n2 − a)v 2 = b(w2 + 2λu + λ2 b). On en déduit alors: {b
divise (n2 − a)v 2 et b ne divise pas v 2 } ⇒ {b divise n2 − a} ce qui est impossible par
hypothèse. L′ implication (3) en résulte, ce qui achève la démonstration du corollaire.
Exemples. Les couples (a, b) suivants vérifient les hypothèses du corollaire:
(a, b) = (5k + 2, 5), (a, b) = (5k + 3, 5)
(a, b) = (7k + 3, 7), (a, b) = (7k + 5, 7), (a, b) = (7k + 6, 7), où k ∈ N.
En effet:
− si b = 5, alors pour tout n ∈ N, n2 ≡ 0 ou 1 ou 4(mod5), donc a ≡ 2 ou
a ≡ 3(mod5).
− si b = 7, alors pour tout n ∈ N, n2 ≡ 0 ou 1 ou 2 ou 4(mod7) et par suite a ≡ 3
ou a ≡ 5 ou a ≡ 6(mod7).
Remarque. Soit E le sous - ensemble de A formé par les matrices M (x, y, z, t),
où x, y, z et t appartiennent à Z. N étant un entier relatif non nul on dsigne par EN
l′ ensemble
EN = {M (x, y, z, t) ∈ E/detM (x, y, z, t) = N }
On démontre alors le théorème suivant (la démonstration de ce théorème dépasse
le niveau de cet article):
Théorème 7. a) E1 est un groupe multiplicatif.
b) Il existe un sous - ensemble fini IN de l′ ensemble EN vérifiant: ∀M ∈ EN ,
∃(G, E) ∈ E1 × IN tels que: M = G × E - décomposition canonique de la matrice
M ∈ EN .
Références
1. J. M. Arnaudiès, H. Fraysse – Cours de mathématiques, Tome 1, Algèbre, Ed.
Dunod, Paris, 1987.
2. J. Fresnel – Algèbre des matrices, Ed. Hermann, Paris, 1997.
3. C. Năstăsescu, C. Niţă, C. Vraciu – Bazele algebrei, Vol. 1 , Ed. Academiei,
Bucureşti, 1986.
40
O problemă complexă
Marian TETIVA1
Abstract. This paper can be perceived as an author′ s invitation to his own working laboratory.
A problem, initially formulated for equation (1), is eventually solved for the more general equation
(2). The stages of going along this way are presented and motivated and the employed procedures
are commented.
Keywords: equations, resolvent, root.
MSC 2000: 00A35, 97C20.
1. Pe când eram elev de liceu, una din problemele care mi-au dat bătaie de
cap, rezistând tentativelor mele disperate (dacă exagerăm puţin putem zice şi aşa) de
rezolvare a fost următoarea:
Problema 1. Fie α un număr real de modul mai mic ca 1. Să se arate că ecuaţia
(1)
z 4 + αz 3 + αz + 1 = 0
are toate rădăcinile de modul 1.
Enunţul se află cu siguranţă ı̂n [1], ı̂n capitolul ”Polinoame”, probabil ca problemă
propusă la capătul paragrafului despre ecuaţii reciproce (nu mai pot spune exact,
deoarece nu mai am, sau nu mai găsesc, acest manual de pe care am ı̂nvăţat primele
noţiuni de algebră avansată). Mai târziu, peste câţiva ani, am rezolvat problema, ca
tânăr profesor care nu se putea prezenta ı̂n faţa elevilor fără a cunoaşte foarte bine
(perfect ar trebui) cartea pe care o foloseşte la clasă. Am făcut ceea ce părea atunci
să decurgă obligatoriu din formularea problemei şi poziţionarea ei ı̂n cadrul textului:
am considerat ecuaţia ca pe una reciprocă; deci rădăcinile ei sunt rădăcinile ecuaţiilor
z+
1
1
= y1 şi z + = y2 ,
z
z
unde y1 şi y2 sunt, la rândul lor, soluţiile rezolventei
y 2 + αy − 2 = 0
1
(care se obţine notând z + = y etc). Se vede apoi că ecuaţia care dă pe z1 şi z2 ,
z
adică
1
z + = y1 ,
z
se mai scrie z 2 − y1 z + 1 = 0, deci z1 z2 = 1 şi, analog, z3 z4 = 1 (unde z3 şi z4
sunt soluţiile ecuaţiei z 2 − y2 z + 1 = 0). Dacă mai arătăm că aceste ecuaţii au
1 Profesor,
Colegiul Naţional ”Gheorghe Roşca Codreanu”, Bârlad
41
coeficienţi reali şi rădăcini complexe nereale, rezultă că z1 şi z2 sunt conjugate, deci,
având produsul 1, au modulele egale cu 1; similar, z3 şi z4 au aceeaşi proprietate şi
Problema 1 este rezolvată.
Exerciţiul 1. Completaţi detaliile acestei rezolvări!
Nu vă va fi greu să demonstraţi că y1 şi y2 sunt numere reale. Ca să dovediţi că
∆1 = y12 − 4 < 0 şi ∆2 = y22 − 4 < 0 arătaţi că ∆1 + ∆2 < 0 şi ∆1 ∆2 > 0. Folosiţi
relaţiile ı̂ntre rădăcini şi coeficienţi!
Exerciţiul 2. Deduceţi următorul rezultat, o idee mai general decât Problema 1:
Problema 1′ . Dacă α este un număr real oarecare, atunci ecuaţia
z 4 + αz 3 + αz + 1 = 0
are ori două rădăcini de modul 1, ori toate rădăcinile de modul 1.
2. Anii au trecut şi am găsit, ı̂n [2] la pagina 137, problema 147 (şi soluţia ei la
pagina 290). Enunţul este următorul (uşor modificat de noi, dar numai formal):
Problema 2. Fie α un număr real cu |α| ≤ 1 şi n un număr natural, n ≥ 3.
Ecuaţia
z n + αz n−1 + αz + 1 = 0
are toate rădăcinile de modul 1.
E vorba, evident, de cazul general al Problemei 1; metoda utilizată mai sus nu
prea dă speranţe de a obţine această generalizare, dar, cum spuneam, există soluţia
ı̂n [2]. Tot o soluţie algebrică, dar utilizând subtil (şi elegant) proprietăţile modulului
şi conjugatului unui număr complex.
Examinând cu atenţie această soluţie constatăm că ea se potriveşte la fel de bine
următorului enunţ:
Problema 2′ . Fie α un număr complex cu |α| ≤ 1 şi n un număr natural, n ≥ 3.
Ecuaţia
(2)
z n + αz n−1 + αz + 1 = 0
are toate rădăcinile de modul 1.
Soluţie. Fie z o rădăcină a ecuaţiei, pentru care avem z n−1 (z + α) = −αz − 1,
deci şi
|z|2(n−1) |z + α|2 = |αz + 1|2 .
Un calcul simplu (care utilizează |w|2 = ww şi proprietăţile conjugatului unui număr
complex) ne arată că putem să ı̂nlocuim |αz + 1|2 cu |z + α|2 − (|z|2 − 1)(1 − |α|2 ),
astfel că egalitatea de mai sus se mai scrie
(|z|2(n−1) − 1)|z + α|2 + (|z|2 − 1)(1 − |α|2 ) = 0.
Evident, dacă |α| < 1, de aici rezultă |z| = 1. Dacă |α| = 1, ne rămâne egalitatea
(|z|2(n−1) − 1)|z + α|2 = 0,
42
din care obţinem fie (iar) |z| = 1, fie z = −α, număr care are tot modulul 1. Ceea ce
ı̂ncheie demonstraţia.
3. Au mai trecut ani şi m-am mai gândit că există şi posibilitatea abordării acestei
probleme cu ajutorul formei trigonometrice a numerelor complexe. După părerea mea,
această abordare conduce la soluţia cea mai interesantă a Problemei 2′ , adică a cazului
celui mai general (din câte vedem ı̂n această notă) al problemei iniţiale. Vom vedea
de ce.
Ideea este să arătăm că funcţia F definită prin
F (z) = z n + αz n−1 + αz + 1
se anulează de n ori pe cercul unitate (adică pe mulţimea numerelor complexe de
modul 1). Avantajul este că lucrăm direct cu numere complexe de modul 1, care pot
fi exprimate ı̂n forma cos t + i sin t, pentru un anumit t real (de fapt din intervalul
[0, 2π)). Pentru prima parte a acestei a II-a soluţii a Problemei 2′ e nevoie de nişte
calcule trigonometrice, pe care vă invit a le face.
Exerciţiul 3. Arătaţi că, dacă notăm α = a + ib, atunci
‹

F (cos t + i sin t) = 2 cos

nt
nt
·
+ i sin
2
2
‹
· cos
nt
(n − 2)t
(n − 2)t
.
+ a cos
− b sin
2
2
2
Evident, numai a doua paranteză se poate anula şi vom arăta că asta se ı̂ntâmplă
pentru n valori (distincte) ale lui t din intervalul [0, 2π]. Considerăm deci funcţia,
desigur continuă, φ : [0, 2π] → R, dată de
φ(t) = cos
nt
(n − 2)t
(n − 2)t
+ a cos
− b sin
, ∀t ∈ [0, 2π].
2
2
2
Avem
√
1
φ(t) =
+ b2
a2
1
nt
a
(n − 2)t
b
(n − 2)t
cos
+√
cos
−√
sin
=
2
2
2
a2 + b2
a2 + b2
a2 + b2

‹
1
nt
nt
=√
cos
+ cos
−t+θ
2
2
a2 + b2
unde θ este un număr real astfel ı̂ncât
=√
cos θ = √
b
a
şi sin θ = √
.
2
2
+b
a + b2
a2
Observăm că avem, pentru t = (2kπ)/n (k ı̂ntreg), nt/2 = kπ, deci
√

1
2kπ
φ
n
a2 + b2
‹

= (−1)k
√
43

1
2kπ
+ cos θ −
n
a2 + b2
‹‹
.
√
√
În cazul |α| = a2 + b2 < 1 avem 1/ a2 + b2 > 1, deci a doua paranteză este strict
pozitivă, ceea ce ı̂nseamnă că semnele valorilor φ(2kπ/n) pentru k ı̂ntreg alternează.
Dacă alegem pentru k valorile 0, 1, . . . , n rezultă că funcţia continuă φ se anulează de
cel puţin n ori ı̂n intervalul [0, 2π]: câte o dată ı̂n fiecare interval

‹
2kπ 2(k + 1)π
,
, k = 0, 1, . . . , n − 1.
n
n
Asta ı̂nseamnă că F are cel puţin n rădăcini distincte pe cercul unitate; dar, fiind o
funcţie polinomială de grad n, rezultă că are exact n asemenea rădăcini, sau că toate
rădăcinile sale au modulele egale cu 1.
Dacă |α| = 1, paranteza de mai sus devine

2kπ
1 + cos θ −
n
‹
2kπ
şi este, ı̂n general, nenegativă. Deoarece numerele θ −
pentru k = 0, 1, . . . , n se
n
găsesc ı̂ntr-un interval de lungime 2π, pentru cel mult o valoare a lui k această expresie
este 0. Dacă asta se ı̂ntâmplă, respectiva valoare a lui k furnizează o rădăcină a lui F
şi, din cauza ei, se pierd cel mult două schimbări de semn ı̂n şirul valorilor φ(2kπ/n),
deci oricum rămân n − 1 rădăcini de modul 1 pentru F de care putem fi siguri. Cum
produsul rădăcinilor lui F este 1 sau −1, cea de-a n-a rădăcină rezultă tot de modul
1 şi astfel se ı̂ncheie şi această soluţie.
Despre care oricine va spune, probabil, că e mai complicată decı̂t cea tradiţională
(nici n-am scris toate calculele!). Totuşi, eu o consider mai instructivă şi (aici e partea
interesantă) mai productivă. Spun asta deoarece (cititorul atent probabil s-a ı̂ntrebat
deja) apare ı̂n mod natural o chestiune adiacentă: dar dacă |α| > 1? Ce mai putem
spune despre modulele rădăcinilor ecuaţiei z n + αz n−1 + αz + 1 = 0 ı̂n cazul ı̂n care
modulul lui α este mai mare ca 1? Deocamdată ne oprim aici, lăsându-vă cadou
următoarea ı̂ntrebare:
Exerciţiul 4. Dacă α este un număr complex, cu |α| > 1, se poate garanta existenţa
unor rădăcini de modul 1 ale ecuaţiei
z n + αz n−1 + αz + 1 = 0?
Eu aş zice că da.
Bibliografie
1. C. Niţă, C. Năstăsescu, S. Popa – Algebră. Manual pentru clasa a X-a, Editura
Didactică şi Pedagogică, Bucureşti, 1980.
2. L. Panaitopol, I. C. Drăghicescu – Polinoame şi ecuaţii algebrice, Editura Albatros, Bucureşti, 1980.
44
Axe şi centre radicale
ale cercurilor adjuncte unui triunghi
Ion PĂTRAŞCU1
Abstract. The properties of the adjoint circles associated to a triangle, with respect to circles
radical axes and centres, are exhaustively presented in this Note.
Keywords: adjoint circle, Brocard point, radical axis, radical centre, symmedian.
MSC 2000: 51M04.
Proprietăţile prezentate ı̂n acest articol se referă la axele şi centrele radicale ale
cercurilor adjuncte unui triunghi.
Fie ABC un triunghi oarecare. Notăm cu C Ā cercul care trece prin vârfurile C, A
şi este tangent laturii AB ı̂n vârful A. Semnificaţii analoage au notaţiile AC, AB,
BA, BC şi CB. Aceste cercuri asociate unui triunghi se numesc cercuri adjuncte.
Aşadar, unui triunghi ı̂i corespund ı̂n general şase cercuri adjuncte; dacă triunghiul
este isoscel, vor fi cinci cercuri adjuncte, iar dacă este echilateral, vor fi numai trei.
Teorema 1. i) Cercurile adjuncte CA, AB, BC au un punct comun Ω cu propriÕ ≡ ΩBC
Õ ≡ ΩCA.
Õ
etatea: ΩAB
′ AC ≡
Õ
ii) Cercurile adjuncte AC, CB, BA au un punct comun Ω′ cu proprietatea: Ω
′ BA ≡ Ω
′ CB.
Ö
Ö
Ω
Demonstraţie. i) Fie Ω al doilea punct de intersecţie a cercurilor CA şi AB
Õ ≡ ΩAB
Õ (ı̂n cercul CA) şi ΩAB
Õ ≡
(fig. 1). Avem: ΩCA
A
Õ
Õ
Õ
Õ
ΩBC (ı̂n AB). Obţinem: ΩAB ≡ ΩBC ≡ ΩCA, adică
w
Õ ≡ ΩCA
Õ rezultă că Ω se găseşte
relaţia cerută, iar din ΩBC
pe cercul ce trece prin B şi C şi este tangent laturii AC ı̂n
W
w
C, adică cercul BC. Afirmaţia ii) se demonstrează analog.
Punctul Ω se numeşte primul punct al lui Brocard, iar Ω′
al doilea punct al lui Brocard.
B
w
Fig. 1
C
Observaţie. Punctul Ω este centrul radical al cercurilor adjuncte CA, AB, BC,
iar Ω′ este centrul radical al cercurilor adjuncte AC, CB, CA. Într-adevăr, atât Ω cât
şi Ω′ au puteri egale (nule) faţă de tripletele de cercuri adjuncte indicate.
Teorema 2. Punctele lui Brocard Ω şi Ω′ sunt izogonale ı̂n triunghiul ABC.
Õ = ω. Aplicând teorema sinusurilor ı̂n triunDemonstraţie. Notăm m(ΩAB)
ghiurile AΩB şi AΩC obţinem:
c
AΩ
AΩ
b
AΩ
BΩ
=
=
şi
=
=
.
sin ω
sin(BΩA)
sin(B − ω)
sin ω
sin(AΩC)
sin ω
1 Profesor,
Colegiul Naţional ”Fraţii Buzeşti”, Craiova
45
Õ = 180◦ − B şi m(AΩC)
Õ = 180◦ − A, rezultă că
Deoarece m(BΩA)
AΩ
b sin B
sin(B − ω)
=
=
.
BΩ
c sin A
sin ω
Dezvoltând sin(B − ω) şi ţinând cont că
(1)
b
sin B
=
şi sin(A + C) = sin B, se obţine:
c
sin C
ctgω = ctgA + ctgB + ctgC.
′ AC) = ω ′ şi vom raţiona analog, vom obţine ctgω ′ = ctgA +
Õ
Dacă vom nota m(Ω
ctgB + ctgC. Aceasta şi relaţia (1) conduc la ω = ω ′ , ceea ce arată că punctele Ω şi
Ω′ sunt izogonale.
Unghiul ω se numeşte unghiul lui Brocard şi apare ı̂n multe ı̂mprejurări ı̂n geometria triunghiului (v. [1]).
Teorema 3. Cercurile adjuncte CA şi BA se intersectează pe simediana din A.
Demonstraţie. Fie S al doilea punct de intersecţie al cercurilor CA şi BA
Õ ≡ SAB
Õ şi
(fig. 2) şi {P } = AS ∩ BC. Din faptul că SCA
A
Õ ≡ SAC,
Õ deducem că △SBA ∼ △SAC, de unde SB =
SBA
SC
AB 2
S
. Pe de altă parte, din congruenţele de mai sus obţinem
AC 2
Õ ≡ CSP
Õ , iar cu teorema bisectoarei ı̂n triunghiul BSC
BSP
C
B
SB
PB
PB
AB 2
Fig.
2
vom avea
=
. Ca urmare,
=
, ceea ce
SC
PC
PC
AC 2
arată că AP este simediana din A.
Teorema 4. Cercurile adjuncte AB şi AC se intersectează pe mediana din A.
Demonstraţie. Fie Q al doilea punct de intersecţie a cercurilor AB şi AC şi
{M } = AQ ∩ BC. Dreapta AQ fiind axa radicală a cercurilor AB şi AC, avem
M B 2 = M Q · M A = M C 2 , de unde rezultă că M B = M C.
Observaţii. a) Teorema 3 exprimă faptul că axa radicală a două cercuri adjuncte
care sunt tangente la două laturi ce au un vârf comun este simediana dusă din acel
vârf, iar Teorema 4 spune că axa radicală a două cercuri adjuncte tangente la aceeaşi
latură este mediana corespunzătoare acestei laturi.
b) Drept axă radicală a două cercuri adjuncte unui triunghi pot fi: cevienele Brocard (AΩ, AΩ′ etc.), simedianele, medianele sau laturile triunghiului (latura BC, de
exemplu, este axa radicală a cercurilor BC şi CB).
Pentru noi precizări privind centrele radicale ale tripletelor de cercuri adjuncte,
vom utiliza următoarea
Lemă. Coardele comune a trei cercuri secante două câte două sunt concurente.
Demonstraţie. Fie C1 , C2 , C3 trei cercuri secante şi a1 , a2 , a3 axele radicale ale
perechilor (C2 , C3 ), (C3 , C1 ) şi respectiv (C1 , C2 ). Notăm P intersecţia dintre a1 şi a2
şi observăm că P va avea puteri egale faţă de toate cercurile, deci P va fi şi pe axa
radicală a3 a cercurilor C1 şi C2 .
Teorema 5. Au loc afirmaţiile:
46
i) Ceviana AΩ, simediana din B şi mediana din C sunt concurente (ı̂ntr-un punct
notat JA );
ii) Ceviana BΩ, simediana din C şi mediana din A sunt concurente (ı̂n JB );
iii) Ceviana CΩ, simediana din A şi mediana din B sunt concurente (ı̂n JC ).
Demonstraţie. Conform Teoremei 1, cercurile CA şi AB se intersectează ı̂n
Ω. Din Teorema 3, al doilea punct comun cercurilor
A
AB şi CB, notat S (fig. 3), este situat pe simediana
din B. În sfârşit, aplicând Teorema 4, cercurile CA
şi CB se intersectează a doua oară ı̂ntr-un punct M ,
W
care aparţine medianei din C. Conform Lemei, ceJ
M
S
vienele AΩ, BS şi CM sunt concurente, ceea ce era
de demonstrat.
B
C
În acelaşi mod se poate dovedi şi
Fig. 3
Teorema 6. Au loc afirmaţiile:
i) Ceviana AΩ′ , simediana din C şi mediana din B sunt concurente (ı̂ntr-un punct
′
JA );
′
);
ii) Ceviana BΩ′ , simediana din A şi mediana din C sunt concurente (ı̂n JB
′
iii) Ceviana CΩ , simediana din B şi mediana din A sunt concurente (ı̂n JC′ ).
A
Observaţii. 1. Teoremele 5 şi 6 se pot demonstra şi cu reciproca teoremei lui
Ceva. Într-adevăr, ştim că piciorul simedianei din A ı̂mparte latura BC ı̂n raportul
c2
. Pe de altă parte, notând cu A1 şi A′1 picioarele cevienelor Brocard AΩ şi AΩ′ ,
b2
avem
a2
BA′1
= 2
′
A1 C
b
BA1
c2
= 2,
A1 C
a
(2)
BA1
A△ABA1
c sin ω
c2
=
=
= . . . (1) . . . = 2 , iar
A1 C
A△AA1 C
b sin(A − ω)
a
pentru a doua se procedează similar sau se aplică Teorema lui Steiner cevienelor
izogonale AΩ şi AΩ′ ).
2. În general, un triunghi are şase cercuri adjuncte, deci pot fi C63 triplete diferite
de cercuri adjuncte. Unele triplete au, ı̂nsă, acelaşi centru radical. Vârfurile triunghiului sunt centre radicale, fiecare pentru patru triplete; de exemplu, C este centru
radical pentru (BC, CB, AC), (BC, CB, CA), (AC, CA, BC) şi (AC, CA, CB).
Ca urmare, drept centru radical a trei cercuri adjuncte unui triunghi pot fi: punctele
′
′
lui Brocard Ω şi Ω′ , punctele JA , JB , JC , JA
, JB
, JC′ şi vârfurile A, B, C.
3. Dacă triunghiul este isoscel, AB = AC, atunci BC coincide cu CB şi simediana
din B şi mediana din C se intersectează ı̂n punctul Ω al lui Brocard [3].
(pentru prima egalitate:
Bibliografie
1. T. Lalescu – Geometria triunghiului, Editura Apollo, Craiova, 1993.
2. R.A. Johnson – Modern Geometry: An Elementary Treatise on the Geometry of the
Triangle and the Circle, 1929.
3. I. Pătraşcu – O teoremă relativă la punctul lui Brocard, G.M.-9/1984, 328-329.
47
Şcoala Normală ”Vasile Lupu” din Iaşi
– o istorie zbuciumată –
A fost creată pentru a pregăti serii de ı̂nvăţători menite să răspândească lumina
cărţii ı̂n satele şi oraşele ţării şi să contribuie la ridicarea lor social-economică. Lunga
ei istorie, de mai bine de 150 de ani, este presărată de mari ı̂mpliniri, dar şi de
momente dramatice.
În Aşezământul pentru organizarea ı̂nvăţăturilor publice din principatul Moldova din 1851, care proclama libertatea şi gratuitatea ı̂nvăţământului,
se prevedea, ı̂ntre altele, faptul că statul avea obligaţia de a ı̂nfiinţa şi ı̂ntreţine şcoli
pentru populaţia de la oraşe şi sate. Prin hrisovul domnitorului Grigore Alexandru Ghica din 15 decembrie 1855 lua fiinţă Institutul Preparandal, prima şcoală
din cele două ţări româneşti având ca scop pregătirea ı̂nvăţătorilor ce trebuiau să
răspândească lumina ı̂n lumea satelor.
Acest institut, cu durata studiilor de un an, a funcţionat timp de 35 de ani ı̂n
ı̂ncăperile mănăstirii Trei-Ierarhi din Iaşi, unde mai exista o şcoală, Şcoala Primară
Vasiliană, şi unde s-a ı̂nfiinţat şi internatul preparanzilor. Director a fost numit
Anton Velini, care a condus ı̂ntregul complex şcolar de la Trei-Ierarhi până ı̂n anul
1863. Câteva dintre meritele lui: a ı̂mbunătăţit programele şcolii, adăugând şi discipline noi, a tipărit un Manual de metodică şi pedagogie pentru profesorii şcoalelor
primare utilizat de preparanzi şi ı̂nvăţători pentru pregătirea lor pedagogică, a preconizat realizarea şcolii primare de aplicaţie (”preparanzii să fie şi pedagogi ...”) ş.a.
48
Al doilea director al Şcolii Preparandale a fost Titu Maiorescu, ı̂n perioada
1864-1868. Renunţând la poziţii mai bine situate şi remunerate, primeşte direcţia
acestei şcoli cu convingerea că ”aici este un câmp de activitate modestă, patientă,
ı̂n aparenţă inferioară, ı̂n realitate de importanţă suverană pentru poporul nostru”.
Îndrumătorul culturii româneşti consideră că ”regenerarea poporului român ı̂ncepe de
la cultivarea limbii române” şi ı̂şi ı̂ndreaptă atenţia spre şcoala primară.
Considera că trebuie asigurată viitorului ı̂nvăţător o cultură generală şi o pregătire
profesională de calitate şi o ı̂naltă conştiinţă a misiunii sale de educator. Foştii săi
elevi Ion Creangă şi Mihai Busuioc au ilustrat strălucit acest profil de ı̂nvăţător.
A publicat anuarul şcolii pe anul 1863-1864, primul de acest fel pentru şcoli normale. Termenul ”Normală” asociat de Titu Maiorescu acestui tip de şcoală vine
din Apus. De altfel, la propunerea lui Institutul Preparandal este numit Institutul lui Vasile Lupu. Au fost utilizate şi alte denumiri: Şcoala Normală de la
Trei-Sfetite etc., ı̂n cele din urmă s-a statornicit numele Şcoala Normală ”Vasile
Lupu”. Începând cu anul şcolar 1864-1865, s-a trecut la doi ani de studiu. A pus
bazele şcolii primare de aplicaţie, instituţie ı̂n care elevii urmau să deprindă arta
predării lecţiilor.
La 1 februarie 1868 Titu Maiorescu este ı̂nlocuit la direcţie, dar cadrul activităţii
şcolii fixat de el a dăinuit ı̂ncă timp de aproape 20 de ani. Pe parcursul celor două
ministeriate ale sale a continuat să vegheze şi să susţină Şcoala Normală ”Vasile Lupu”
cât şi ı̂ntregul ı̂nvăţământ primar şi normal-primar.
Directoratul lui Constantin Meissner, 1886-1892, a ridicat prestigiul şcolii prin
importante shimbări (organizare internă, programe, manuale, grădina şi atelierele
şcolii etc.) şi i-a dat perspectiva de dezvoltare pentru o perioadă lungă de timp. În
1891 şcoala se mută ı̂n noul local de la via ”Pester” din Breazu-Copou, care este şi
actualul ei local. C. Meissner este ctitorul şcolii primare de aplicaţie, un local anexă
al şcolii ı̂n care viitorii ı̂nvăţători deprindeau arta predării.
Cerinţe sporite de la etapă la etapă au făcut ca numărul anilor de studii să crească.
Şcoala Normală a trecut treptat la trei ani de studii (ı̂n 1874), patru ani (1877), cinci
ani (1893), şase ani (1903), ajungând la şapte ani ı̂n 1930.
Ioan Mitru, cel mai longeviv director normalist ieşean (1896-1919), a fost un
excelent organizator - ”gospodarul desăvârşit”, cum l-a caracterizat Şt. Bârsănescu. În
ferma şcolii, cu sprijinul agronomului N. Cojocaru, elevii dobândeau ştiinţa lucrărilor
agricole. Viitorii ı̂nvăţători, prin ceea ce vor realiza pe lotul şcolii sau cel propriu,
urmau să fie nu numai educatori ai copiilor, ci şi ai satului. Aleile şi parcul şcolii
au fost amenajate prin proiectul inginerului peisagist al oraşului şi profesor al şcolii,
Gh. Apostolescu, ı̂ncepând cu anul 1900.
Faima şcolii ieşeane trecuse dincolo de graniţele ţării. Profesorii erau dintre cei
mai aleşi din Iaşi. Elevii purtau uniforme şi, o parte, activau ı̂n formaţii artistice
sau sportive: cor, orchestră, fanfară, formaţia de dansuri, echipa de oină etc., renumite pentru nivelul artistic sau competivitatea lor. Biblioteca şcolii, iniţiată de
Titu Maiorescu, şi-a sporit fondul prin strădaniile şi donaţiile profesorilor P. Cujbă,
C. Meissner, I.V. Praja, I. Mitru şi, mai apoi, Şt. Bârsănescu, V. Petrovanu ş.a.
Urgia războiului s-a abătut şi asupra Şcolii Normale din Iaşi: ı̂n anul şcolar 19161917 cursurile au fost suspendate, iar ı̂n cel următor doar clasa a IV-a a funcţionat
49
ı̂n localul şcolii, celelalte fiind repartizate ı̂n diferite case din Iaşi sau alte localităţi.
Între cele două războaie mondiale, la conducerea Şcolii Normale vin câţiva pedagogi eminenţi, ce aduc ı̂nnoiri ı̂n pas cu ultimele curente pedagogice apărute ı̂n
Europa: Vasile Todicescu - adept al pedagogiei experimentale (a creat un laborator de psihologie experimentală), Ştefan Bârsănescu – promotor al unei politici
şcolare bazată pe cultură şi care să fie ı̂n ritm cu transformările sociale, Gheorghe
Comicescu – adept al integralismului pedagogic. Vasile Petrovanu, distins profesor de istorie şi director, 1938-1941, a făcut istoricul Şcolii Normale de la origini
şi până ı̂n 1892 şi a contribuit la apariţia anuarelor şcolii din această perioadă. În
septembrie 1939, se deschid, numai la această şcoală, două secţii cu profil practic:
agricolă şi industrială. Către sfârşitul perioadei interbelice Şcoala Normală atinge cel
mai ı̂nalt nivel de organizare, realizări şi concepţie. Ministrul instrucţiunii de atunci,
Petre Andrei, o considera cea mai bună şcoală de acest tip din Peninsula Balcanică.
Al Doilea Război Mondial şi instaurarea regimului comunist au cauzat Şcolii Normale un şir lung de greutăţi, schimbări de profil şi programe, contopiri cu alte unităţi
şcolare similare, schimbări de nume, peregrinări prin diverse localuri etc.
Clădirea şcolii s-a aflat ı̂n zona frontului şi a fost avariată: o bombă a distrus
laboratorul de fizică şi chimie, aviaţia inamică a distrus acoperişul, armata germană
a transformat sălile de clasă ı̂n grajd de cai. Localul şcolii de aplicaţie a ars până la
temelie. Cel mai grav lucru a fost pierderea bibliotecii cu fondul ei preţios de cărţi.
Să notăm şi câteva date din odiseea şcolii: martie 1944 – refugiul la Craiova şi
apoi com. Pocruia-Gorj; martie 1945 – revenirea la Iaşi şi reluarea cursurilor ı̂n localul sumar reparat; decembrie 1954 – mutarea abuzivă ı̂n clădirea Şcolii Normale
de ı̂nvăţătoare ”Mihail Sturdza”, str. Toma Cozma (actualul corp D al Universităţii);
1955 – se unifică cu şcoala gazdă şi şi devine mixtă sub numele de Liceul Pedagogic;
acesta se mută, ı̂n anul şcolar 1958-1959, ı̂n clădirea din dealul Copoului; septembrie
1959 – transferarea la complexul şcolar din Bârlad; septembrie 1966 – revine ı̂n Iaşi
(fără dotare), cu titulatura de Liceul pedagogic ”Vasile Lupu”, ı̂n localul din str.
M. Kogălniceanu, iar internatul ı̂n alte două locuri; 1970 – se mută ı̂ntr-un local nou şi
corespunzător din cartierul Tudor Vladimirescu; 1977 – liceul va avea profil nu numai
de ı̂nvăţători, ci şi de educatoare, prin unificarea cu Şcoala de conducătoare (ı̂nfiinţată
ı̂n Iaşi ı̂n 1919); 1985 – Liceul pedagogic ”Vasile Lupu” revine la propriul lui local din
Breazu-Copou. Perioada de după anul 1990 a ı̂nsemnat o nouă perioadă de incertitudini cauzate de multele schimbări de profil şi structură. Şcoala a supravieţuit şi a
trecut peste multele momente dificile datorită devotamentului, abnegaţiei şi spiritului
de ı̂naltă responsabilitate ale corpului profesoral faţă de generaţiile tinere. Din galeria
profesorilor care au slujit cu dragoste şcoala, spicuim doar câteva nume: A. Hasgan,
V. Mitrofan, V. Fetescu, S. Rădoi, R. Măcăreanu şi mulţi alţii.
Intr-o societate care nu are o politică educaţională clară, Şcoala Normală ”Vasile
Lupu” ı̂ncearcă să păstreze tradiţia normalistă şi să fie fidelă sistemului de valori universale – garanţii sigure ale afirmării sale.
Prof. Viorel PARASCHIV
Director al Şcolii Normale, 2002-2006
50
Concursul ”Recreaţii Matematice”
Ediţia a VII-a, Durău, 28 august 2009
Clasa a V-a
1. a) Completaţi şirul următor cu ı̂ncă trei termeni: 1, 3, 7, 15, 31, 63, ... .
b) Puneţi paranteze ı̂n expresia 2 · 12 + 18 : 6 + 1 astfel ı̂ncât rezultatul să fie
numărul natural cel mai mic posibil.
2. Câtul ı̂mpărţirii a două numere naturale este 3 iar restul este 4. Dacă dublăm
deı̂mpărţitul şi păstrăm ı̂mpărţitorul, atunci restul obţinut este 3. Determinaţi numerele iniţiale.
3. Paginile unei cărţi sunt numerotate de la 1 la 336. Din această carte se rup, la
ı̂ntâmplare, 111 foi. Să se arate că:
a) suma numerelor de pe foile rămase nu se ı̂mparte exact la 10;
b) produsul numerelor de pe foile rămase se ı̂mparte exact la 3.
Clasa a VI-a
1. a) Utilizând operaţiile cunoscute (adunarea, scăderea, ı̂nmulţirea, ı̂mpărţirea
sau ridicarea la putere) determinaţi cel mai mare număr natural care se poate scrie,
folosind o singură dată cifrele 1, 2 şi 3. Justificaţi.
Petre Bătrâneţu
b) Fie a, b ∈ N. Să se arate că, dacă ultima cifră a numărului a2 + b2 este 9, atunci
ultima cifră a lui (a + b)2 este tot 9.
Recreaţii Matematice - 2/2007
2. Într-un regat sunt 2009 oraşe. Regele a poruncit să se realizeze drumuri ı̂ntre
oraşe astfel ı̂ncât din fiecare oraş să pornească exact 5 drumuri. Au putut supuşii să
ı̂ndeplinească ordinul regelui?
Mihai Crăciun
3. Fie mulţimea A = {1, 2, 3, ..., 98}. Arătaţi că oricum am alege 50 de elemente
ale lui A, există două printre ele având suma cub perfect.
Titu Zvonaru
Clasa a VII-a
•
1. a) Fie numărul A=aa...aa
| {z } bb...bb
| {z } . Arătaţi că A=
˜
10n − 1 a(10n − 1) a + b
.
·
+
3
3
3
de n ori de n ori
b) Arătaţi că numărul B = 44...4
| {z } 22...2
| {z } poate fi scris ca un produs de două
de n ori de n ori
numere naturale consecutive.
Mihai Crăciun
2. Demonstraţi că triunghiul determinat de picioarele bisectoarelor unui triunghi
cu un unghi de măsură 1200 este dreptunghic.
3. Fie m şi n numere naturale nenule cu proprietatea că m ≤ 1 + 2 + ... + n. Să se
arate că m poate fi scris ca suma câtorva numere distincte dintre numerele 1, 2, ..., n,
unde n ≥ 3.
Recreaţii Matematice - 2/2008
Clasa a VIII-a
1. Fie x, y ∈ Z. Dacă x2010 +y 2010 se divide cu 11, arătaţi că x+y se divide cu 11.
Andrei Nedelcu
51
2. Fie triunghiul △ABC ı̂nscris ı̂n cercul C(O, R). Se cere locul geometric al
punctelor M din planul triunghiului, pentru care are loc relaţia M A2 +M B 2 +M C 2 =
3R2 .
Dan Brânzei
1
3
1
x
y+2
3. Determinaţi x, y, z pentru care x + y + z = 2 + 3 · 2
+ 2z+2 = 9.
2
2
2
Recreaţii Matematice - 1/2008
Clasa a IX-a
1. Rezolvaţi, ı̂n necunoscuta (x; y) ∈ Q × Q, ecuaţia x2009 + y 2009 = x2010 + y 2010 .
2. Într-un careu cu 41 linii şi 49 coloane se scriu la ı̂ntâmplare 2009 numere reale
distincte. Fie A mulţimea ce are ca elemente cele mai mici numere de pe fiecare linie,
respectiv B mulţimea ce are ca elemente cele mai mari numere de pe fiecare coloană.
Determinaţi probabilitatea ca cel mai mare element din mulţimea A să fie chiar cel
mai mic element din mulţimea B.
3. Fie D un punct ı̂n planul unui triunghi echilateral △ABC ı̂ncât BD = DC,
m(^BDC) = 300 şi dreapta BC separă A şi D. Dacă E ∈ (BD) şi m(^BAE) = 150 ,
să se arate că CE⊥AC.
Recreaţii Matematice - 2/2007
Clasa a X-a
1. Rezolvaţi, ı̂n necunoscuta (x; y) ∈ N × N, ecuaţia x · (x + 2) · (x + 8) = 3y .
2. Fie triunghiul △ABC cu m(^ABC) = m(^ACB) = 800 şi P ∈ (AB) astfel
ı̂ncât m(^BP C) = 300 . Arătaţi că AP = BC.
3. Determinaţi funcţiile f : N → N, pentru care are loc egalitatea 2 · f (n + 3) ·
f (n + 2) = f (n + 1) + f (n) + 1, ∀n ∈ N.
Recreaţii Matematice - 2/2007
Clasa a XI-a
1. Fie triunghiul △ABC nedreptunghic. Paralela prin B la AC şi simetrica
dreptei AC ı̂n raport cu BC se intersectează ı̂n A1 ; analog se obţin punctele B1 şi
C1 . Dacă dreptele AA1 , BB1 , CC1 sunt concurente, să se arate că triunghiul △ABC
este echilateral.
Recreaţii Matematice - 1/2006
2. Fie funcţia f : A → A, unde A este o mulţime finită din R. Dacă |f (x)−f (y)| <
|x − y|, ∀x, y ∈ A, x ̸= y, să se arate că funcţia f nu este nici injectivă, nici surjectivă.
Lucian• Georges
Lăduncă
˜
3xn
+ 1, ∀n ∈ N∗ .
3. Fie şirul (xn )n≥1 definit recurent prin x1 ∈ N, xn+1 =
2
Este posibil să alegem x1 număr natural, astfel ı̂ncât primii 2008 termeni ai şirului să
fie numere pare şi al 2009-lea să fie impar?
Lucian Georges Lăduncă
Clasa a XII-a
π 1. Fie şirul (xn )n≥1 dat de x1 ∈ − , 0 , xn+1 =2xn −tg xn , ∀n ≥ 1. Să se studieze
√4
existenţa limitelor lim xn şi lim n −xn .
Recreaţii Matematice - 1/2006
n→∞
n→∞
2. Să se determine funcţiile derivabile f : I → (0; +∞) şi intervalul I ⊂ R, ştiind
că f (0) = 1 şi f 3 (x) + f ′ (x) = 0, ∀x ∈ I.
Gabriel Mı̂rşanu
3. Fie funcţia g : [0; 1] → R derivabilă pe (0; 1) cu g(0) = 0, iar f : [0; 1] → R+
o funcţie cu proprietatea f (x) = g ′ (x), ∀x
[0; 1]. Să se arate că există cel puţin un
π∈ π
c < f (c).
Gabriel Mı̂rşanu
punct c ∈ (0; 1) astfel ı̂ncât · g(c) · cos
2
2
52
Concursul de matematică ”Gaudeamus”
Ediţia I, Iaşi, 2009
În perioada 30 octombrie - 1 noiembrie 2009, la Facultatea de Matematică a Universităţii
”Alexandru Ioan Cuza” Iaşi, s-a desfăşurat prima ediţie a Concursului de Matematică
”Gaudeamus”.
Organizatorii manifestării sunt: Facultatea de Matematică şi Fundaţia Seminarului Matematic ”Alexandru Myller”. Colaboratori: Inspectoratul Şcolar Judeţean Iaşi şi Liceul de
Informatică ”Grigore Moisil” Iaşi.
Manifestarea s-a desfăşurat pe două secţiuni: 1)proba scrisă individuală; 2) concursul de
proiecte ”Matematica ı̂n lumea reală”.
1) Proba scrisă individuală s-a adresat elevilor claselor a X-a, a XI-a, a XII-a şi a
conţinut subiecte din programele şcolare ale anilor anteriori; tematica a fost publicată pe
site-ul Facultăţii de Matematică, www.math.uaic.ro . Listele cu elevii participanţi, precum
şi rezultatele lor, pot fi consultate la aceeaşi adresă internet.
2) Concursul de proiecte ”Matematica ı̂n lumea reală” s-a adresat elevilor sau
echipelor de elevi (maxim patru), fără limitare de vârstă, şi a constat ı̂n prezentarea unor
soluţii matematice la probleme concrete ale lumii reale. Au fost susţinute 12 proiecte.
Acest concurs doreşte să fie, ı̂n primul rând, un mijloc de apropiere ı̂ntre Facultatea de
Matematică şi elevii din ı̂nvăţământul preuniversitar. Numărul de participanţi (aproximativ
150 de elevi din judeţele Bacău, Botoşani, Neamţ, Vaslui şi Iaşi), precum şi rezultatele
obţinute, sunt ı̂ncurajatoare.
Subiectele propuse la proba scrisă individuală
Clasa a X-a
1. i) Să se arate că, dacă 3 | a2 + b2 , unde a, b ∈ N, atunci 3 | a şi 3 | b.
ii) Să se arate că nu există (a, b, c, d) ∈ N4 , (a, b, c, d) ̸= (0, 0, 0, 0) astfel ı̂ncât
a2 + b2 = 3(c2 + d2 ).
2. Se consideră n puncte S = {A1 , A2 , ..., An } dintr-un plan π şi n numere reale
Λ = {λ1 , λ2 , ..., λn } astfel ı̂ncât λ1 + λ2 + · · · + λn ̸= 0. Spunem că A′ ∈ π este centrul
de masă al sistemului (S, Λ) dacă există un punct O ∈ π astfel ı̂ncât
−−→′
−−→
−−→
λ1
λn
OA =
OA1 + · · · +
OAn .
λ1 + · · · + λn
λ1 + · · · + λn
′
i) Să se arate că dacă A este centrul de masă al sistemului (S, Λ) atunci pentru
orice punct M ∈ π are loc
−−→′
−−−→
−−−→
λn
λ1
M A1 + · · · +
M An .
MA =
λ1 + · · · + λn
λ1 + · · · + λn
Pentru un triunghi A1 A2 A3 , considerăm λ1 , λ2 , λ3 ∈ R astfel ı̂ncât |λ1 |, |λ2 | şi |λ3 |
reprezintă lungimile laturilor [A2 A3 ], [A3 A1 ] şi respectiv [A1 A2 ].
ii) Să se arate că A′ ∈ A1 A2 este piciorul bisectoarei interioare (respectiv exc3 dacă şi numai dacă A′ este centrul de masă al sistemului
terioare) a unghiului A
({A1 , A2 }, {λ1 , λ2 }), pentru o anume alegere a semnelor scalarilor λ1 şi λ2 .
iii) Să se arate că I ∈ π este centrul cercului ı̂nscris (respectiv centrul unui cerc
exı̂nscris) triunghiului A1 A2 A3 dacă şi numai dacă I este centrul de masă al sistemului
({A1 , A2 , A3 }, {λ1 , λ2 , λ3 }), pentru o anume alegere a semnelor scalarilor λ1 , λ2 şi λ3 .
53
3. Fie n ∈ N\0, 1} şi a1 , a2 , ..., an numere reale. Construim: s0 = −a1 −a2 −...−an ;
sm = a1 +a2 +...+am −(am+1 +am+2 +...+an ), ∀m ∈ 1, n − 1; sn = a1 +a2 +...+an .
Să se arate că există un m ∈ 0, n astfel ı̂ncât |sm | ≤ max{|ak | | k ∈ 1, n}.
Clasa a XI-a
1. Două sute de elevi participă la un concurs de matematică, la care se propun
6 probleme. După corectare, se observă că fiecare problemă a fost rezolvată corect
de cel puţin 120 de elevi (nu neapărat aceiaşi pentru fiecare problemă). Arătaţi că
se pot găsi doi participanţi, astfel ca fiecare problemă să fi fost rezolvată de cel puţin
unul din cei doi.
2. i) Fie A = {a1 , a2 , ..., an } ⊆ Z şi f : A → A o funcţie bijectivă. Presupunând
că a1 < a2 < ... < an şi că ai +f (ai ) < aj +f (aj ) pentru orice i < j, i, j ∈ {1, 2, ..., n},
să se arate că f = 1A . ii) Să se găsească o funcţie bijectivă f : Z →Z care satisface
condiţiile m + f (m) < n + f (n) pentru orice m < n, m, n ∈ Z, şi să fie diferită de 1Z .
3. Fie a ∈ (0, +∞). Într-un plan π, relativ la un sistem de coordonate carteziene
xOy, considerăm dreapta de ecuaţie (δa ) : y = ax. Fie Π = [0, 1) × [0, 1).
a) Definim f : R × R → Π, f (x, y) = ({x}, {y}), unde prin { } am notat partea
fracţionară. Să se arate că f este periodică, adică există T ∈ R astfel ı̂ncât f (x +
T, y + T ) = f (x, y) pentru orice (x, y) ∈ R × R.
b) Considerăm restricţia fa , a lui f la dreapta (δa ). Să se arate că dacă a ∈ Q
atunci fa este de asemenea periodică.
Imaginea lui (δa ) prin fa este o reuniune de segmente paralele cu direcţia lui (δa ).
c) Pentru a ∈ Q să se arate că numărul acestor segmente este finit.
Vom nota cu N acest număr.
10
10
d) Pentru a =
să se determine N . Cât este N pentru a =
?
2009
2010
e) Ce se poate spune
√ despre imaginea lui (δa ) prin aplicaţia fa când a este număr
iraţional (de exemplu 2)?
Clasa a XII-a
1. Fie A, B ∈ M(n, C) astfel ı̂ncât A + B = In , A2 = A3 . Să se arate că:
i) AB = BA; ii) In − AB şi In + AB sunt inversabile.
√
2. Se consideră funcţia f : (−1, +∞) → R , f (x) = 1 + x.
i) Să se scrie ecuaţia tangentei y = ax + b la graficul funcţiei f , ı̂n punctul x0 = 0.
f (x) − 1 − x2
ii) Să se determine valoarea a pentru care lim
există şi este ̸= 0.
x→0
xa 
‹
1 1
x x2
iii) Arătaţi că, pentru x ∈ − ,
are loc inegalitatea: f (x) − 1 − ≤ √ .
2 2
2
2 2
iv) Fie N numărul natural cu 2010 cifre, toate egale cu 1, adică N = |1111{z
. . . 11}.
√
Determinaţi primele 2010 zecimale ale numărului N .
3. Se cer valorile lui a ∈ R pentru care sistemul
¨
2|x| + |x| = y + x2 + a
x2 + y 2 = 1
are o unică soluţie reală.
54
2010 cifre
Soluţiile problemelor propuse ı̂n nr. 1/2009
Clasele primare
P.164. Scrie vecinii vecinului comun al numerelor 16 şi 18.
(Clasa I)
Diana Tănăsoaie, elevă, Iaşi
Soluţie. Vecinii numărului 16 sunt numerele 15 şi 17, iar ai numărului 18 sunt
numerele 17 şi 19. Vecinul comun este 17. Vecinii numărului 17 sunt 16 şi 18.
P.165. După ce dau celor doi fraţi mai mari câte două banane, mănânc şi eu trei
banane. În coş ı̂mi rămâne un număr de banane ce poate fi scris cu două cifre diferite
şi care este cel mai mic număr de acest fel. Câte banane am avut ı̂n coş?
(Clasa I )
Inst. Maria Racu, Iaşi
Soluţie. Numărul bananelor rămase ı̂n coş este 10. Numărul iniţial de banane
din coş este 2 + 2 + 3 + 10 = 17.
P.166. Din cei 8 căţeluşi albi şi negri, cel mult 3 sunt albi. Care este numărul
maxim de căţeluşi negri? Dar cel minim?
(Clasa a II-a)
Ioana Bărăgan, elevă, Iaşi
Soluţie. Deoarece avem căţeluşi albi şi negri, cel puţin un căţeluş este alb.
Numărul maxim de căţeluşi negri este 7. Cum cel mult 3 căţeluşi sunt albi, cel
puţin 5 vor fi negri.
P.167. Într-o cameră se joacă un pisoi cu doi pisici, un căţeluş care ţine ı̂n gură
o păpuşă şi un băieţel, călare pe un căluţ de lemn. Câte picioare participă la joc?
(Clasa a II-a)
Alexandru Dumitru Chiriac, elev, Iaşi
Soluţie. La joc participă pisoiul cu cei doi pisici, căţeluşul şi băieţelul. În total
participă la joc 4 + 4 + 4 + 4 + 2 = 18 picioare.
P.168. Există numerele naturale a, b, c, d astfel ı̂ncât a + b + c + d = 123 şi
a : b = b : c = c : d = 1?
(Clasa a III-a)
Amalia Cantemir, elevă, Iaşi
Soluţie. Din a : b = b : c = c : d = 1, rezultă a = b = c = d. Concluzionăm că
numărul 123 trebuie să se ı̂mpartă exact la 4, ceea ce este fals.
P.169. Calculează diferenţa următoare, fără a efectua parantezele: (2 + 4 + 6 +
8 + . . . + 1000) − (1 + 3 + 5 + 7 + . . . + 999) =
(Clasa a III-a)
Mădălina Bucşă, elevă, Iaşi
Soluţie. De la 1 la 1000 sunt 500 numere pare şi 500 numere impare. Exerciţiul
devine (2 − 1) + (4 − 3) + (6 − 5) + . . . + (1000 − 999) = 1 + 1 + 1 + . . . + 1 = 500.
|
{z
}
de 500 ori
P.170. Doi fraţi au cumpărat un teren ı̂n formă de pătrat pe care l-au ı̂mpărţit
ı̂n două dreptunghiuri egale. Fiecare doreşte să ı̂mprejmuiască propriul teren cu gard.
Cât mai are de lucru fiecare, dacă primul a realizat 430 m, al doilea 470 m, iar
perimetrul pătratului este de 1000 m?
(Clasa a III-a)
Dragoş Iacob, elev, Iaşi
55
Soluţie. Latura pătratului este de 1000m : 4 = 250m. Împrejmuirea fiecărui frate
cuprinde jumătate din perimetrul pătratului şi ı̂ncă jumătate din porţiunea comună
de gard. Primul frate mai are de lucrat (500m + 125m) − 430m = 195m, iar al doilea
(500m + 125m) − 470m = 155m.
P.171. Dacă a+b+c = 175 şi a+2c = 200, calculaţi produsul (2a+b+3c)·(c−b).
(Clasa a IV-a)
Inst. Marian Ciuperceanu, Craiova
Soluţie. 2a + b + 3c = (a + b + c) + (a + 2c) = 175 + 200 = 385. Din a + b + c = 175
şi a + c + c = 200, rezultă c − b = 200 − 175 = 25. Aşadar, (2a + b + 3c) · (c − b) =
375 × 25 = 275 × 100 : 4 = 9375.
P.172. Câte numere abc au suma cifrelor 7 şi pot fi rotunjite cu numărul ab0?
(Clasa a IV-a)
Maria Nastasiu, elevă, Iaşi
Soluţie. Cifra unităţilor poate fi 0, 1, 2, 3 sau 4. Dacă c = 0, atunci a + b = 7
şi găsim numerele 160, 250, 340, 430, 520, 610 şi 700. Dacă c = 1, atunci a + b = 6
şi obţinem numerele 151, 241, 331, 421, 511 şi 601. Analog, ı̂n celelalte cazuri găsim
numerele 142, 232, 322, 412, 502; 133, 223, 313, 403; 124, 214 şi 304. În total, există 25
de numere cu proprietăţile din enunţ.
P.173. Se formează şirul de numere: 34, 334, 344, 3334, 3444, . . .. Câte cifre de
3 are numărul de pe locul 2008?
(Clasa a IV-a)
Petru Asaftei, Iaşi
Soluţie. Numerele de pe locurile pare au o singură cifră de 4, restul fiind cifre de
3. Astfel, numărul de pe locul 2008 are 2008 : 2 + 1 = 1005 cifre de 3.
Clasa a V-a
V.102. Un ı̂ntreprinzător doreşte să cumpere un număr de frigidere de la un
angrosist, pe care urmează să le transporte către firma sa cu ajutorul unui camion de
mare tonaj, care consumă 10 l de motorină la 100 km (1l de motorină costă 3 lei).
Întreprinzătorul poate opta ı̂ntre doi furnizori: A vinde frigiderul cu 1000 lei/buc.,
iar B vinde acelaşi produs cu 990 lei/buc., ı̂nsă are depozitul mai departe decât A, la
o distanţă pe şosea AB = 150 km.
a) Dacă ı̂ntreprinzătorul doreşte să cumpere 20 de frigidere, ce furnizor va alege?
b) La ce număr de frigidere costurile de achiziţie nu depind de furnizor?
Marian Ciuperceanu, Craiova
Soluţie. Fie n numărul de frigidere achiziţionate, iar x cheltuielile de transport
de la firma ı̂ntreprinzătorului până la furnizorul A. Cheltuielile de transport până
la furnizorul B vor fi x + 3 · 10 · 3 = x + 90 (distanţa AB se parcurge dus-ı̂ntors, ı̂n
total 300 km). Chletuielile totale cu furnizorul A vor fi CA = x + 1000n, iar cele cu
furnizorul B vor fi CB = x + 90 + 990n.
a) Dacă n = 20, atunci CA = x+20000, iar CB = x+19890, prin urmare CB < CA .
Furnizorul ales va fi B.
b) Avem că CA = CB , de unde se obţine n = 9.
3x + 5
2y + 5
5z + 2
V.103. Se consideră numerele naturale m =
, a=
, b=
,
2x + 2
3
5
unde x, y, z ∈ N. Demonstraţi că m nu poate fi divizor al lui a, dar poate fi divizor al
lui b.
Claudiu Ştefan Popa, Iaşi
56
Soluţie. Pentru că m ∈ N, trebuie să avem că 2x+2 | 3x+5, de unde 2x+2 | 2(3x+
5
5) − 3(2x + 2), adică 2x + 2 | 4. Găsim că x ∈ {0, 1}; dacă x = 0, atunci m = ∈
/ N,
2
iar dacă x = 1, atunci m = 2. Presupunând că 2 | a, s-ar obţine că 2y + 5 = 6k, cu
y, k ∈ N, absurd (membrul stâng este impar, iar cel drept par). Pentru z = 2, avem
că b = 4, număr care se divide cu 2.
V.104. Scrieţi numărul 2008 ca sumă de trei cuburi perfecte pare. (Găsiţi toate
posibilităţile!)
Veronica Plăeşu şi Dan Plăeşu , Iaşi
Soluţie. Deoarece 2008 = 23 · 251, este destul să-l scriem pe 251 ca sumă de
trei cuburi perfecte. Cel mai mare dintre cele trei cuburi nu poate depăşi 261 = 63 ,
deoarece 73 = 343 > 251. După o analiză a cazurilor posibile, găsim doar două situaţii
favorabile: 251 = 13 +53 +53 şi 251 = 23 +33 +63 . În concluzie, 2008 = 23 +103 +103 =
23 + 63 + 123 .
V.105. Se consideră numărul a = 7 + 72 + 73 + . . . + 72009 .
a) Demonstraţi că a nu poate fi pătrat perfect.
b) Aflaţi restul ı̂mpărţirii lui a la 400.
Damian Marinescu, Târgovişte
Soluţie. a) Cum a se divide cu 7, dar nu şi cu 72 , ı̂nseamnă că nu poate fi pătrat
perfect.
b) Avem că a = 7 + 72 (1 + 7 + 72 + 73 ) + . . . + 72006 (1 + 7 + 72 + 73 ) = 7 + 400(72 +
. . . + 72006 ), deci restul ı̂mpărţirii lui a la 400 este 7.
V.106. Să se determine numărul natural a şi cifra b, dacă (a + 3) · 200b = a · 2009.
Enache Pătraşcu, Focşani
.
.
2
Soluţie. Cum 2009 = 7 · 41, rezultă că (a + 3) · 200b..72 şi (a + 3) · 200b..41.
Evident că b ≤ 8 (deoarece a + 3 > a), iar dintre numerele 2000, 2001, . . . , 2008,
.
.
nicunul nu se divide nici cu 72 , nici cu 41. Deducem că a + 3..7 şi a + 3..41, prin
urmare α + 3 = 287k. Înlocuind, obţinem că 200b · k = 7(287k − 3), de unde k(2009 −
200b) = 21. De aici, (k, b) ∈ {(21, 8); (7, 6); (3, 2)}, deci soluţiile problemei sunt (a, b) ∈
{(6024, 8); (2006, 6); (858, 2)}.
O altă rezolvare se poate da ı̂ncercând pentru b fiecare dintre valorile 0, 1, 2, . . . , 8;
se obţin astfel nouă ecuaţii simple, doar trei dintre acestea având soluţii naturale.
V.107. Dacă n ∈ N\{0, 1} este dat, determinaţi x, y ∈ N∗ pentru care x(x + 2y +
1) = 2n · 135.
Petru Asaftei, Iaşi
Soluţie. Dacă x = 2i , 1 ≤ i ≤ n − 1, atunci am avea că 2i + 2y + 1 = 2n−i · 135,
contradicţie (membrul stâng este impar, iar cel drept este par). Analog se arată că
nu putem avea x = 2i · p, unde 1 ≤ i ≤ n − 1, p ∈ D135 \{1}. Rămâne că x = 2n · p,
cu p ∈ D135 . Cum x < x + 2y + 1, trebuie cercetate doar cazurile ı̂n care p ∈ {1, 3, 5}.
Dacă x = 2n , obţinem că y = 67−2n−1 , iar y ∈ N∗ doar pentru n ≤ 7. Dacă x = 2n ·3,
atunci y = 22−3·2n−1 , care este număr natural când n ≤ 3. În sfârşit, dacă x = 2n ·5,
atunci y = 13 − 5 · 2n−1 , soluţie convenabilă pentru n ≤ 2. În concluzie, obţinem 3, 2, 1
sau 0 perechi (x, y), după cum n = 2, n = 3, n ∈ {4, 5, 6, 7}, respectiv n ≥ 8.
57
V.108. Pe tablă sunt scrise numerele 2, 0, 0, 9. Putem şterge de pe tablă oricare
două numere, scriind ı̂n loc succesorii acestora. Este posibil ca, ı̂n urma mai multor
operaţii de acest fel, să obţinem patru numere egale?
Cătălin Budeanu, Iaşi
Soluţie. Suma numerelor de pe tablă creşte cu 2 la fiecare pas. După a n-a
operaţie, ea devine 2n + 2 + 0 + 0 + 9 = 2n + 11, număr care este impar, deci nu poate
fi suma a patru numere egale.
Clasa a VI-a
VI.102. O asociaţie de locatari este formată din trei familii care au consumat
ı̂ntr-o lună 27m3 , 16m3 , respectiv 4m3 de apă potabilă. Din consumul total, pentru
38m3 de apă trebuie plătită o taxă de canalizare, care se ı̂mparte proporţional cu
consumul fiecărei familii. Dacă preţul apei este de 1, 6 lei/m3 , taxa de canalizare este
de 0, 56 lei/m3 şi fiecărei sume i se aplică T.V.A. de 19 %, aflaţi ce sumă trebuie să
plătească fiecare familie (efectuaţi calculele cu două zecimale exacte).
Petru Asaftei, Iaşi
Soluţie. Pentru apă, prima familie plăteşte 27 × 1, 6 × 1, 19 = 51, 4 lei, a doua
16 × 1, 6 × 1, 19 = 30, 46 lei, iar a treia 4 × 1, 6 × 1, 19 = 7, 61 lei (am trunchiat
rezultatele la cifra sutimilor, conform cerinţelor problemei). Observând că 38m3
reprezintă 80, 85% din 47m3 (unde 47m3 este consumul total), deducem că pentru
canalizare prima familie plăteşte 0, 8085 × 28 × 0, 56 × 1, 19 = 14, 54 lei, a doua
0, 8085 × 16 × 0, 56 × 1, 19 = 8, 62 lei, iar a treia 0, 8085 × 4 × 0, 56 × 1, 19 = 2, 15 lei.
În total, prima familie are de plătit 65, 94 lei, a doua 39, 08 lei, iar a treia 9, 76 lei.
VI.103. Să se determine numărul prim p şi numerele ı̂ntregi a şi x pentru care
(x − a)(x − 1)(a − 1) = p.
Gheorghe Iurea, Iaşi
Soluţie. Dacă p ≥ 3, atunci toţi divizorii săi ar fi impari, deci x − a, x − 1 şi a − 1
vor fi numere impare. Însă (x − 1) − (x − a) = a − 1, egalitate care nu poate avea loc
pentru trei numere impare. Rămâne să studiem cazul p = 2; atunci a − 1 ∈ {±1, ±2},
deci a ∈ {−1, 0, 2, 3}. Considerând fiecare dintre cele patru situaţii, găsim soluţiile
a = −1, x = 0 şi a = 2, x ∈ {0, 3}.
VI.104. Determinaţi numerele prime p şi q, ştiind că există x, y ∈ N∗ astfel ı̂ncât
x + y 2 = p, iar x + y + 1 = q.
Andrei Cozma, elev, Bucureşti
Soluţie. Observăm că p − q = x(x − 1) + y(y − 1) − 1, prin urmare p − q este
număr impar. Rezultă că unul dintre numerele p sau q este par, deci egal cu 2. Dacă
p = 2, din x2 + y 2 = 2 obţinem că x = y = 1, prin urmare q = 3. Dacă q = 2, atunci
x + y = 1, fals, deoarece x, y ∈ N∗ . În concluzie, p = 2 şi q = 3.
2
2009
2008
VI.105. Să se arate că numărul N = 33
− 33
se poate scrie ca produs a
trei numere naturale consecutive.
Dan Nedeianu, Drobeta-Tr. Severin
2008
2009
2008
Soluţie. Notând a = 33 , observăm că 33
= 33 ·3 = a3 . Astfel, N =
a3 − a = a(a2 − 1) = (a − 1) · a · (a + 1), ceea ce ı̂ncheie rezolvarea.
Ô şi punctele A, B ∈ (Ox, C, D ∈ (Oy astfel ı̂ncât
VI.106. Se consideră unghiul xOy
58
A ∈ (OB), iar C ∈ (OD). Mediatoarele segmentelor [AB] şi [CD] se intersectează ı̂n
Õ ≡ SCD.
Õ
S, iar SAB
a) Demonstraţi că BC = AD.
Õ = 60◦ , arătaţi
b) Dacă, ı̂n plus, punctele B, D şi S sunt coliniare, iar m(SAB)
că AC⊥SC ⇔ BS = 2 · SD.
Romanţa Ghiţă şi Ioan Ghiţă, Blaj
Õ = SCD
Õ
Soluţie. a) Triunghiurile SAB si SCD fiind isoscele, din ipoteza SAB
◦
◦
O
Õ = 180 − 2m(SCD),
Õ deci ASB
Õ=
obţinem că 180 − 2m(SAB)
Õ
Õ
Õ
CSD. Atunci BSC ≡ ASD, ceea ce ne arată că △BSC ≡
△ASD (L.U.L.), de unde BC = AD.
A
b) În ipotezele acestui punct, triunghiurile ABS, CDS şi
Q
C
OBD sunt echilaterale, iar AS∥Oy, CS∥Ox. Dacă AC ⊥ SC,
◦
◦
Õ = 60 , atunci m(CAS)
Õ = 30 . În triunghiul B
D
cum m(ASC)
S
y
dreptunghic ACS vom avea că AS = 2CS, prin urmare BS = x
2SD. Reciproc, dacă BS = 2SD, atunci AB = 2SC. Notând cu Q mijlocul lui AB,
obţinem că AQ = CS. Avem şi că AQ∥CS, deci ACSQ este paralelogram. În plus,
CA ⊥ AB (o mediană a unui triunghi echilateral este şi ı̂nălţime); deducem că ACSQ
este dreptunghi, de unde AC ⊥ SC.
VI.107. Se consideră A, B, C, D, E, F şase puncte ı̂n plan astfel ı̂ncât AB =
CD = CF = DF = 3cm, BC = BE = CE = 5cm, iar AD = 11cm. Stabiliţi câte
drepte determină cele şase puncte.
Gabriel Popa, Iaşi
Soluţie. Cum AB + BC + CD = AD, rezultă că punctele A, B, C şi D sunt
coliniare şi se află ı̂n această ordine pe dreapta pe care o determină. Mai observăm
şi faptul că triunghiurile CDF şi BCE sunt echilaterale. În cazul ı̂n care aceste
triunghiuri se află ı̂ntr-un acelaşi semiplan faţă de dreapta AB, cele şase puncte
determină 10 drepte: AB, AE, AF, BE, BF, CE, CF, DE, DF şi EF . Dacă punctele
E şi F sunt separate de dreapta AB, atunci C, E şi F vor fi coliniare, prin urmare
EF, CE şi CF sunt una şi aceeaşi dreaptă; ı̂n acest caz, cele şase puncte determină 8
drepte.
VI.108. Un ogar situat ı̂n vârful A al unei curţi dreptunghiulare ABCD (AB =
80m, BC = 160m), porneşte ı̂n urmărirea a trei iepuri aflaţi ı̂n B, C şi D, alergând
de-a lungul gardurilor. Dacă viteza ogarului este 4m/s, iar vitezele iepurilor sunt
3m/s, aflaţi după cât timp reuşeşte ogarul să prindă fiecare iepure.
Marian Ciuperceanu, Craiova
Soluţie. Dacă ogarul porneşte către vârful B, va prinde iepurele aflat iniţial ı̂n B
după 80 : (4 − 3) = 80s. Iepurele din C va fi ajuns după (80 + 160) : (4 − 3) = 240s,
iar cel din D după (80 + 160 + 80) : (4 − 3) = 320s. Dacă ı̂nsă ogarul aleargă ı̂n sens
contrar, pornind ı̂ntâi către D, va prinde iepurele de acolo după 160 : (4 − 3) = 160s,
apoi iepurele din C după (160 + 80) : (4 − 3) = 240s, iar iepurele aflat iniţial ı̂n B
după (160 + 80 + 160) : (4 − 3) = 400s.
.
59
Clasa a VII-a
VII.102. În urma unui război dus ı̂ntre două triburi de canibali, ı̂n mâinile
ı̂nvingătorilor rămân zece prizonieri, printre care şi căpetenia ı̂nvinşilor. Şeful de
trib al ı̂nvingătorilor alege, pentru prepararea cinei, câţiva prizonieri (măcar unul), la
ı̂ntâmplare. Care este probabilitatea ca şeful tribului ı̂nvins să rămână ı̂n viaţă?
Gabriel Popa, Iaşi
Soluţie. Numărul cazurilor egal posibile este numărul submulţimilor nevide ale
mulţimii cu 10 elemente a prizonierilor, deci 210 − 1 = 1023. Căpetenia ı̂nvinşilor
rămâne ı̂n viaţă dacă se alege o submulţime nevidă a mulţimii formată din ceilalţi
nouă prizonieri, deci există 29 − 1 = 511 cazuri favorabile. Probabilitatea cerută este
511
.
1023
VII.103. Aflaţi numerele ı̂ntregi x şi y pentru care y − 4x + 6 < 0, 2y − x − 2 > 0
şi 3y + 2x − 24 < 0.
Gheorghe Iurea, Iaşi
Soluţie. Scriem cele trei condiţii din ipoteză astfel:
y < 4x − 6 (1) ; y >
x+2
24 − 2x
(2); y <
(3).
2
3
x+2
Din (1) şi (2) deducem că
< 4x − 6, deci x > 2. Din (2) şi (3) rezultă că
2
x+2
24 − 2x
<
, de unde x < 6, prin urmare x ∈ {3, 4, 5}. Din (1), (2) şi (3),
2
3
ı̂nlocuind pe rând x cu 3, 4 şi 5, obţinem soluţiile (3, 3); (3, 4); (4, 4); (4, 5); (5, 4).
Notă. La nivelul clasei a IX-a, se poate da o soluţie folosind ı̂mpărţirea planului
ı̂n regiuni.
VII.104. Spunem că un număr natural are proprietatea (P ) dacă este prim, cel
puţin egal cu 5 şi se poate scrie ca sumă de două pătrate perfecte. Dacă numerele
p1 , p2 , . . . , pn au proprietatea (P ), arătaţi că numărul A = p1 +p2 +. . .+pn +n2 −n+2
nu poate fi pătrat perfect.
Cosmin Manea şi Dragoş Petrică, Piteşti
Soluţie. Un pătrat perfect poate fi M4 sau M4 + 1, deci o sumă de două pătrate
va fi M4 , M4 + 1 sau M4 + 2. Dacă dorim ca această sumă de pătrate să fie număr
prim cel puţin egal cu 5, atunci ea va fi neapărat de forma M4 + 1. Deducem că
A = (M4 +1)+(M4 +1)+. . .+(M4 +1)+n2 −n+2 = M4 +n+n2 −n+2 = M4 +n2 +2
şi, cum n2 = M4 sau n2 = M4 + 1, atunci A = M4 + 2 sau A = M4 + 3, prin urmare
A nu poate fi pătrat perfect.
VII.105. Pentru x, y ∈ R, definim a(x, y) = min(2x − y 2 , 2y − x2 ). Arătaţi că:
a) a(x, y) ≤ 1, ∀x, y ∈ R;
b) max{a(x, y)|x, y ∈ R} = 1.
Ovidiu Pop, Satu Mare
Soluţie. a) Dacă, prin absurd, ar exista x, y ∈ R pentru care a(x, y) > 1, ar
ı̂nsemna că 2x − y 2 > 1 şi 2y − x2 > 1, pentru anumite valori ale numerelor x şi y.
Prin adunare, am obţine că (x − 1)2 + (y − 1)2 < 0, imposibil.
b) Folosind a) şi observând că a(1, 1) = 1, rezultă cerinţa.
60
VII.106. Se consideră paralelogramul ABCD, E şi F mijloacele laturilor [AB],
respectiv [AD], {G} = CE∩BD, {H} = CF ∩BD, {P } = F G∩BC, {Q} = EH ∩CD.
Arătaţi că 3EF = 2P Q.
Mirela Marin, Iaşi
DH
DF
1
Soluţie. Din △DHF ∼ △BHC, deducem că
=
= . Cum △DHQ ∼
HB
BC
2
DQ
DH
1
Q
=
= , prin urmare
△BHE, obţinem că
D
C
BE
HB
2
1
DQ
1
H
DQ = AB, adică
= . Analog se arată că
F
4
DC
4
P
BP
1
G
= , deci P Q∥BD (reciproca teoremei lui Thales),
A
E
B
BC
4
PQ
CQ
3
3
iar
=
= (teorema fundamentală a asemănării). Astfel, 2P Q = · BD =
BD
CD
4
2
1
3 · BD = 3F E (deoarece [F E] este linie mijlocie ı̂n △ABD).
2
Ò = 60◦ , L proiecţia lui A pe BC, M
VII.107. Fie ABC un triunghi cu m(C)
proiecţia lui B pe AC, iar D mijlocul lui [AB]. Demonstraţi că triunghiul DM L este
echilateral.
Neculai Roman, Mirceşti (Iaşi)
Soluţie. În triunghiurile LAB şi M AB, LD şi respectiv M D sunt mediane,
1
A
prin urmare LD = M D = AB, deci △DM L este isoscel, la fel
2
Ö
b şi, cum patruca şi triunghiul ADM . Vom avea că AM
D=A
Õ
Ò Astfel,
laterul ABLM este inscriptibil, avem şi că CM L ≡ B.
D
◦
◦
Ö
Ö
Õ
b −
M
m(DM L) = 180 − m(AM D) − m(CM L) = 180 − m(A)
◦
Ò = m(C)
Ò = 60 , deci △DM L va fi chiar echilateral.
m(B)
VII.108. Considerăm ı̂n plan trei cercuri distincte, congru- B
C
L
ente, ale căror centre nu sunt coliniare. Construiţi cu rigla şi compasul un cerc la
care cercurile date să fie tangente interior.
Adrian Corduneanu, Iaşi
Soluţie. Putem determina, folosind rigla şi compasul, centrele celor trei cercuri
date; să notăm cu A, B şi C aceste centre. Aflăm, cu rigla şi compasul, centrul O
al cercului circumscris triunghiului ABC şi punctul M de intersecţie al dreptei OA
cu cercul de centru A, astfel ı̂ncât OM > OA. Astfel, cercul de centru O şi rază
OM este cercul căutat: dacă r este raza cercurilor date, iar {N } = OB ∩ C(B, r),
{P } = OC ∩ C(C, r), atunci ON = OB + BN = OA + r = OM şi analog OP = OM.
Mai trebuie justificat că C(O, OM ) are câte un singur punct comun cu cercurile
date. Dacă, de exemplu, ar exista un al doilea punct Q comun cercurilor C(O, OM ) şi
C(A, r), atunci OQ < OA + AQ (inegalitatea triunghiului), deci OQ < OA + r = OM
şi se ajunge la o contradicţie.
Clasa a VIII-a
VIII.102. Rezolvaţi ı̂n R ecuaţia

x+2
x−1
61

‹2
+
x−2
x+1
‹2
26 x2 − 4
·
= 0.
5 x2 − 1
Vasile Chiriac, Bacău
−
Soluţie. Se impune ca x ∈ R\{1, −1}. Dacă u =
x−2
x+2
, v =
, ecuaţia
x−1
x+1
26
devine u2 + v 2 −
uv = 0 şi, cum u şi v nu pot fi simultan egale cu zero, putem
5
v
26
1
nota t = şi obţinem că t2 − t + 1 = 0, cu soluţiile t1 = , t2 = 5. Dacă v = 5u,
u
5
5
√
9 ± 113
2
, iar dacă u = 5v, deducem că
rezultă că 2x − 9x − 4 = 0, de unde x1,2 =
4
1
2x2 − 9x + 4 = 0, deci x3 = 4, x4 = . Ecuaţia din enunţ are patru soluţii reale.
2
VIII.103. Arătaţi că oricare ar fi n ∈ N∗ , există m ∈ N∗ astfel ı̂ncât n4 · m + 1
este număr compus.
Lucian Tuţescu şi Ion Vişan, Craiova
Soluţia 1 (a autorilor). Pentru m = n4 + 2, avem că n4 · m + 1 = n8 + 2n4 + 1 =
(n4 + 1)2 şi, cum n4 + 1 ≥ 2, urmează concluzia problemei.
Soluţia 2 (Titu Zvonaru). Dacă n = 1, putem lua m = pq − 1, cu p, q ∈ N,
p, q ≥ 2. Dacă n > 1, luăm m = n3k−4 , k ∈ N, k ≥ 2 şi vom avea că n4 · m + 1 =
(nk )3 + 1 = (nk + 1)(n2k − nk + 1), unde ambele paranteze sunt cel puţin egale cu 2.
VIII.104. Fie x, y, z ∈ R∗+ astfel ı̂ncât x2 y 2 +y 2 z 2 +z 2 x2 = 3x2 y 2 z 2 . Demonstraţi
1
1
1
că 2
+
+
≤ 1.
x + x + 1 y2 + y + 1 z2 + z + 1
Răzvan Ceucă, elev, Iaşi
Soluţie. Problema este oarecum ı̂nrudită cu G89 din RecMat2/2005 sau cu
VIII.66 din RecMat 1/2006: observând că x2 + x + 1 ≥ 3x şi ţinând seama de faptul
1
1
că x > 0, obţinem că 2
≤
şi ı̂ncă două relaţii analoage. Astfel, membrul
x 
+x+1
3x
‹
1 1
1 1
stâng este majorat de
. Folosind inegalitatea dintre media aritmetică
+ +
3 x y zr

‹

‹
1 1
1 1
1 1
1
1
şi cea pătratică,
≤
. Însă condiţia din ipoteză
+ +
+
+
3 x y z
3 x2
y2
z2
1
1
1
se poate scrie sub forma 2 + 2 + 2 = 3 şi de aici urmează inegalitatea din enunţ.
x
y
z
Egalitatea se atinge pentru x = y = z = 1.
VIII.105. Determinaţi x, y ∈ N∗ pentru care x3 − y 3 = 3xy + 17.
Liviu Smarandache şi Ion Vişan, Craiova
Soluţia 1 (a autorilor). Cum 3xy + 17 ∈ N∗ , rezultă că x > y, deci există p ∈ N∗
astfel ı̂ncât x = y + p. Înlocuind ı̂n relaţia din enunţ, obţinem că 3(p − 1)y 2 + 3p(p −
1)y + p3 − 17 = 0. Observăm că 3(p − 1)y 2 ≥ 0 şi 3p(p − 1)y ≥ 0, prin urmare
p3 − 17 ≤ 0, adică p ∈ {1, 2}. Dacă p = 1 se ajunge la contradicţia −16 = 0, iar
pentru p = 2 deducem că y 2 + 2y − 3 = 0, ecuaţie a cărei singură soluţie naturală este
y = 1. Rezultă că x = 3, deci soluţia ecuaţiei din enunţ este perechea (3, 1).
Soluţia 2 (Gheorghe Iurea). Rezolvăm ecuaţia ı̂n numere ı̂ntregi. Notăm d = x−y,
p = xy, cu d, p ∈ Z. Cum x3 − y 3 = (x − y)3 + 3xy(x − y) = d3 + 3dp, ecuaţia devine
16
17 − d3
=
− d2 − d − 1, deci d − 1 este
d3 + 3dp = 3p + 17. Rezultă că 3p =
d−1
d−1
62
divizor al lui 16. Analizând cazurile posibile, determinăm d şi p şi apoi aflăm soluţiile
ecuaţiei iniţiale: (x, y) ∈ {(3, 1); (−1, −3)}.
VIII.106. În tetraedul V ABC, avem AB = 4cm,
√ BC = 5cm, CA = 6cm, iar
15 7 2
cm . Calculaţi sinusurile unariile feţelor V AB, V BC şi V CA sunt egale cu
4
Õ
Õ
Õ
ghiurilor AV
B, BV
C şi CV
A.
Vlad Emanuel, student, Bucureşti
Soluţie. Calculând
aria
triunghiului
ABC
cu formula lui Heron, obţinem că
√
15 7 2
aceasta este
cm , prin urmare tetraedrul V ABC este echifacial. Rezultă că
4
Õ
V A = BC =√5cm, V B = CA = 6cm, iar √
V C = AB = 4cm, de unde sin AV
√ B =
2AV AB
7
2
·
A
5
7
2A
3
7
V
BC
V
CA
Õ
Õ
=
, sin BV
C=
=
, iar sin CV
A=
=
.
VA·VB
8
VB·VC
32
VC ·VA
16
VIII.107. Fie ABCD un tetraedru, iar m1 , m2 şi m3 lungimile bimedianelor sale.
Demonstraţi că 3(AB 2 + AC 2 + AD2 + BC 2 + CD2 + DB 2 ) ≥ 4(m1 + m2 + m3 )2 .
D.M. Bătineţu-Giurgiu, Bucureşti
Soluţie. Se ştie că ı̂n orice tetraedru ABCD are loc identitatea 4(m21 +m22 +m23 ) =
AB 2 +AC 2 +AD2 +BC 2 +CD2 +DB 2 (a se vedea, de exemplu, D. Brânzei, S. Aniţa,
C. Cocea - Planul şi spaţiul euclidian, Ed. Academiei, Bucureşti, 1986). Folosind
inegalitatea dintre media aritmetică şi cea pătratică, obţinem că 3(m21 + m22 + m23 ) ≥
(m1 + m2 + m3 )2 , de unde cerinţa problemei. Egalitatea se atinge atunci când m1 =
m2 = m3 .
VIII.108. Într-un reper cartezian xOy, se consideră punctele Aij (i, j), unde
1 ≤ i, j ≤ 5. Determinaţi numărul triunghiurilor care au ca vârfuri trei dintre punctele
date.
Gabriel Popa, Iaşi
25 · 24 · 23
= 2300
Soluţie. Cum avem 5 · 5 = 25 de puncte, putem considera
6
de mulţimi formate din câte trei puncte. Pentru a găsi numărul triunghiurilor, trebuie să eliminăm mulţimile formate din puncte coliniare. Punctele Ai1 , i = 1, 5,
5·4·3
= 10 mulţimi de câte trei puncte coliniare; aceeaşi situaţie are
generează
6
loc pe fiecare dintre cele cinci orizontale, cinci verticale, precum şi pe cele două
diagonale A11 A55 şi A15 A51 . Pe fiecare dintre direcţiile A12 A45 , A21 A54 , A14 A41 şi
A25 A52 avem câte 4 mulţimi de trei puncte coliniare, iar pe fiecare dintre direcţiile
A31 A53 , A13 A35 , A13 A31 , A53 A35 , A11 A53 , A12 A54 , A13 A55 , A13 A51 , A14 A52 , A15 A53 ,
A11 A35 , A21 A45 , A31 A55 , A31 A15 , A41 A25 şi A51 A35 , există câte o singură mulţime
formată din trei puncte coliniare. Astfel, numărul mulţimilor care trebuie eliminate
este 10 · 12 + 4 · 4 + 1 · 16 = 152. Rămân 2300 − 152 = 2148 de triunghiuri.
Clasa a IX-a
IX.96. Determinaţi triunghiurile ı̂n care tangentele unghiurilor se exprimă prin
numere naturale. (În legătură cu X.78 din RecMat 1/2007.)
Titu Zvonaru, Comăneşti
√
π
Soluţie. Fie A unghiul cel mai mic al triunghiului; atunci A ≤ , deci tg A ≤ 3
3
63
π
. Mai departe, din tg A + tg B +
4
tg C = tg A · tg B · tg C, obţinem că (tg B − 1)(tg C − 1) = 2, de unde B = arctg 2,
C = acrtg 3 (sau invers).
şi, cum tg A ∈ N, rezultă că tg A = 1, adică A =
2
2
IX.97. Demonstraţi
că ı̂n orice triunghi are loc inegalitatea ma hb hc + mb hc ha +
r
.
m2c ha hb ≥ 4S 2 2 +
2R
Cătălin Cristea, Craiova
2(b2 + c2 ) − a2
Soluţie. Observăm că m2a hb hc + m2b hc ha + m2c ha hb = 4S 2
+
4bc
2(a2 + c2 ) − b2
2(a2 + b2 ) − c2
+
. Prin aplicarea inegalităţii Cebı̂şev pentru şiruri
4ac
4ab
2(b2 + c2 ) − a2 2(a2 + c2 ) − b2 2(a2 + b2 ) − c2
+
+
≥
de monotonii contrare, deducem că
4ac
4ab

‹ 4bc
1
1
1
1
1
9
· 3(a2 + b2 + c2 ) ·
+
+
≥ , ultima relaţie obţinându-se cu ajutorul
3
4 bc ac bc
4
inegalităţii mediilor. Inegalitatea de demonstrat se deduce imediat, ţinându-se seama
că R ≥ 2r.

‹
1 2
IX.98. Aflaţi a, b, c ∈ R, a ̸= 0, pentru care |ax2 + bx + c| ≤ x −
, ∀x ∈ R.
a
Marian Ursărescu,
Roman
1
1
b
Soluţia I (Paul Georgescu). Pentru x = , obţinem că + + c ≤ 0, de
a
a a
b
1
unde c = − − . Substituind ı̂n inegalitatea din enunţ, obţinem că |ax2 + bx −
a‹

‹
‹

‹
‹
a

1
1 1
b
1 2
1
1 2
, deci a x −
x+
+b x−
. Notând
− | ≤ x−
≤
x
−
a
a
a
a
a
a a
1
x − = y, obţinem că |ay 2 + (b + 2)y| ≤ y 2 , ∀y ∈ R, de unde |ay + (b + 2)| ≤ |y|,
a ∗
∀y ∈ R . Rezultă că b + 2 = 0 şi |ay| ≤ |y|, ∀y ∈ R∗ , deci |a| ≤ 1. Urmează că
1
a ∈ [−1, 0) ∪ (0, 1], b = −2, c = .
a
1
1
b
Soluţia a II-a (a autorului). Pentru x = , obţinem că + + c ≤ 0, de
a
a a
‹

1 2
, deci
unde 1 + b + ac = 0. Conform ipotezei, avem că ax2 + bx + c ≤ x −
a
‹

2
1
(a − 1)x2 + b −
x + c − 2 ≤ 0, ∀x ∈ R. Acest fapt se petrece dacă şi numai dacă
a
a
a−1 ≤ 0 şi ∆ ≤ 0, adică atunci când a < 1 şi (ab+2)2 −4(a−1)(a2 c−1) ≤ 0. Înlocuind
b = −1 − ac, ultima condiţie conduce la a2 (1 + ac)2 − 4a(1 + ac) − 4a3 c + 4a + 4a2 c ≤ 0,
prin urmare a2 (1 − ac)2 ≤ 0, de unde ac = 1. Se observă uşor că, ı̂n ipoteza ac = 1,

‹
1 2
are loc inegalitatea ax2 + bx2 + c ≥ − x −
dacă şi numai dacă a ≥ −1. În
a
1
concluzie, a ∈ [−1, 0) ∪ (0, 1], b = −2 şi c = .
a
IX.99. Fie k ∈ [0, 1), n ∈ N∗ şi numerele αi ∈ R∗ , βi ∈ R, εi ∈ {−1, 1}, i = 1, n,
64
astfel ı̂ncât ε1 α1 + ε2 α2 + . . . + εn αn = 0. Rezolvaţi ecuaţia
|α1 x + β1 | + |α2 x + β2 | + . . . + |αn x + βn | = k|ε1 β1 + ε2 β2 + . . . + εn βn |.
Dumitru Mihalache
! şi Gabi Ghidoveanu, Bârlad
n
X
Soluţie. Cum
εi (αi x + βi ) = x
n
X
εi αi
+
n
X
ε i βi =
n
X
εi βi , rezultă că
i=1
i=1
i=1 i=1
n
n
n
n
X
X
X
X
εi (αi x + βi ) = εi βi , de unde
|αi x + βi | ≥ εi βi şi atunci
i=1
i=1
i=1
i=1
n
n
n
X
X
X
εi βi ̸= 0, obţinem k ≥ 1, prin urmare ecuaţia nu
εi βi . Dacă
ε i βi ≥ k·
i=1
n
X
i=1
are soluţii. Dacă
i=1
εi βi = 0, ecuaţia dată este echivalentă cu sistemul αi x + βi = 0,
i=1
i = 1, n. Acest sistem nu are soluţii dacă (α1 , a2 , . . . , αn ) şi (β1 , β2 , . . . , βn ) nu sunt
β1
β2
βn
proporţionale şi are soluţia x = −γ, unde γ =
=
= ... =
, ı̂n caz contrar.
α1
α2
αn
IX.100. Fie (an )n≥1 şi (bn )n≥1 două şiruri de numere reale, cu an ̸= 0, ∀n ≥ 1
şi 3 ·
n
X
(ak b2k − a2k bk ) =
k=1
n
X
!3
−
ak
k=1
n
X
a3k , ∀n ≥ 1. Demonstraţi că, pentru orice
k=1
n ≥ 1, există αn ∈ {0, 1} astfel ı̂ncât bn = αn (a1 +. . .+an )−(1−αn )(a1 +. . .+an−1 ).
Marian Tetiva, Bârlad
Soluţie. Pentru n = 1, relaţia din enunţ devine 3a1 b1 (b1 − a1 ) = 0, deci b1 = 0
(şi luăm α1 = 0) sau b1 = a1 (şi alegem α1 = 1). Pentru n ≥ 2, scădem din
relaţia din enunţ pe aceea obţinută din ea prin ı̂nlocuirea lui n cu n − 1; ajungem la
3(an b2n
an + 3
−
a2n bn )
n−1
X
n
X
=
!
ak
!3
ak
−
k=1
a2n
⇔ an
k=1
rezultă fie că bn =
bn −
n−1
X
k=1
n
X
!3
−
ak
ak
!
bn +
k=1
n
X
a3n
⇔
n−1
X
3(an b2n
−
a2n bn )
=3
!
ak
n−1
X
!2
ak
·
k=1
= 0. Deoarece an ̸= 0, de aici
k=1
ak (deci se poate lua αn = 1), fie că bn = −
k=1
n−1
X
ak (deci putem
k=1
alege αn = 0), ceea ce ı̂ncheie soluţia.
Autorul remarcă faptul că este adevărată şi reciproca afirmaţiei din eneunţ.
Clasa a X-a
X.96. Dacă a, b, c sunt numere reale pozitive cu suma 1, demonstraţi că ab · bc · ca
+ b · cb · ac ≤ 2(ab + bc + ca).
Dorin Mărghidanu, Craiova
Soluţie. Folosim inegalitatea dintre media aritmetică ponderată şi media geometrică ponderată: p1 a + p2 b + p3 c ≥ ap1 · bp2 · cp3 , ∀p1 , p2 , p3 > 0 cu p1 + p2 + p3 = 1
Considerând p1 = b, p2 = c şi p3 = a, rezultă că ba + cb + ac ≥ ab · bc · ca . Luând apoi
p1 = c, p2 = a şi p3 = b, obţinem că ca + bc + ab ≥ ba · cb · ac . Adunând aceste relaţii,
se obţine inegalitatea din enunţ.
a
65
X.97. Fie a, b, c ∈ C∗ numere complexe distincte astfel ı̂ncât (a − b)3 = (b − c)3 =
(c − a)3 . Arătaţi că |2a − b − c| = |2b − c − a| = |2c − a − b|.
Dan Nedeianu, Drobeta-Tr. Severin

‹

‹
b−c 3
c−a 3
Soluţie. Condiţia dată este echivalentă cu
=
= 1. Cum
a−b
a−b
a+c
a − b ̸= b − c (altfel b =
şi, folosind relaţia din enunţ, s-ar deduce că a = c),
2
b−c ̸= c−a şi c−a ̸= a−b, găsim că b−c = zε şi c−a = zε2 , unde ε este rădăcină cubică
√
a unităţii, iar z = a − b. Deducem că |2a − b − c| = |2b − c − a| = |2c − a − b| = |z| · 3.
X.98. Fie Ai (zi ), i = 1, 3 vârfurile unui triunghi din planul xOy şi P (z) un punct
din acest plan (zi şi z sunt afixele punctelor Ai , respectiv P ). Să se arate că P este
situat ı̂n interiorul triunghiului A1 A2 A3 sau pe una din laturile sale dacă şi numai
dacă există αi ≥ 0, i = 1, 3, astfel ı̂ncât α1 + α2 + α3 = 1 şi z = α1 z1 + α2 z2 + α3 z3 .
Adrian Corduneanu, Iaşi
Soluţie. Punctul P este situat ı̂n interiorul triunghiului A1 A2 A3 sau pe una
din laturile sale dacă şi numai dacă există M ∈ [A1 A2 ] cu P ∈ [M A3 ]. Deoarece
M ∈ [A1 A2 ] ⇔ ∃t ∈ [0, 1] astfel ı̂ncât zM = (1 − t)z1 + tz2 şi P ∈ [M A3 ] ⇔ ∃s ∈ [0, 1]
cu proprietatea z = (1 − s)z3 + szM , rezultă că z = (1 − t)sz1 + stz2 + (1 − s)z3 .
Considerând α1 = s(1 − t), α2 = st şi α3 = 1 − s, obţinem αi ≥ 0, α1 + α2 + α3 = 1
şi z = α1 z1 + α2 z2 + α3 z3 , ceea ce ı̂ncheie demonstraţia.
X.99. Considerăm triunghiurile echilaterale ABC şi A1 B1 C1 şi construim triunghiurile echilaterale AA1 A2 , BB1 B2 , CC1 C2 , AB1 A3 , BC1 B3 , CA1 A3 , AC1 A4 ,
BA1 B4 şi CB1 C4 ; toate triunghiurile citate sunt orientate pozitiv. Fie punctele
M2 ∈ A2 B, N2 ∈ B2 C, P2 ∈ C2 A, M3 ∈ A3 B, N3 ∈ B3 C, P3 ∈ C3 A, M4 ∈ A4 B,
M2 A2
N2 B2
P2 C2
M3 A3
N3 B3
N4 ∈ B4 C şi P4 ∈ C4 A astfel ı̂ncât
=
=
=
=
=
M2 B
N2 C
P2 A
M3 B
N3 C
P3 C3
M4 A4
N4 B4
P4 C4
=
=
=
. Demonstraţi că triunghiurile M2 N2 P2 , M3 N3 P3
P3 A
M4 B
N4 C
P4 A
şi M4 N4 P4 sunt echilaterale şi au acelaşi centru.
Cătălin Ţigăeru, Suceava
Soluţie. Notăm afixul fiecărui punct care apare, cu litera mică ce ı̂i corespunde.
Scriem condiţiile ca cele 11 triunghiuri care apar ı̂n ipoteză să fie echilaterale:
a + εb + ε2 c = 0;
a + εa1 + ε2 a2 = 0;
a + εb1 + ε2 a3 = 0,
a + εc1 + ε2 a4 = 0,
a1 + εb1 + ε2 c1 = 0;
b + εb1 + ε2 b2 = 0,
c + εc1 + ε2 c2 = 0;
2
b + εc1 + ε b3 = 0,
c + εa1 + ε2 a3 = 0,
2
b + εa1 + ε b4 = 0, c + εb1 + ε2 c4 = 0,
unde ε este rădăcina primitivă de ordin trei a unităţii. Demonstrăm că triunghiurile
Ai Bi Ci , i = 2, 3, 4, sunt echilaterale; trebuie verificate relaţiie ai + εbi + ε2 ci = 0,
i = 2, 3, 4. Vom da justificarea doar pentru i = 2 :
a2 + εb2 + ε2 c2 = −(εa + ε2 a1 ) − ε(εb + ε2 b1 ) − ε2 (εc + ε2 c1 ) =
= −ε(a + εb + ε2 c) − ε2 (a1 + εb1 + ε3 c1 ) = 0.
66
Fie k valoarea comună a rapoartelor egale din enunţ. Obţinem că mi =
1
ai +
1+k
k
1
k
1
k
b, ni =
bi +
c, pi =
ci +
a, i = 2, 3, 4. Atunci, mi + εni +
1+k
1+k
1+k
1+k
1+k
k
1
εpi =
· (εb+ε2 c+ε3 a) = 0, i = 2, 3, 4, prin urmare △Mi Ni Pi sunt echilaterale.
1+k ε
Notăm cu G şi Gi , i = 1, 4, centrele (de greutate) ale triunghiurilor ABC, respectiv
1
Ai Bi Ci , i = 1, 4. Observăm că gi = (ai + bi + ci ) = −(εg + ε2 g1 ), i = 2, 3, 4, deci
3
triunghiurile Ai Bi Ci , i = 2, 3, 4, au acelaşi centru, fie acesta G, de afix g = −εg−ε2 g1 .
Din relaţia g + εg + ε2 g1 = 0, deducem că G, G şi G1 formează un triunghi echilateral.
1
Notăm cu Gi centrele triunghiurilor Mi Ni Pi , i = 2, 3, 4; avem că g i = (mi +ni +pi ) =
3
1
k
g+
g, prin urmare △Mi Ni Pi , i=2, 3, 4, au acelaşi centru Gk , plasat pe
1+k
1+k
latura GG a triunghiului echilateral GGG1 , pe care o ı̂mparte ı̂n raportul k.
X.100. Demonstraţi că ı̂n orice triunghi ABC are loc inegalitatea
1
1
1
4
+
+
≥ .
3
sin2 A(sin B + sin C)2
sin2 B(sin C + sin A)2
sin2 C(sin A + sin B)2
Soluţie.
Marius Olteanu, Rm. Vâlcea
1
1
1
9
Este cunoscută inegalitatea
+
+
≥
·
2
2
2
(x + y)
(y + z)
(z + x)
4
1
, ∀x, y, z > 0 (a se vedea, de exemplu, Old and New Inequalities de
xy + yz + zx
T. Andreescu, G. Dospinescu, V. Cârtoaje, M. Lascu, apărută la GIL, Zalău, 2004,
pg. 22, ex. 114). Înlocuind x = sin A sin B, y = sin A sin C, z = sin B sin C, obţinem
X
1
1
9
X
că
. Pe de altă parte, avem
≥
4 sin A sin B sin C(
sin2 A(sin B + sin C)2
sin A)

‹
sin A + sin B + sin C 3
(inegalitatea mediilor), iar
3
√
sin A + sin B + sin C
A+B+C
3
≤ sin
=
(inegalitatea lui Jensen aplicată funcţiei
3
3
2
sinus pe [0, π]). Înlocuind, rezultă concluzia problemei.
că sin A sin B sin C ≤
Clasa a XI-a
XI.96. Fie ε rădăcina primitivă de ordin trei a unităţii, iar A, B ∈ M3 (R) cu
det(A + εB) = 0. Demonstraţi că det(A − B) = det A − det B.
Dan Popescu, Suceava
Soluţie. Considerăm polinomul f ∈ R[X], f (X) = det (A + XB) = det A + αX +
βX 2 + (det B) · X 3 . Cum f (ε) = 0, rezultă că det A + αε + β(−ε − 1) + det B = 0, de
unde α = β = det A + det B. Calculând f (−1) prin cele două modalităţi de scriere
ale lui f , obţinem că f (−1) = det (A − B) = detA − detB.
XI.97. Fie n ≥ 3 un număr natural. Arătaţi că pentru orice k ∈ {2, 3, . . . , n − 1},
există A ∈ Mn ({0, 1}) astfel ı̂ncât Ap ̸= In , ∀p ∈ {1, 2, . . . , k − 1} şi Ak = In .
Gheorghe Iurea, Iaşi
67
ˆ
’
0 1 0 ... 0
0 0 1 ... 0
..............
Soluţie. Considerăm B =
∈ Mk ({0, 1}). Se constată
0 0 0 ... 1
1 0 0 ... 0
că, pentru p ∈ {1, 2, . . . , k − 1}, avem că B p = (bij ), unde b1,p+1 = b2,p+2 = . . . =
bk−p,p = 1, bk−p+1,1 = bk−p+2,2 = . . . = bk,p = 1, iar bij = 0 ı̂n rest. În plus, B k = Ik .
‹

B
0
verifică cerinţele problemei.
Atunci, matricea A =
0 In−k
É
π
1 − cos x
→ R, f (x) = ln
este
2
1 + cos x
concavă şi, folosind eventual acest lucru, arătaţi că ı̂n orice triunghi ascuţitunghic
1 − cos A 1 − cos B 1 − cos C
1
ABC are loc inegalitatea
·
·
≤
.
1 + cos A 1 + cos B 1 + cos C
27
Bogdan Victor Grigoriu, Fălticeni
cos x
Soluţie. Funcţia f este de două ori derivabilă, iar f ′′ (x) = − 2 < 0, ∀x ∈
sin x
π
0,
, prin urmare f este concavă. Aplicând inegalitatea lui Jensen, obţinem că
‹
2
1
A+B+C
≥ [f (A) + f (B) + f (C)], deci
f
3
3
Ê
‹

1 − cos π3
1
1 − cos A 1 − cos B 1 − cos C
ln
,
ln
·
·
π ≥
1 + cos 3
3
1 + cos A 1 + cos B 1 + cos C
rezultă imediat din monotonia funcţiei logaritmice.
XI.98. Demonstraţi că funcţia f :
0,
Nota autorului. În aceeaşi manieră se poate demonstra că, ı̂n orice triunghi
1 − sin A 1 − sin B 1 − sin C
1
√ .
ABC, are loc inegalitatea
·
·
≤
1 + sin A 1 + sin B 1 + sin C
(2 + 3)6
XI.99. Studiaţi convergenţa şirului (vn )n≥1 definit prin vn+1 =
(vnc + d)1/c
,
vn
∀n ≥ 1, unde v1 , c şi d sunt numere reale pozitive date.
Gheorghe Costovici şi Adrian Corduneanu, Iaşi
√
1/c
1 + 1 + 4d
. Dacă
Soluţie. Vom demonstra că lim vn = l, unde l =
n→∞
2
v1 = l, se demonstrează prin inducţie matematică faptul că vn = l, ∀n ≥ 1. În cazul
ı̂n care v1 ∈ (0, l), se arată (tot prin inducţie) că v2n−1 ∈ (0, l) şi v2n ∈ (l, +∞),
∀n ∈ N∗ , apoi că subşirul (v2n−1 )n≥1 este strict crescător, ı̂n timp ce (v2n )n≥1 este
strict descrescător. Urmează că există şi sunt finite α = lim v2n−1 , β = lim v2n şi,
n→∞
n→∞
(β c + d)1/c
prin trecere la limită ı̂n relaţiile de recurenţă, obţinem că α =
, iar β =
β

‹
1/c
αc + d
(αc + d)1/c
αc + d + dαc
(αc + d)1/c
. De aici, α =
+d
:
, deci αc =
,
c
α
α
α
αc + d
prin urmare α2c − αc − d = 0, de unde găsim că α = l. Asemănător se arată că β = l.
În sfârşit, analog se tratează cazul ı̂n care v1 ∈ (l, +∞).
68
Notă. De fapt, şirul un = vnc , ∀n ∈ N∗ , verifică relaţia de recurenţă un+1 =
un + d
, ∀n ∈ N∗ , recurenţă omografică care se studiază ı̂n mod uzual.
un
XI.100. Demonstraţi că

(x + 1) sin
π
π
− cos
x+1
x+1
‹
< x sin
π
π
− cos
, ∀x ∈ [2, ∞).
x
x
Petru Răducanu, Iaşi
Soluţie. Inegalitatea de demonstrat este echivalentă cu f (x+1)−f (x) < g(x+1)−
π
π
g(x), x ∈ [2, ∞), unde f, g : [2, +∞) → R, f (t) = t sin , g(t) = t cos , ∀t ∈ [2, ∞).
t
t
Aplicând teorema lui Lagrange funcţiei f pe intervalul
π [x, x + 1], x ≥ 2, obţinem
c ∈ (x, x + 1) pentru care f (x + 1) − f (x) = f ′ (c) = h
, unde h(t) = sin t − t cos t,
c
πi
t ∈ 0, . Observăm că h′ (t) = t sin t > 0, deci h este crescătoare, astfel că h(t) ≤
πi
π
π 2
π
h
= 1, ∀t ∈ 0, . Deducem că f (x + 1) − f (x) < 1, ı̂ntrucât
∈ 0,
.
2
2
c
2
Analog se demonstrează că g(x + 1) − g(x) > 1, ∀x ≥ 2, ceea ce ı̂ncheie rezolvarea.
Clasa a XII-a

‹
1 2 3 4 5
.
5 3 4 1 2
Liviu Smarandache
şi Ionuţ Ivănescu, Craiova

‹
1 2 3 4 5
Soluţie. Notăm σ =
şi observăm că ordσ = 5. Din x11 = σ
5 3 4 1 2
rezultă că x55 = e, prin urmare ordx|55, deci ordx ∈ {1, 5, 11, 55}. Pe de altă parte,
cum ordS5 = 120, atunci ordx|120 şi rămâne că ordx ∈ {1, 5}. Dacă ordx = 1, ar
rezulta că x = e şi se ajunge la contradicţie e = e11 = x11 = σ. Dacă ordx = 5,
obţinem că σ = x11 = x · x5 · x5 = x, adică singura soluţie a ecuaţiei date este x = σ.
m
X
ak
XII.97. Fie ak ∈ R, k = 0, n, iar m ∈ (0, ∞) astfel ı̂ncât
= 0. Să se
m+k
k=0
arate că ecuaţia a0 + a1 x + . . . + an xn = 0 admite soluţie ı̂n intervalul (0, 1).
Mihail Bencze, Braşov
Soluţie. Aplicăm teorema de medie funcţiei f : [0, 1] → R, f (x) = (a0 + a1 x +
Z 1
n
X
ak
n
m−1
. . . + an x ) · x
, pentru care
f (x)dx =
= 0.
m
+k
0
XII.96. Rezolvaţi ı̂n S5 ecuaţia x11 =
k=0
XII.98. Determinaţi primitivele funcţiei f : (0, π) → R, f (x)=
n ∈ N∗ .
Z
Soluţie. Fie I =
x ∈ (0, π). Observăm că
I.V. Maftei, Bucureşti şi Mihai Haivas, Iaşi
Z
sin3n−1 x · cosn−1 x
cos3n−1 x · sinn−1 x
dx,
J
=
dx, unde
4n
4n
sin x + cos x
sin4n x + cos4n x
Z
I +J =
sin3n−1 x · cosn−1 x
,
sin4n x + cos4n x
sinn−1 x cosn−1 x(sin2n x + cos2n x)
dx =
sin4n x + cos4n x
69
Z tgn−1 x ·
=
1
1
Z
+ ctgn−1 x ·
2
(tgn x − ctgn x)′
cos x
sin2 x dx = 1
dx =
n
(tgn x − ctgn x)2 + 2
tg2n x + ctg2n x
tgn x − ctgn x
1
√
= √ arctg
+ C.
n 2
2
√ n
n
1
tg x + ctg x − 2 √ ln √ + C. Adunând membru
Analog se obţine că I − J =
2n 2 tgn x + ctgn x + 2 cu membru cele două relaţii, găsim valoarea lui I.
XII.99. Se consideră funcţia f : (0, ∞) → (0, 1) continuă şi descrescătoare

‹ şi
an+1
şirul strict crescător (an )n≥1 de numere reale pozitive, astfel ı̂ncât şirul
an n≥1
Z an+1
1
este strict descrescător. Definim In =
f (x)dx, ∀n ∈ N∗ .
an an
a) Demonstraţi că (In )n≥1 este un şir descrescător.
an+1
b) Dacă lim
= 1, calculaţi lim In .
n→∞ an
n→∞
Cosmin Manea şi Dragoş Petrică, Piteşti
Soluţie. Din teorema de medie,
pentru

‹fiecare n ∈ N, găsim cn ∈ (an , an+1 ) astfel
1
an+1
că In =
(an+1 − an )f (cn ) =
− 1 f (cn ).
an
an
a) Cum cn < an+1 < cn+1
 iar f este
‹ descrescătoare, urmează că (f (cn ))n≥1 este
an+1
descrescător. De asemenea,
−1
este descrescător, de unde (In )n≥1 , este
an
n≥1
descrescător, ca produs de două şiruri descrescătoare strict pozitive.
b) Cum (f (cn ))n≥1 , este descrescător şi mărginit, el admite o limită finită l.
Urmează că lim In = 0 · l = 0.
n→∞
XII.100. În raport cu reperul xOy, considerăm punctele A(a, 0), B(0, b) şi T ∈
(AB), unde a > 0, b > 0. Determinaţi parabola y = λx2 + µ care este tangentă ı̂n T
la AB, ştiind că aria suprafeţei determinată de parabolă şi axele de coordonate este
maximă.
Adrian Corduneanu, Iaşi
b
Soluţie. Ecuaţia dreptei AB este y = (a − x), deci T are coordonatele x0 ∈
a
b
b
b
(0, a), y0 = (a − x0 ). Din condiţiile de tangenţă λx20 + µ = (a − x0 ) şi 2λx0 = − ,
a
a
a
b
b(2a − x0 )
b
b(2a − x0 )
rezultă λ = −
şi µ =
. Parabola de ecuaţie y = −
x2 +
2ax0
2a
2ax0
2a
È
va tăia axa Ox ı̂n punctul M (xM , 0), unde xM = 2ax0 − x20 . Aria cerută este
Z xM
b È
λ
S =
x0 (2a − x0 )3 . Se observă că S este
(λx2 + µ)dx = x3M + µxM =
3
3a
0
b
3b
a
maximă pentru x0 = , deci parabola cătată are ecuaţia y = − 2 · x2 + .
2
a
4
70
Soluţiile problemelor pentru pregătirea
concursurilor propuse ı̂n nr. 1/2009
A. Nivel gimnazial
G156.
√
Dacă a, b, c ∈ R∗+ ,
c2 + 1
b2 + 1
+√
≥ 6.
b2 − b + 1
c2 − c + 1
1
1
1
a2 + 1
+ +
≤ 3, demonstraţi că √
+
a
b
c
a2 − a + 1
I.V. Maftei, Bucureşti şi Mihai Haivas, Iaşi
√
a2 + 1
a
Soluţia I (a autorilor). Observăm că √
= a2 − a + 1+ √
≥
2−a+1
2−a+1
a
a
√
2 a, ∀a ∈ R∗+ şi analog pentru celelalte două fracţii ale sumei din membrul stâng.
√
√
√
Rezultă că această sumă este cel puţin egală cu 2( a + b + c). Pe de altă parte,
√
2a
a≥
(inegalitatea M G ≥ M H, aplicată numerelor a şi 1), deci
1+a
√
√
√
2( a + b + c) ≥ 4
≥ 36 ·
1
1+a
a
+
1+b
b
+
1+c
c

=
a
b
c
+
+
1+a 1+b 1+c
3+
1
a
36
+ 1b +
1
c
≥
‹
≥
36
= 6,
3+3
de unde inegalitatea din enunţ.
Soluţia a II-a (Oana Adăscăliţei şi Florina Toma, eleve,
Iaşi). Vom
È
2
face aceeaşi demonstraţie ı̂n doi paşi, cu alte argumente. Cum a(a − a + 1) ≤
a + a2 − a + 1
a2 + 1
1
2
a2 + 1
=
, atunci È
≥ 2
, deci √
≥
2
2
a +1
a2 − a + 1
a(a2 − a + 1)
√
2 a şi ı̂ncă două inegalităţi similare. Apoi, din inegalitatea C-B-S, obţinem că

‹
‹

1 2
1
1
1
1
1
1 1 1
√ +√ +√
(1 + 1 + 1), de unde √ + √ + √ ≤ 3.
≤
+ +
a
b
c
a
c
a
c
b
b
‹
√
√
√
√
√
√
1
1
1
Însă √ + √ + √ ( a + b + c) ≥ 9, prin urmare a + b + c ≥ 3 şi astfel
a
c
b
rezultă inegalitatea din enunţ.
G157. Spunem că un număr natural are proprietatea (P) dacă se poate scrie ca
sumă a trei pătrate perfecte nenule şi că are proprietatea (Q) dacă se poate scrie ca
sumă a patru pătrate perfecte nenule.
a) Daţi exemple de numere naturale care au: numai proprietatea (P ); numai
proprietatea (Q); atât proprietatea (P ) cât şi proprietatea (Q).
b) Dacă a, b, c ∈ N∗ au suma pară şi oricare dintre ele este diferit de suma celorlaltor două, demonstraţi că numărul a2 + b2 + c2 are proprietatea (Q).
Ovidiu Pop, Satu Mare
Soluţie. a) 6 = 12 + 12 + 22 are numai proprietatea (P ), 7 = 12 + 12 + 12 + 22 are
numai proprietatea (Q), iar 30 = 12 + 22 + 52 = 12 + 22 + 32 + 42 are şi proprietatea
(P ), şi proprietatea (Q).
71
b) Cum suma şi diferenţa au aceeaşi paritate, concluzia problemei rezultă din iden
‹

‹

‹

‹
a+b+c 2
−a + b + c 2
a−b+c 2
a+b−c 2
+
+
+
.
titatea a2 + b2 + c2 =
2
2
2
2
G158. Se consideră ecuaţia x2 +y 2 +z 2 = (x−y)2 +(y −z)2 +(z −x)2 , x, y, z ∈ N.
a) Arătaţi că ecuaţia are o infinitate de soluţii.
b) Dacă (x, y, z) este soluţie a ecuaţiei, demonstraţi că fiecare dintre numerele
xy, yz, zx şi xy + yz + zx este pătrat perfect.
Liviu Smarandache, Craiova
Soluţie. a) De exemplu, putem considera x = 1, y = a2 , z = (a + 1)2 , cu a ∈ N.
b) Din relaţia din enunţ, obţinem că (x+y +z)2 = 4(xy +yz +zx), de unde rezultă
că xy + yz + zx este pătrat perfect. Apoi, tot din relaţia din enunţ, rezultă succesiv
x2 +y 2 +z 2 = 2(xy+yz+zx) ⇔ x2 −2x(y+z)+(y+z)2 −4yz = 0 ⇔ (x−y−z)2 = 4yz,
deci yz este pătrat perfect. La fel se arată că xy şi zx sunt pătrate perfecte.
G159. Aflaţi ultimele două cifre ale numerelor (70n + 6) · 6n−1 , n ∈ N.
Ion Săcăleanu, Hârlău
Soluţie (Gheorghe Iurea). Notăm an = (70n + 6) · 6n−1 , n ∈ N. Deoarece
an+1 − an = 50(7n + 9) · 6n−1 = M100 , n ∈ N∗ , rezultă că toate numerele an au
aceleaşi ultime două cifre. Cum a1 = 76, deducem că toate numerele considerate se
termină ı̂n 76.
§
a
b
G160. Se consideră mulţimile A = {1, 2, 3, . . . , 2009}, B =
+
+
a+
b+d
d
ª
§
c d
d d
+
+
a, b, c, d ∈ A, a, b, c, d distincte şi C =
a, b, c, d ∈ A,
c+d
a + d b + d c + d
ª
a, b, c, d distincte . Determinaţi A ∩ B ∩ C. (În legătură cu E: 13650 din G.M. 56/2008.)
Andrei Crăcană, elev, Iaşi
a
b
c
Soluţie. Se constată uşor că, dacă x =
+
+
∈ B, atunci 3−x ∈ C.
a+d b+d c+d
În cazul ı̂n care x ∈ A∩B ∩C, obţinem că x ∈ {1, 2}. Vom arăta că {1, 2} ⊂ A∩B ∩C
şi astfel va rezulta că A ∩ B ∩ C = {1, 2}. Pentru (a, b, c, d) = (14, 21, 30, 42), obţinem
că x = 1, deci 1 ∈ B şi 3 − 1 = 2 ∈ C. Pentru (a, b, c, d) = (75, 375, 875, 125), avem
că x = 2, prin urmare 2 ∈ B şi 3 − 2 = 1 ∈ C şi astfel rezolvarea problemei este
ı̂ncheiată.
G161. Fie M mulţimea numerelor de forma abc, cu a · b · c ̸= 0. Determinaţi
cardinalul maxim al unei submulţimi N a lui M astfel ı̂ncât x + y ̸= 1109, ∀x, y ∈ N.
Petru Asaftei şi Gabriel Popa, Iaşi
Soluţie. Numărul cerut este 405. Cum fiecare dintre cifrele a, b, c este nenulă,
cardinalul lui M este 9 · 9 · 9 = 729. Dintre elementele lui M , 9 · 9 · 1 = 81 se termină
ı̂n 9. Grupăm celelalte elemente (ı̂n număr de 729 − 81 = 648) ı̂n perechi de forma
(x, 1109 − x), obţinând 324 de perechi. Dacă se consideră o submulţime N a lui M
cu cel puţin 406 elemente, cum 324 + 81 = 405, din principiul cutiei rezultă că măcar
două dintre elementele lui N aparţin unei aceleiaşi perechi (x, 1109 − x), deci au suma
1109. Luând câte un element din fiecare dintre perechile considerate, precum şi toate
72
numerele din M care se termină ı̂n 9, obţinem o submulţime a lui M de cardinal 405,
care verifică proprietatea din enunţ.
G162. Putem ı̂nlocui un triplet de numere ı̂ntregi (a, b, c) cu unul dintre tripletele
(2b + 2c − a, b, c), (a, 2a + 2c − b, c) sau (a, b, 2a + 2b − c). Arătaţi că dacă pornim
de la tripletul (31329, 24025, 110224) şi efectuăm succesiv asemenea ı̂nlocuiri, se obţin
mereu triplete formate numai din pătrate perfecte.
Marian Tetiva, Bârlad
Soluţie. Dacă pornim de la un triplet de forma (x2 , y 2 , (x + y)2 ) şi efectuăm
oricare dintre cele trei transformări, obţinem tot triplete de forma (m2 , n2 , (m + n)2 ).
Într-adevăr, dacă ı̂nlocuim x2 cu 2y 2 + 2(x + y)2 − x2 = (x + 2y)2 , atunci m = x + 2y,
n = −y; dacă ı̂nlocuim y 2 cu 2x2 +2(x+y)2 −y 2 = (2x+y)2 , putem considera m = −x,
n = 2x+y; ı̂n sfârşit, dacă ı̂nlocuim (x+y)2 cu 2x2 +2y 2 −(x+y)2 = (x−y)2 , vom lua
m = x, n = −y. Observăm acum că tripletul iniţial este de forma (x2 , y 2 , (x + y)2 ),
unde x = 177, y = 155 şi de aici urmează concluzia problemei.
b ̸= 90◦ şi punctele B1 ∈ (AC) şi C1 ∈ (AB).
G163. Fie ABC un triunghi cu m(A)
Arătaţi că axa radicală a cercurilor de diametre [BB1 ] şi [CC1 ] trece prin punctul A
dacă şi numai dacă B1 C1 ∥BC.
Neculai Roman, Mirceşti (Iaşi)
Soluţie. Fie M al doilea punct de intersecţie dintre cercul de diametru [BB1 ]
×
A
şi dreapta AB, iar N proiecţia lui C pe AB. Unghiul BM
B1
fiind ı̂nscris ı̂ntr-un semicerc, avem că B1 M ⊥ AB, de unde
M
AB1
AM
B1
C1
=
. Atunci: A se află pe axa radicală
M B1 ∥N C, deci
AN
AC
AM
AC1
a celor două cercuri ⇔ AM ·AB = AC1 ·AN ⇔
=
⇔
N
AN
AB
AC1
AB1
=
⇔ B1 C1 ∥BC şi astfel rezolvarea problemei este
C
B
AC
AB
completă.
G164. Fie B, b numere reale date, cu B > b > 0. Dintre toate trapezele circumscriptibile care au lungimile bazelor B şi b, determinaţi-l pe cel de arie maximă.
Claudiu Ştefan Popa, Iaşi
Soluţie. Fie ABCD trapez circumscriptibil, cu bazele AB = B, CD = b; notăm
x = BC, y = AD. Vom avea că B + b = x + y. Construim
D
C
CF ⊥ AB, CE∥AD, cu E, F ∈ AB. Calculăm h = CF exprimând ı̂n două moduri aria triunghiului BCE. Observăm
că CE = y, BE = B − b, iar semiperimetrul △BCE este
1
1
[x + y + (B − b)] = [(B + b) + (B − b)] = B. Din for2
2 È
mula lui Heron, ABCE = B(B − x)(B − y)(B − (B − b)) = A
E FB
È
Bb[B 2 − B(x + y) + xy] =
altă parte, ABCE
Cum AABCD =
È
Bb[B 2 − B(B + b) + xy] =
È
Bb(xy − Bb). Pe de
È
2 Bb(xy − Bb)
1
1
= BE · h = (B − b) · h, prin urmare h =
.
2
2
B−b
1
h(B + b), iar lungimile B şi b sunt date, atunci aria trapezului este
2
73
maximă când produsul xy este maxim. Suma x + y fiind constantă (este egală cu
B + b), produsul xy va fi maxim când x √
= y, deci ı̂n cazul trapezului isoscel. Prin
calcul direct, ı̂nălţimea acestuia este h = Bb.
G165. Fie ABC un triunghi isoscel (AB = AC), M mijlocul laturii [BC], iar P
un punct ı̂n interiorul triunghiului ABM . Notăm {D} = BP ∩ AC, {E} = CP ∩ AB.
Demonstraţi că BE < CD şi P E < P D.
Cristian Pravăţ, Iaşi şi Titu Zvonaru, Comăneşti
BF
CD
AE
Soluţie. Fie {F } = AP ∩ BC şi x =
, y =
, z =
. Atunci, din
FC
DA
EB
A
teorema lui Ceva, xyz = 1, iar x < 1 deoarece P ∈ IntABM .
1
y
Avem că BE < CD ⇔ AB ·
< AC ·
⇔ y+1 <
z+1
y+1
D
D
yz + y ⇔ 1 < yz şi, cum ultima inegalitate este adevărată,
′
rezultă prima parte a concluziei. Fie D simetricul lui D faţă
E
de AM ; trapezul BCDD′ este isoscel, deci inscriptibil, iar
P
E se va afla ı̂n interiorul cercului circumscris. Deducem că
′ C) < m(DEC),
Ö
Õ
Ö
m(DD
de unde m(PÕ
DE) < m(P
DD′ ) =
B F
C
M
′ C) < m(DEP
Ö
Õ ), prin urmare P E < P D.
m(DD
B. Nivel liceal
L156. Fie M un punct exterior cercului C de centru O şi rază R. Notăm cu
T1 , T2 punctele de contact cu cercul ale tangentelor duse din M la C şi cu A punctul
de intersecţie a dreptei OM cu cercul C, astfel ı̂ncât A ∈
/ [OM ]. Determinaţi punctele
M cu proprietatea că se poate construi un triunghi cu segmentele [M T1 ], [M T2 ] şi
[M O], dar nu se poate construi un triunghi cu [M T1 ], [M T2 ] şi [M A].
Temistocle Bı̂rsan, Iaşi
Soluţie. A se vedea Recreaţii Matematice 1/2009, pg. 41.
L157. În planul △ABC definim transformarea P → P ′ astfel: 1. punctul P se
proiectează pe dreptele BC, CA, AB ı̂n D, E şi respectiv F ; 2. simetricele punctelor
D, E, F ı̂n raport cu mijloacelor laturilor [BC], [CA] şi respectiv [AB] se notează
D′ , E ′ , F ′ ; 3. P ′ este punctul de concurenţă a perpendicularelor ı̂n D′ , E ′ , F ′ pe
BC, CA şi respectiv AB. Arătaţi că transformarea P → P ′ coincide cu simetria
ı̂n raport cu O, centrul cercului circumscris △ABC.
Temistocle Bı̂rsan, Iaşi
Soluţie (Daniel Văcaru, Piteşti). Cum avem de-a face cu două izometrii, este
suficient să demonstrăm că transformatele punctelor A, B, C sunt aceleaşi. Să vedem
cine sunt D, E, F când P = A; avem că D = A′ , E = A şi F = A, unde A′ este
proiecţia lui A pe BC. Observăm apoi că D′ este simetricul lui A′ faţă de mijlocul
lui [BC], E ′ coincide cu C şi F ′ cu B. Construind perpendicularele ı̂n B pe AB şi ı̂n
C pe AC, se obţine patrulaterul inscriptibil ABP ′ C. Deducem că P ′ coincide cu A′ ,
simetricul lui A faţă de O, centrul cercului circumscris. Raţionamentul este analog ı̂n
cazul ı̂n care P = B, respectiv P = C şi astfel se ı̂nchide demonstraţia.
Notă. Această problemă este, până la diferenţă de formulare, tocmai Propoziţia
3 din articolul Simetria faţă de centrul cercului cricumscris unui triunghi de Bogdan
Ioniţă şi Titu Zvonaru (G.M. - 3/1997, p. 98). Faptul ne este adus la cunoştinţă de
74
dl. Titu Zvonaru. Autorul problemei regretă şi ı̂şi cere scuze pentru acest incident.
(Menţionăm ı̂ncă o soluţie diferită de cea din articol: Perpendicularele ı̂n D şi D′
pe BC determină ı̂n cercul C(O, OP ) un dreptunghi, deci perpendiculara ı̂n D′ trece
prin simetricul fată de O al punctului P etc.).
L158. În interiorul triunghiului ABC cu latura [BC] fixă şi vârful A mobil, conÕ
Õ
Õ
siderăm punctul T asfel ı̂ncât AT
B ≡ BT
C ≡ CT
A. Determinaţi poziţia punctului
Õ = α < 5π , iar suma distanţelor de la
A ı̂n planul triunghiului pentru care m(BAC)
6
T la vârfurile triunghiului este maximă.
Cătălin Calistru, Iaşi
Soluţie. Remarcăm faptul că T ese tocmai punctul lui Toricelli asociat triunA
ghiului ABC. Astfel, dacă △P AB este echilateral, conP
struit ı̂n exteriorul △ABC, atunci punctele P, T şi C sunt
coliniare, iar T A + T B + T C = CP (vezi, de exemplu,
T
L. Niculescu şi V. Boskoff - Probleme practice de geomeC
trie, Ed. Tehnică, 1990). Folosind teorema cosinusului
B
ı̂n triunghiurile AP C şi ABC, obţinem că
π
CP 2 = AP 2 + AC 2 − 2AP · AC · cos A +
=
3
π
=
= BC 2 + 2AB · AC · cos A − 2AB · AC · cos A +
3
π
= BC 2 + 2AB · AC cos A − cos A +
=
3
sin(A + π6 )
π
π
sin = BC 2 + BC ·
· ha .
= BC 2 + 4AB · AC · sin A +
6
6
sin A
π
5π
Cum BC este constantă, iar sin A +
> 0 (deoarece α <
), deducem că CP
6
6
este maxim atunci când ha este maxim. Însă A se mişcă pe un arc capabil de unghiul
α, prin urmare poziţiile căutate ale punctului A sunt date de intersecţiile mediatoarei
segmentului [BC] cu arcele capabile de unghiul α, construite pe [BC].
π
L159. Dacă a, b, c ∈ R∗+ şi x ∈ 0,
, demonstraţi inegalitatea
2

‹

‹

‹

‹
√
√
sin x 3
sin x 2
sin x
tg x
3
3
a
+b
+c
+ 3 abc
> 6 · abc.
x
x
x
x
D.M. Bătineţu-Giurgiu, Bucureşti
Soluţie. Din inegalitatea mediilor, rezultă că

sin x
a
x
‹3

sin x
+b
x
‹2

sin x
+c
x
Ê
‹
≥3
3
abc ·

sin x
x
‹6

‹
√
sin x 2
3
= 3 abc
.
x

Pentru a obţine inegalitatea din enunţ, ar fi suficient să demonstrăm că
π
sin x
x
‹2
+
tg x
> 2, ∀x ∈ 0,
. Această inegalitate, atribuită lui Wilker, poate fi găsită ı̂n
x
2
G.M. 1/2007, pg. 1.
75
Notă. Soluţie corectă, bazată pe considerente de analiză matematică, s-a primit
de la Dl. Daniel Văcaru, Piteşti.
L160. Demonstraţi că ı̂n orice triunghi are loc inegalitatea

ma
mb
mc
ma + mb + mc ≥ 6r
+
+
mb + mc
ma + mc
ma + mb
‹
≥ 9r.
Marius Olteanu, Rm. Vâlcea
1
1
1
1
Soluţie. Avem că =
+
+
şi ma ≥ ha , mb ≥ hb , mc ≥ hc . În plus, ı̂n
r
ha
hb
hc
orice triunghi cu laturile a, b, c, are loc inegalitatea

(a + b + c)
1 1 1
+ +
a b
c
‹

≥6
b
c
a
+
+
b+c c+a a+b
‹
(a se vedea, de exemplu, Algebraic Inequalities de V. Cârtoaje, apărută la GIL, Zalău,
2006, problema 70, pg. 379). Cum medianele unui triunghi pot fi laturi ale unui alt
triunghi, obţinem că
1
(ma + mb + mc ) =
r

≥
1
1
1
+
+
ma
mb
mc
‹

1
1
1
+
+
ha
hb
hc
‹
(ma + mb + mc ) ≥

mb
mc
ma
(ma + mb + mc ) ≥ 6
+
+
mb + mc
ma + mc
ma + mb
‹
,
adică tocmai prima inegalitate cerută. Pentru a doua inegalitate, folosim binecunosx
y
z
3
cuta
+
+
≥ , ∀x, y, z ∈ R∗+ (Nesbitt).
y+z
z+x x+y
2
L161. Dacă a, b, c ∈ R∗+ şi a + b + c = 1, demonstraţi inegalitatea
3+
X (a − b)2 + (a − c)2
1+a
≤ 4(a2 + b2 + c2 )
X
‹
1
.
1+a
Titu Zvonaru, Comăneşti
Soluţie. Notând Q = a2 + b2 + c2 , avem că
4
3a
4a2 + 4b2 + 4c2 − 3a(2a + b + c)
− 2
=
=
1 + a a + b2 + c2
(1 + a)(a2 + b2 + c2 )
=
(b − a)(4b + a) (c − a)(4c + a)
+
Q(1 + a)
Q(1 + a)
şi ı̂ncă două identităţi similare. Pe de altă parte,
(b − a)(4b + a) (a − b)(4a + b)
a − b 4a2 − 4b2 + 3a − 3b
+
=
·
=
Q(1 + a)
Q(1 + b)
Q
(1 + a)(1 + b)
=
(a − b)2
4a + 4b + 3
(a − b)2 1 + a + 1 + b
(a − b)2
(a − b)2
·
≥
·
=
+
.
Q
(1 + a)(1 + b)
Q
(1 + a)(1 + b)
Q(1 + a) Q(1 + b)
76
Analog se obţin ı̂ncă două minorări şi deducem că
4
4
4
3(a + b + c)
+
+
−
≥
1 + a 1 + b 1 + c a2 + b2 + c2
≥
(a − b)2 + (a − c)2
(b − c)2 + (b − a)2
(c − a)2 + (c − b)2
+
+
,
(1 + a)Q
(1 + b)Q
(1 + c)Q
inegalitate echivalentă cu cea din enunţ. Egalitatea se atinge pentru a = b = c =
hxi
•
˜
1
.
3
[x]
.
n
n
Dumitru Mihalache şi Gabi Ghidoveanu, Bârlad
Soluţie. Dacă n ∈ N∗ , mulţimea soluţiilor ecuaţiei este R, conform unei cunoscute
proprietăţi a părţii ı̂ntregi. Într-adevăr,
L162. Dacă n ∈ Z∗ este fixat, rezolvaţi ı̂n R ecuaţia
hxi
n
=
•
= k ⇔ kn ≤ x < (k + 1)n ⇔ kn ≤ [x] < (k + 1)n ⇔
˜
[x]
= k.
n
În cazul ı̂n care n ∈ Z\N∗ , există şi sunt unice numerele q ∈ Z şi r ∈ R, 0 ≤ r < |n|,
astfel ca x = nq + r. Urmează că
hxi
•
n
˜
=
•
h nq + r i
n
˜
h
= q+
•
hri
ri
=q+
,
n
n
˜
•
˜
[nq + r]
nq + [r]
[r]
[x]
=
=
=q+
.
n
n
n
n
˜
•
hri
hri
[r]
Dacă r = 0, atunci
= 0. Dacă r ∈ (0, 1), atunci
=
= −1, deoarece
n
n
˜

‹
•n ˜
r |r|
r
1
1
[r]
= 0. Dacă r ∈ [1, |n|), atunci ,
, deci
∈ − , 0 , iar
∈ −1, −
n
|n|
n
n n
|n|
h r i • [r] ˜
=
= −1. De aici se obţine că, pentru n ∈ Z\N∗ , mulţimea soluţiilor ecuaţiei
n
n
S
din enunţ este R\
(nq, nq + 1).
q∈Z
L163. Fie a un număr ı̂ntreg impar, iar n ∈ N∗ . Arătaţi că polinomul X 2 + a2
este ireductibil ı̂n Z[X] ı̂nsă, pentru orice număr prim p, polinomul redus modulo p
este reductibil ı̂n Zp [X].
Dorel
Miheţ, Timişoara
n
n
n
n
n
Soluţie. Deoarece f (X + a) = X 2 + C21n aX 2 −1 + . . . + C22n −1 a2 −1 X + 2a2 , iar
n
coeficienţii binomiali C21n , . . . , C22n −1 sunt toţi pari, din criteriul lui Eisenstein rezultă
că f (X + a) (şi la fel f ) este ireductibil peste Q, deci şi peste Z (având coeficientul
dominant 1).
n
Dacă p = 2, atunci fb = X 2 + b
1 este reductibil ı̂n Z2 [X], deoarece este de grad
mai mare decât unu şi are rădăcina 1. Fie p un număr prim impar; putem scrie că
n
d
2n = (X 2
fb = X 2 + a
n
= (X
2n−1
n−1
d
2n ) = (X 2
)2 − (−b
1a
− aÕ ) − (−b
2aÕ · X
2n−1
2
2n−1
2n−1
77
).
n−1
2n−1 )2 − b
2n−1 · X 2
+ aÕ
2aÕ
n−1
n
=
În cazul ı̂n care b
a=b
0, concluzia este imediată. În caz contrar, b
a va fi generator al
grupului ciclic (Z∗p , ·), prin urmare −b
1 = as , b
2 = at şi −b
2 = as+t , pentru anumite
numere naturale s şi t. Deoarece cel puţin unul dintre numerele s, t şi s + t este
par, rezultă că unul dintre elementele −b
1, b
2, −b
2 este pătrat perfect ı̂n Zp , deci fb va fi
reductibil ı̂n Zp (X).
L164. O secvenţă x1 , x2 , . . . , xn , y1 , y2 , . . . , yn de 2n numere reale are proprietatea
(P ) dacă x2i + yi2 = 1, ∀i = 1, n. Fie n ∈ N∗ astfel ı̂ncât pentru orice secvenţă cu
proprietatea (P ), există 1 ≤ i < j ≤ n cu xi xj + yi yj ≥ 0, 947. Determinaţi cea mai
bună constantă α aşa ı̂ncât xi xj + yi yj ≥ α, pentru orice secvenţă cu proprietatea
(P ).
Vlad Emanuel, student, Bucureşti
−−−→
Soluţie. Considerăm punctele Mk (xk , yk ), k = 1, n şi vectorii ⃗vk = OMk . Pentru
o secvenţă cu proprietatea (P), avem că |⃗vk | = 1, k = 1, n, deci xi xj + yi yj = ⃗vi ·
→
→
→
→
×
×
⃗v = |⃗v | · |⃗v | cos(−
v ,−
v ) = cos(−
v ,−
v ). Vom demonstra că, ı̂n ipotezele problemei,
j
i
j
i
j
i
j
n ≥ 20. Presupunem, prin absurd, că n ≤ 19 şi alegem Mk -imaginile rădăcinilor de
2π
2π
ordin n ale unităţii. Atunci xi xj +yi yj ≤ cos
≤ cos
< 0, 947, după cum se poate
n
19
X
→
→
Ú
constata cu ajutorul unui calculator. Fie deci n ≥ 20; deoarece
m((−
v k, −
v k+1 )) =
−
→
−
→
Ú
2π, unde ⃗vn+1 = ⃗v1 , ı̂nseamnă că cel puţin unul dintre unghiurile ( v k , v k+1r
) este cel
√
5+ 5
2π
π
2π
. Pentru acel unghi, xk xk+1 + yk yk+1 ≥ cos
≥ cos
=
mult egal cu
n
n
10
8
şi această constantă nu poate fi ı̂mbunătăţită, căci putem
r lua√n = 20 şi Mk imaginile
5+ 5
rădăcinilor de ordin 20 ale unităţii. În concluzie, α =
.
8
L165. Fie n ≥ 2 un număr natural. Determinaţi cel mai mare număr natural
m pentru care există submulţimile nevide şi distincte A1 , A2 , . . . , Am ale lui A =
{1, 2, . . . , n}, cu proprietatea că fiecare element al lui A este conţinut ı̂n cel mult k
dintre ele, unde:
a) k = 2; b) k = n; c) k = n + 1.
Marian Tetiva, Bârlad
Soluţie. Fie xi , i = 1, n, numărul acelora dintre submulţimile A1 , A2 , . . . , An care
au i elemente şi fie yj , j = 1, m, numărul elementelor lui A care se găsesc ı̂n exact
j dintre mulţimile A1 , A2 , . . . , Am . Avem evident că x1 + x2 + . . . + xn = m, iar
y1 + y2 + . . . + ym ≤ n (se poate să fie elemente ale lui A care să nu aparţină niciuneia
dintre submulţimi). Egalitatea
(1)
x1 + 2x2 + . . . + nxn = y1 + 2y2 + . . . + mym
se obţine numărând ı̂n două feluri toate elementele care apar ı̂n A1 , A2 , . . . , Am , incluzând repetiţiile (şi este adevărată chiar dacă A1 , A2 , . . . , Am nu sunt distincte).
a) În ipoteza k = 2, avem că yj = 0, ∀j ≥ 3, deci y1 +y2 ≤ n şi x1 +2x2 +. . .+nxn =
y1 + 2y2 . Atunci
2m − x1 = x1 + 2(m − x1 ) = x1 + 2(x2 + . . . + xn ) ≤
78
≤ x1 + 2x2 + . . . + nxn = y1 + 2y2 ≤ 2(y1 + y2 ) ≤ 2n,
deci 2m ≤ 2n + x1 ≤ 3n (deoarece x1 nu poate depăşi Cn1 , •numărul
submulţimilor
˜
3n
. Demonstrăm că
lui A cu un singur element). Am obţinut astfel că m ≤
2
• ˜
hni
3n
este chiar maximul căutat: submulţimile cu un element ale lui A şi ı̂ncă
2
2
submulţimi
cu două elemente, alese astfel ı̂ncât să fie două câte două disjuncte, sunt
• ˜
3n
submulţini cu proprietatea că fiecare element al lui A se află ı̂n cel mult două
2
dintre ele.
b) Avem că yj = 0, ∀j ≥ n + 1, deci y1 + y2 + . . . + yn ≤ n, iar relaţia (1) devine
x1 + 2x2 + . . . + nxn = y1 + 2y2 + . . . + nyn . Atunci
3m − 2x1 − x2 = x1 + 2x2 + 3(m − x1 − x2 ) = x1 + 2x2 + 3(x1 + . . . + xn ) ≤
≤ x1 + 2x2 + . . . + nxn = y1 + 2y2 + . . . + yn ≤ n(y1 + . . . + yn ) ≤ n2 ,
deci 3m ≤ n2 + 2x1 + x2 ≤ n2 + 2Cn1 + Cn2 =
3n2 + 3n
. Obţinem, ı̂n cele din urmă,
2
n(n + 1)
. Cum submulţimile nevide cu cel mult două elemente ale lui A
2
n(n − 1)
n(n + 1)
sunt ı̂n număr de n +
=
şi au proprietăţile din enunţ, ı̂nseamnă că
2
2
n(n + 1)
este maximul căutat.
2
n(n + 1) h n i
c) Cu un raţionament asemănător, obţinem că m maxim este
+
.
2
3
că m ≤
ERATĂ
Dintr-o eroare de editare, pe care o regretăm şi pentru care ne cerem scuze, din lista
membrilor Comitetului de redacţie al Recreaţiilor Matematice a fost omis, ı̂ncepând cu
nr. 1/2007, dl. Alexandru CĂRĂUŞU, pasionat susţinător al revistei. În spiritul
corectitudinii, aducem această rectificare de ordin formal.
∗
∗
∗
Recreaţii Matematice – 1/2009
- p. 9, r. 5: ı̂n loc de ”bisectoare” se va considera ”bisectoarea”;
- p. 26, r. 6 de jos: ı̂n loc de ”R” se va considera ”1”;
- p. 79, r. 9: ı̂n loc de ”xi yj + yi yj ” se va considera ”xi xj + yi yj ”;
- p. 81, r. 9 de jos: ı̂n loc de ”xi yi + yi yj ” se va considera ”xi xj + yi yj ”.
Recreaţii Matematice – 2/2009
β
α
β
- p. 96, r. 11 de jos: ı̂n loc de ”aα
1 + . . . + an ” se va considera ”a1 + . . . + an ”;
- p. 172, r. 11 de jos: ı̂n loc de Haşedeu se va considera ”Hasdeu”.
79
Probleme propuse1
Clasele primare
P.184. Vreau să pun ı̂ntr-o cutie bile albe şi verzi, ı̂n total 10, astfel ı̂ncât bile
albe să fie cel mult 6. În câte moduri pot face acest lucru?
(Clasa I )
Inst. Maria Racu, Iaşi
P.185. Ce literă urmează ı̂n ı̂nşiruirea logică VKUJT...?
(Clasa I )
Andreea Amarandei, studentă, Iaşi
P.186. Completaţi dreptunghiurile de mai jos cu numere aşa ı̂ncât suma numerelor scrise ı̂n oricare trei dreptunghiuri alăturate să fie aceeaşi. Ce observaţi?
35
65
35
(Clasa a II-a)
Alexandru Chiriac, student, Iaşi
P.187. Şoricelul Chiţ a primit un zar de la mătuşa Miţ. El a aruncat zarul de
patru ori, obţinând ı̂n total 21 de puncte. Ştiind că la primele două aruncări a obţinut
ı̂n total 9 puncte, aflaţi cât a obţinut la fiecare aruncare. (Găsiţi toate posibilităţile!)
(Clasa a II-a)
Ioana Maria Popa, elevă, Iaşi
P.188. În exerciţiul a + a : a = . . ., există o valoare a lui a pentru care putem să
efectuăm operaţiile ı̂n ordinea scrisă, fără a modifica rezultatul?
(Clasa a III-a)
Ionela Bărăgan, studentă, Iaşi
P.189. Aflaţi numărul natural a ştiind că, dacă se ı̂mparte 25 la 8 − 3 × a, se
obţine restul 1.
(Clasa a III-a)
Mariana Nastasia, elevă, Iaşi
P.190. În grădina casei mele sunt câţiva pomi. Dacă ar fi de patru ori mai mulţi
decât sunt, atunci ar depăşi numărul 20 cu atât cât lipseşte, de fapt, pentru a fi 20.
Câţi pomi sunt ı̂n grădină?
(Clasa a III-a)
Inst. Dumitru Pârâială, Iaşi
?
P.191. Compuneţi o problemă care să se rezolve după schema
alăturată, cu numerele a şi b convenabil alese.
a - ?
(Clasa a III-a)
Amalia Cantemir, elevă, Iaşi
a -
?
P.192. Bunica are mere şi pere. Dacă mi-ar da un sfert din numărul
merelor şi o optime din numărul perelor, aş avea 35 de fructe. Dacă mi-ar
a - b
da o optime din numărul merelor şi o pătrime din numărul perelor, aş avea 40 fructe.
Câte fructe are bunica?
(Clasa a IV-a)
Mihaela Gâlcă, elevă, Iaşi
P.193. Mai multe perechi, formate din câte o fată şi câte un băiat, culeg alune.
În fiecare pereche, alunele culese de băiat sunt fie de patru ori mai multe, fie de patru
ori mai puţine decât cele culese de fată. Numărul alunelor culese ı̂mpreună de fete şi
de băieţi poate fi 2009? Dar 2010?
(Clasa a IV-a)
Mihaela Obreja şi Ioan Lungu, Vaslui
1 Se
primesc soluţii până la data de 31 decembrie 2010.
80
P.194. Arătaţi că există un singur şir format din zece numere naturale consecutive
astfel ı̂ncât suma a opt dintre numere să fie egală cu dublul sumei celorlaltor două.
(Clasa a IV-a)
Petru Asaftei, Iaşi
P.195. Trei fraţi, Ionuţ, Andrei şi Mihai, primesc lunar câte o aceeaşi sumă de
bani de la bunicul lor. Pe ascuns, bunica le dă şi ea aceeaşi sumă. Odată, unul
dintre cei trei a spart un geam. Bunicul, crezându-l vinovat pe Ionuţ, a ı̂mpărţit banii
cuveniţi lui celorlalţi doi. Bunica a procedat la fel, ı̂nsă a crezut că cel vinovat este
Mihai. Andrei, ştiind că el a spart geamul, a ı̂mpărţit jumătate din suma sa celor doi
fraţi şi a constatat că rămâne cu 60 de lei mai puţin decât Ionuţ şi Mihai la un loc.
Ce sumă de bani primea fiecare nepot de la bunicul?
(Clasa a IV-a)
Cosmin Şerbănescu, student, Iaşi
Clasa a V-a
V.116. Se consideră numărul a = (2n · 5n+1 + 4) : 36, n ∈ N∗ . Determinaţi
valorile lui n pentru care a este număr natural, a cărui scriere ı̂n baza 10 are toate
cifrele distincte.
Andrei Nedelcu, Iaşi
1001
V.117. Arătaţi că A = 6
se poate scrie ca diferenţa a două pătrate perfecte.
Damian Marinescu, Târgovişte
V.118. Determinaţi numerele naturale a şi n pentru care a2n − 9 = 8(9 + 92 +
93 + . . . + 92009 ).
Gabriela Popa, Iaşi
V.119. Fie n ∈ N un număr a cărui scriere ı̂n baza 10 este de forma . . . 55.
a) Arătaţi că (n − 5)(n + 5) se divide cu 1000.
b) Aflaţi ultimele trei cifre ale lui n2 .
Mihai Crăciun, Paşcani
V.120. Considerăm numărul natural a = 12345 . . . 9899. Aflaţi restul ı̂mpărţirii
lui a prin 45.
Elena Iurea, Iaşi
1
2
2010
1
V.121. Arătaţi că
+
+ ... +
< .
2·3·4 3·4·5
2011 · 2012 · 2013
4
Tinuţa Bejan, Iaşi
20xy
V.122. Câte fracţii ireductibile de forma
există?
2y
Diana Gregoretti, Galaţi
Clasa a VI-a
Õ cu măsura de 126◦ şi semidrepte (OM1 , (OM2 ,
VI.116. Se consideră unghiul AOB
Ö1 , M
Ø
. . . , (OMn−1 interioare lui, astfel ı̂ncât interioarele unghiurilor AOM
1 OM2 , . . . ,
◦
2 ◦
Ù
Ö
Ø
M
OB
sunt
disjuncte
două
câte
două,
iar
m(
AOM
)
=
2
,
m(
M
OM
n−1
1
1
2 ) = (2 ) ,
n ◦
Ù
Ö
. . . , m(M
n−1 OB) = (2 ) . Dacă (OM este bisectoarea unghiului AOM4 , determinaţi
Ö.
măsura complementului lui AOM
Cătălina Drăgan, Galaţi
81
Õ
b
VI.117. Fie I centrul cercului ı̂nscris ı̂n triunghiul ABC, iar x = m(BIC)−m(
A),
Ô
Ò
Ô
Ò
y = m(AIC)−m(B), z = m(AIB)−m(C). Arătaţi că numerele x, y şi z sunt măsurile
unghiurilor unui triunghi ascuţitunghic.
Constantin Apostol, Rm. Sărat
b
VI.118. În triunghiul isoscel ABC, cu m(A) = 120◦ , se notează cu D mijlocul
laturii [AB]. Perpendiculara din D pe BC intersectează dreptele AC şi BC ı̂n E,
Õ taie BC ı̂n G. Arătaţi că BC = 4 · F G.
respectiv F. Bisectoarea unghiului DEA
Cătălin Budeanu, Iaşi
VI.119. Un număr natural N se termină ı̂n 0 şi are exact 323 de divizori. Aflaţi
ultimele 16 cifre ale numărului N .
Mirela Obreja, Vaslui
2
2
2
VI.120. Demonstraţi că numărul A = 1 + 2 + 3 + . . . + 20102 − 2000 se divide
cu 3.
Nicolae Ivăşchescu, Craiova
VI.121. Fie n ∈ N∗ şi a1 , a2 , . . . , an numere naturale consecutive. Arătaţi că
suma S = a1 + a2 + . . . + an se divide cu n dacă şi numai dacă n este număr impar.
Andrei Paşa, elev, Iaşi
VI.122. Vom spune că un număr natural este anti-Goldbach dacă poate fi scris
ca sumă de două numere compuse. Determinaţi toate numerele anti-Goldbach.
Ionel Nechifor, Iaşi
Clasa a VII-a
1
2008
2 2 3 2009 VII.116. Calculaţi suma S = − + − + . . . + −
.
2 3
3 4
2009 2010 Daniela Munteanu, Iaşi
VII.117. Fie x, y numere reale astfel ı̂ncât x > 2011, iar xy = x + y. Arătaţi că
1
partea fracţionară a lui y este mai mică decât
.
2010
Claudiu Ştefan Popa, Iaşi
VII.118. Arătaţi că x2010 + 1 ≥ x1010 + x1000 , ∀x ∈ R. Când se atinge egalitatea?
Cătălin Melinte, student, Iaşi
n
VII.119. Considerăm numărul natural A = 5 + 2 · 3n−1 + 1, n ∈ N∗ .
a) Demonstraţi că A se divide cu 8, oricare ar fi n ∈ N∗ .
b) Determinaţi n ∈ N∗ pentru care A se divide cu 40.
Ciprian Baghiu, Iaşi
VII.120. Un poligon are 170 de diagonale. Măsurile unghiurilor sale se exprimă,
ı̂n grade, prin numere naturale impare. Demonstraţi că poligonul are cel puţin două
unghiuri congruente.
Cătălin Budeanu, Iaşi
VII.121. În triunghiul ascuţitunghic ABC, notăm cu X, Y şi Z mijloacele ı̂nălţimilor [AA′ ], [BB ′ ], respectiv [CC ′ ] şi cu M, N şi P mijloacele laturilor [BC], [CA],
respectiv [AB]. Demonstraţi că dreptele XM, Y N şi ZP sunt concurente.
Doru Buzac, Iaşi
82
√
VII.122. Se consideră dreptunghiul ABCD, cu AB = 1 şi AD = 1 + 3, iar
Ö
Ö
M este un punct interior dreptunghiului, astfel ı̂ncât m(M
DC) = m(M
CD) = 75◦ .
Arătaţi că triunghiul M AB este echilateral.
Dumitru Mihalache, Bârlad
Clasa a VIII-a
VIII.116. Raportăm planul la un reper
p cartezian xOy. Determinaţi mulţimea
punctelor M (x, y) din plan, pentru care 2 x2 − y 2 = x + y.
Romanţa Ghiţă şi Ioan Ghiţă, Blaj
x
VIII.117. Numerele reale pozitive x, y, z sunt astfel ı̂ncât xy = z+1 şi
+y =
z+1
x y
2. Demonstraţi că + ≥ 3. Când se atinge egalitatea?
y
z
Gheorghe Molea, Curtea de Argeş
VIII.118. Fie x, y, z numere reale astfel ı̂ncât x2 + 5yz ≥ 6, y 2 + 5zx ≥ 6 şi
z + 5xy ≥ 6. Aflaţi valoarea minimă a lui |x + y + z|.
Dan Nedeianu, Drobeta-Tr. Severin
˜
•
˜
•
5x x
5
şi y ∈ − ,
pentru care
VIII.119. Determinaţi x ∈ 0,
4
2 2
2
È
(7 − 2x)(5x + 2y) +
È
(x − 2y)(5 − 4x) = 6.
Liviu Smarandache şi Lucian Tuţescu, Craiova
VIII.120. Rezolvaţi ı̂n numere naturale ecuaţia x + 2y + 3z = 4xyz − 5.
Titu Zvonaru, Comăneşti
VIII.121. Demonstraţi că nu putem alege trei puncte necoliniare A, B, C, de
aceeaşi parte a unui plan α, astfel ı̂ncât dreptele AB, BC şi CA să formeze cu planul
α unghiuri de aceeaşi măsură nenulă.
Petru Asaftei, Iaşi
VIII.122. În fiecare pătrăţel al unei table de şah este scris câte un număr natural.
Fiecare număr n de pe tablă apare de câte n ori, iar pe primul rând al tablei apar,
ordonat strict crescător, toate numerele folosite.
a) Arătaţi că, dintre cele opt numere de pe primul rând, cel mult şase sunt pare.
b) Determinaţi cel mai mare număr care poate apărea pe tablă.
c) Arătaţi că există o singură modalitate de alegere a numerelor care apar pe tablă,
pentru care produsul numerelor de pe primul rând este impar.
Silviu Boga, Iaşi
Clasa a IX-a
IX.106. Stabiliţi valoarea de adevăr a propoziţiilor p şi q, unde
p : (∃a ∈ Z)(∃n ∈ N∗ )(a4 + 1 = 3n );
q : (∃a ∈ Z)(∃n ∈ N)(a4 − 1 = 3n ).
Dan Popescu, Suceava
83
3a + 4b
IX.108.
pentru orice
IX.109.
IX.110.
√ 1 a + 2b √ 1 a √ − 2 <
2 < − 2.
2a + 3b
4 a+b
16 b
Mihail Bencze, Braşov

‹
1 1
4
4
Dacă a ∈ (0, 1), demonstraţi că (a + b)
+ −
≥
,
a b (a + 1)2
(a + 1)2
b ∈ (0, ∞).
Ovidiu Pop, Satu Mare
h πi
Demonstraţi că tg x > 4 sin x − 2, oricare ar fi x ∈ 0, .
2
Ionuţ Ivănescu, Craiova
b = 60◦ .
În △ABC, cu notaţiile uzuale, arătaţi că OI ⊥ OIa ⇔ m(A)
Temistocle Bı̂rsan, Iaşi
IX.107. Dacă a, b ∈ N∗ , atunci −
Clasa a X-a
X.106. Cele m × n pătrăţele ale unui dreptunghi cu m linii şi n coloane se
colorează cu p culori, unde m < p < n. Spunem că o colorare are o tăietură dacă,
pe una dintre cele n coloane, toate cele m pătrăţele au aceeaşi culoare. Determinaţi
numărul colorărilor care au k tăieturi, unde 0 ≤ k ≤ n. (În legătură cu problema 2,
OJM 2006.)
Cecilia Deaconescu, Piteşti

‹
1 0
X.107. Se consideră variabila aleatoare X :
, unde p, q ∈ (0, 1), p ̸= q.
p q
Arătaţi că variabilele aleatoare |X − M (X)| şi (X − M (X))2 sunt dependente.
Laurenţiu Modan, Bucureşti
X.108. Dacă a, b, c ∈ (1, ∞), demonstraţi că are loc inegalitatea
(logb a + logc a) · (loga b + logc b)(loga c + logb c) ≥ 8 log b+c a · log c+a b · log a+b c.
2
2
2
Lucian Tuţescu, Craiova
X.109. Se consideră mulţimile nedisjuncte A şi B şi funcţiile f : A → (0, ∞),
g : B → (0, 1). Determinaţi numărul a ∈ (0, 1) ∪ (1, ∞) pentru care af (x)·g(x) +
loga g(x) ≥ af (x) , ∀x ∈ A ∩ B.
Mihai Haivas, Iaşi
X.110. Fie D = {z = x + iy|y > 0} mulţimea numerelor complexe din semiplanul
superior, iar D′ = {z = x + iy|x2 + y 2 < 1} discul unitate (fără frontieră). Dacă
z − z0
z0 ∈ D este fixat, arătaţi că funcţia f : D → D′ , f (z) =
este bijectivă.
z − z0
Adrian Corduneanu, Iaşi
Clasa a XI-a
3
XI.106. Dacă A ∈ M2 (R), arătaţi că det(A2 + A + I2 ) + det(A2 − A + I2 ) ≥ .
2
Dan Nedeianu, Drobeta Tr. Severin
2
XI.107. Se dă parabola y = ax (a > 0). În fiecare punct P (x, y) al parabolei,
−−→ −
−−→
→
−
→
se consideră vectorul tangent P P ′ = i + 2axj şi vectorul normal P Q′ , orientat spre
84
−−→
−−→
−−→
−−→
exterior, astfel ı̂ncât |P Q′ | = |P P ′ |. Apoi, se consideră vectorul P Q = α(x) · P Q′ ,
unde α(x) este o funcţie dată. Determinaţi locul geometric al punctului Q, atunci
când P descrie parabola, ı̂n fiecare din cazurile:
1
1
a) α(x) = a, ∀x ∈ R; b) α(x) = − , ∀x ∈ R; c) α(x) =
, ∀x ∈ R∗ .
a
2ax
Adrian Corduneanu, Iaşi
√
1+ ln12 + ln13 +...+ ln1n
n
XI.108. Calculaţi lim ( n)
.
n→∞
Cezar Lupu, student, Bucureşti

‹n
a
XI.109. Determinaţi a, b, c ∈ (0, ∞) pentru care limita lim √
există
n
n→∞
b+c
şi este finită.
Constantin Chirilă, Iaşi
XI.110. Date funcţiile continue f, g : R → R, sunt echivalente afirmaţiile:
i) f = g;
ii) dacă h : R → R este continuă, ecuaţia f (x) = h(x) are soluţii reale atunci şi
numai atunci când ecuaţia g(x) = h(x) are soluţii reale.
Marian Tetiva, Bârlad
Clasa a XII-a
Z
ax + 2
dx, unde x ∈ (0, +∞)
x(a + x2 eax )
I.V. Maftei, Bucureşti şi Mihai Haivas, Iaşi
Z π
Z π
2 sin(2n + 1)x
2 cos(2n + 1)x
XII.107. Fie Un =
dx şi Vn =
dx, unde n ∈
sin x
cos x
0
0
π
N∗ . Arătaţi că Un = (−1)n Vn = , ∀n ∈ N∗ .
2
Gheorghe Costovici, Iaşi
Z n
ln x
XII.108. Demonstraţi că şirul (an )n≥1 definit prin an =
dx, ∀n ≥ 1,
p
1 x +1
unde p ∈ (1, ∞) este fixat, este convergent.
Rodica Luca Tudorache, Iaşi
XII.109. Fie (G, ·) un grup comuativ, cu proprietatea că există n ∈ N∗ astfel
ı̂ncât din xn = y n , x, y ∈ G, rezultă că x = y. Dacă f, g sunt endomorfisme ale lui G,
arătaţi că ecuaţia f (x) = g(x−1 ) are soluţie unică ı̂n G, dacă şi numai dacă funcţia
h : G → G, h(x) = f (xn ) · g(xn ) este injectivă.
D.M. Bătineţu-Giurgiu, Bucureşti
XII.106. Pentru a > 0 dat, calculaţi
XII.110. Fie P, Q ∈ C[X] polinoame neconstante, astfel ı̂ncât P şi Q au aceleaşi
rădăcini, iar P − 1 şi Q − 1 au şi ele aceleaşi rădăcini. Arătaţi că P = Q.
Adrian Reisner, Paris
85
Probleme pentru pregătirea concursurilor
A. Nivel gimnazial
G176. Fie a1 , a2 , . . . , an ∈ R∗+ , cu
1
1
1
+ ... + 3
< .
a32 + a23
an + a21
2
1
1
1
1
+
+...+
= 1. Arătaţi că 3
+
a1 a2
an
a1 + a22
Angela Ţigăeru, Suceava
G177. Fie k > 0 şi a, b, c ∈ [0, +∞) astfel ı̂ncât a + b + c = 1. Demonstraţi că
a
b
c
9
+ 2
+ 2
≤
.
2
a +a+k b +b+k c +c+k
9k + 4
Titu Zvonaru, Comăneşti
1−n
n−1
G178. Dacă n ∈ N\{0, 1}, arătaţi că
≤ {nx} − {x} ≤
, ∀x ∈ R.
n
n
Gheorghe Iurea, Iaşi
G179. Determinaţi numerele prime a, b, c, d şi numărul p ∈ N∗ , astfel ı̂ncât a2 +
p
p
+ c2 = d2 + 3.
Cosmin Manea şi Dragoş Petrică, Piteşti
p
b2
p
G180. Aflaţi restul ı̂mpărţirii numărului S = 20102009! + 20092008! + . . . + 21! + 10!
prin 41.
Răzvan Ceucă, elev, Iaşi
G181. Fie k ≥ 1 un număr natural dat. Arătaţi că există o infinitate de numere
naturale n astfel ı̂ncât nk divide n!.
Marian Tetiva, Bârlad
G182. Se consideră triunghiul ABC şi punctele M ∈ [AB], N ∈ [BC], P ∈ [CA]
astfel ı̂ncât M P ∥BC, iar M N ∥AC. Fie {Q} = AN ∩ M P şi {T } = BP ∩ M N .
Demonstraţi că AAM N = AP T N + AQP C .
Andrei Răzvan Băleanu, elev, Motru
b < 30◦ . Ştiind
G183. Se consideră triunghiul isoscel ABC, cu AB = AC şi m(A)
că există D ∈ [AB] şi E ∈ [AC] astfel ı̂ncât AD = DE = EC = BC, determinaţi
b
măsura unghiului A.
Vasile Chiriac, Bacău
G184. În planul xOy, considerăm punctele Aij de coordonate (i, j), unde i, j ∈
{0, 1, 2, 3, 4}. Fie P mulţimea pătratelor care au toate vârfurile printre punctele Aij
considerate. Aflaţi lungimea minimă a unui drum care parcurge numai laturi ale
pătratelor din P şi care uneşte punctele A00 şi A44 .
Claudiu Ştefan Popa, Iaşi
G185. Arătaţi că există o colorare a planului cu n culori, unde n ≥ 2 este un
număr natural dat, astfel ı̂ncât orice segment din plan să conţină puncte colorate cu
fiecare dintre cele n culori.
Paul Georgescu şi Gabriel Popa, Iaşi
86
B. Nivel liceal
L176. Fie D, E, F proiecţiile centrului de greutate G al triunghiului ABC pe
dreptele BC, CA, respectiv AB. Arătaţi că cevienele AD, BE şi CF sunt concurente
dacă şi numai dacă triunghul este isoscel.
Temistocle Bı̂rsan, Iaşi
L177. Considerăm triunghiul ABC, G centrul său de greutate, iar L punctul de
intersecţie al simedianelor. Notăm cu M şi N proiecţiile lui G pe bisectoarea interioară
b iar P şi Q sunt proecţiile lui L pe bisectoarele
şi pe cea exterioară ale unghiului A,
b Demonstraţi că dreptele GK, M N
interioară, respectiv exterioară, ale unghiului A.
şi P Q sunt concurente.
Titu Zvonaru, Comăneşti
L178. Fie ABCD un romb de latură 1 şi punctele A1 ∈ (AB), B1 ∈ (BC), C1 ∈
(CD), D1 ∈ (DA). Demonstraţi că A1 B12 + B1 C12 + C1 D12 + D1 A21 ≥ 2 sin2 A.
Neculai Roman, Mirceşti, Iaşi
L179. Demonstraţi că ı̂n orice triunghi are loc inegalitatea
2(9R2 − p2 )
cos2 A
cos2 B
cos2 C
≥
+
+
≥ 1.
9Rr
sin B sin C
sin C sin A sin A sin B
I.V. Maftei şi Dorel Băiţan, Bucureşti
π
2π
L180. Determinaţi numărul minim de factori din produsul P = sin n · sin n ·
4
4
3π
(22n−1 − 1)π
∗
−9
sin n · . . . · sin
, n ∈ N , astfel ı̂ncât P < 10 .
4
4n
Ionel Tudor, Călugăreni, Giurgiu
L181. Fie P un punct de pe frontiera circulară a unui semidisc, iar d tangenta
ı̂n P la această frontieră. Notăm cu C corpul de rotaţie care se obţine prin rotirea
semidiscului ı̂n jurul dreptei d. Studiaţi variaţia volumului lui C, funcţie de poziţia
punctului P .
Paul Georgescu şi Gabriel Popa, Iaşi
L182. Fie (xn )n≥1 un şir de numere ı̂ntregi cu proprietatea că xn+2 −5xn+1 +xn =
0, ∀n ∈ N∗ . Arătaţi că, dacă un termen al şirului se divide cu 22, atunci o infinitate
de termeni au această proprietate.
Marian Tetiva, Bârlad
L183. Considerăm numerele a ∈ Z, n ∈ N∗ şi polinomul p(X) = X 2 + aX + 1.
Arătaţi că există un polinom cu coeficienţi ı̂ntregi qn şi un număr ı̂ntreg bn , astfel
ı̂ncât p(X)qn (X) = X 2n + bn X n + 1.
Marian Tetiva, Bârlad
2
x
+ y 2 + 2xy − x − 3y + 2
,
L184. Arătaţi că funcţia f : N∗ ×N∗ → N∗ , f (x, y) =
2
este bijectivă.
Silviu Boga, Iaşi
L185. Fie f : R → R o funcţie cu proprietatea că |f (x+y)−f (x)−f (y)| ≤ |x−y|,
∀x, y ∈ R. Arătaţi că lim f (x) = 0 dacă şi numai dacă lim xf (x) = 0.
x→0
x→0
Adrian Zahariuc, student, Princeton
87
Training problems for mathematical contests
A. Junior highschool level
G176. Let a1 , a2 , . . . , an ∈ R∗+ , with
1
1
1
+
+ ... +
= 1. Prove that
a1
a2
an
1
1
1
1
+ 3
+ ... + 3
< .
a31 + a22
a2 + a23
an + a21
2
Angela Ţigăeru, Suceava
G177. Let k > 0 and a, b, c ∈ [0, +∞) such that a + b + c = 1. Prove that
b
c
9
a
+
+
≤
.
a2 + a + k b2 + b + k c2 + c + k
9k + 4
Titu Zvonaru, Comăneşti
1−n
n−1
G178. If n ∈ N\{0, 1}, show that
≤ {nx} − {x} ≤
, ∀x ∈ R.
n
n
Gheorghe Iurea, Iaşi
G179. Determine the prime numbers a, b, c, d and the number p ∈ N∗ , so that
p
p
p
p
a2 + b2 + c2 = d2 + 3.
Cosmin Manea and Dragoş Petrică, Piteşti
G180. Find the remainder of the division of S = 20102009! +20092008! +. . .+21! +10!
by 41.
Răzvan Ceucă, hight-school student, Iaşi
G181. Let k ≥ 1 be a given natural number. Show that there are infinitely many
natural numbers n such that nk divides n!.
Marian Tetiva, Bârlad
G182. The triangle ABC is considered with the points M ∈ [AB], N ∈ [BC],
P ∈ [CA] such that M P ∥BC and M N ∥AC. Let {Q} = AN ∩ M P and {T } =
BP ∩ M N . Prove that AAM N = AP T N + AQP C .
Andrei Răzvan Băleanu, hight-school student, Motru
b < 30◦ . Knowing
G183. Let ABC be an isosceles triangle, with AB = AC şi m(A)
that the points D ∈ [AB] and E ∈ [AC] exist such that AD = DE = EC = BC,
b
determine the measure of the angle A.
Vasile Chiriac, Bacău
G184. The points Aij of coordinates (i, j) are considered in the plane xOy,
where i, j ∈ {0, 1, 2, 3, 4}. Let P be the set of the squares with their vertices among
the considered points Aij . Find the minimum length of a path consisting of square
sides only, which joins the points A00 and A44 .
Claudiu Ştefan Popa, Iaşi
G185. Show that there exists a coloring of the plane by n colours, where n ≥ 2 is
a given natural number, so that any line segment in phe plane contain points colored
by each of the n colours.
Paul Georgescu and Gabriel Popa, Iaşi
88
B. Highschool Level
L176. Let D, E, F be the projections of the centroid G of the triangle ABC onto
the lines BC, CA, and respectively AB. Show that the Cevian lines AD, BE and CF
meet at a unique point if and only if the triangle is isosceles.
Temistocle Bı̂rsan, Iaşi
L177. We consider the triangle ABC with its centroid G and Lemoine′ s point L.
Denote by M and N the projections of G onto the interior and exterior bisector lines
b and let P and Q be the projections of L on the interior and respectively
of angle A
b Prove that the lines GK, M N and P Q meet at the
exterior bisector lines of angle A.
same point.
Titu Zvonaru, Comăneşti
L178. Let ABCD be a rhombus with its side length ℓ = 1 and the points
A1 ∈ (AB), B1 ∈ (BC), C1 ∈ (CD), D1 ∈ (DA). Prove that A1 B12 + B1 C12 + C1 D12 +
D1 A21 ≥ 2 sin2 A.
Neculai Roman, Mirceşti, Iaşi
L179. Prove that the following inequality holds in any triangle:
2(9R2 − p2 )
cos2 A
cos2 B
cos2 C
≥
+
+
≥ 1.
9Rr
sin B sin C
sin C sin A sin A sin B
I.V. Maftei and Dorel Băiţan, Bucureşti
π
L180. Determine the minimum number of factors in the product P = sin n ·
4
2π
3π
(22n−1 − 1)π
∗
−9
sin n · sin n · . . . · sin
, n ∈ N , so that P < 10 .
4
4
4n
Ionel Tudor, Călugăreni, Giurgiu
L181. Let P be a point on the circular boundary of a half-disc, and d the tangent
at P to this boundary. Denote by C the rotation body obtained by the rotation of
the half-disc around the line d. Study the variation of the volume of C as a function
of the position of point P.
Paul Georgescu and Gabriel Popa, Iaşi
L182. Let (xn )n≥1 be a sequence of integer numbers with the property that
xn+2 − 5xn+1 + xn = 0, ∀n ∈ N∗ . Show that if a term of the sequence is divisible by
22 then infinitely many terms there of have this property.
Marian Tetiva, Bârlad
L183. Let us consider the numbers a ∈ Z, n ∈ N∗ and the polynomial p(X) =
X 2 + a X + 1. Show that there exist a polynomial with integer coefficients qn and an
integer number bn such that p(X)qn (X) = X 2n + bn X n + 1.
Marian Tetiva, Bârlad
2
x
+ y 2 + 2xy − x − 3y + 2
,
L184. Show that the function f :N∗ ×N∗ →N∗ , f (x, y)=
2
is bijective.
Silviu Boga, Iaşi
L185. Let f : R → R be a function with the property that |f (x + y) − f (x) −
f (y)| ≤ |x − y|, ∀x, y ∈ R. Show that lim f (x) = 0 if and only if lim xf (x) = 0.
x→0
x→0
Adrian Zahariuc, student, Princeton
89
Pagina rezolvitorilor
CRAIOVA
Colegiul Naţional ”Fraţii Buzeşti”. Clasa a VI-a (prof. BĂLĂŞOIU Ramona).
ENE Cristina: V(109,112), VI(111,112), VIII.113. Clasa a VI-a (prof. IONESCU
Maria). VÎRLAN Leonard: P(171-173), V(102,104,108).
Colegiul Naţional ”Carol I”. Clasa a VII-a. RĂDULESCU Adrian: V(102,105107), VI(104,107,108), VII(102,107).
IAŞI
Şcoala nr. 3 ”Al. Vlahuţă”. Clasa a II-a (inst. MAXIM Gabriela). CUCURUZ
Raluca: P(168,174-177); DASCĂLU Lorena: P(168, 174-177); NICA Daniel: P(168,
174-177); POPESCU Alexandru: P(168, 174-177); ROBU Carmen: P(168, 174177); ŞERBĂNOIU Alexandru: P(168, 174-177); TORAC George: P(168, 174-177).
Clasa a III-a (ı̂nv. MĂRIUŢĂ Valentina). ENEA Codruţ-Alexandru: P(175,177180). Clasa a III-a (inst. CRĂCIUN Marilena). POPESCU Claudia: P(175,177180). Clasa a V-a (prof. MARIN Mirela). CREŢU Cristiana-Paula: P(181183), V(109,114); IFTIME Ioana Evelina: P(181-183), V(100, 111,112); SAFION
Elena Marina: P(181-183), V(109,114). Clasa a VII-a (prof. MARIN Mirela).
ASAVEI Alexandra: VI(113,114), VII(109-112); CELMARE Raluca: VI(113,114),
VII(109-112); MARCU Anca: VI(113,114), VII(109-112); TIBA Ştefana-Alexandra:
VI(113,114), VII(109-112).
Şcoala nr. 11 ”Otilia Cazimir”. Clasa a III-a (inst. PÂRÂIALĂ Dumitru).
POPA Ioana-Maria: P(164-173), V(102,108), VI(108).
Şcoala nr. 13 ”Alexandru cel Bun”. Clasa a II-a (inst. COJOCARIU Ana).
ACATRINEI Andra: P(174-178); BEJAN Matei: P(174-178); BULEI Iasmina-Ioana:
P(174-178); COSTIN Mihăiţă-Alexandru: P(174-178); MUNTIANU Ioana-Andreea:
P(174-178); PERDUN Patricia-Maria: P(174-178); PRISECARU Alexandru-Iulian:
P(174-178); SAMSON Constantin-Cătălin: P(174-178); ŞTEFAN Tudor: P(174-178);
ZAHARIA Ştefan-Eusebiu: P(174-178).
Şcoala nr. 22 ”B.P. Hasdeu”. Clasa a II-a (ı̂nv. NECHIFOR Doina). FETECĂU
Mihai: P(171,174,177,178,184). Clasa a IV-a (inst. DOHOTARIU Liliana). CERCEL Smaranda: P(164,166-169,171,173); CIOFU Alexandra: P(164,166-173); COPĂCEANU CEZARA: P(164-172); FOTEA Oana: P(164-167,172); GHEORGHIŢĂ
Matei: P(164-173); HERGHELEGIU Andreea: P(164-173); JOHN Patricia: P(164172); MANEA Amalia: P(164-173); MIHAI Ana-Maria: P(164-173); OLĂNUŢĂ
Călin: P(168-173); PENESCU Teodora: P(164-172); ROTARU Andreea: P(164167,170); RUSU George: P(164-167, 169,170,172,173); SENDRUC Sânziana: P(164167, 169-172); TUDOSE Miruna: P(164-173); VERUZI Diana: P(164-171).
Şcoala nr. 26 ”George Coşbuc”. Clasa a III-a (inst. VÂRLAN Elena). AMARIEI Romeo: P(174-177,179,180); GHEBAN Andreea: P(174-177, 178,180); PICHIU
Cosmin: P(174-177, 178,180); TĂTARU Alice: P(174-177,178,180); ŢIPLEA Iulian: P(174-177,178,180); TOFAN Raluca: P(174-177, 178,180). Clasa a IV-a (ı̂nv.
BUCATARIU Rica). ANDONESEI Lucian: P(174-175,177-179,181-183); BARHAN
Ştefana-Adina: P(174-175,177-178,181-183); CHIRIAC Alexandra: P(174-175,177179,181-183); CUPEŢ Valeria: P(174-175,177-179,181-183); FRUNZĂ Diana-Mihaela:
90
P(174-175,177-179,181-183); FRUNZĂ Andrei-David: P(174-175,177-179,181-183);
IVANOV Alexandra: P(174-175,177-179,181-183); MÂNDRU Liana: P(174-175,177179,181-183); RADU Andrei: P(174-175,177-179,181-183).
Colegiul Naţional Iaşi. Clasa a V-a (prof. POPA Gabriel). BUDESCU Andrada Ioana: P.183, V(109,110,114,115); MORUZI Mark-Louis: P(181-183), V(109111,114); VERINGĂ Alexandru: P(182,183), V(109,110,115), VI(109,110); VERNICĂ Bianca Elena: P(182,183), V(109-111,114,115). Clasa a VII-a (prof. POPA
Gabriel). ASAFTEI Mircea: VI(109,110), VII(109-111); CIOBANU Ştefan: VI(109,
110,114), VII(109,111); MANGALAGIU Ioan: V(110-112,114), VI(113,114); MURGU
George: VI(109, 110), VII(109-111); PURICE Ioana: V.112, VI.115, VII(110,113,114).
PAŞCANI
Şcoala ”Iordache Cantacuzino”. Clasa a II-a (ı̂nv. MIRON Petru). CRĂCIUN
Ştefana-Maria: P(164-173).
ŢIGĂNAŞI (IAŞI)
Şcoala ”M. Kogălniceanu”. Clasa a III-a (ı̂nv. PĂTRAŞCU Carmen). CAZADOI Ioana-Cristina: P(168,170,173-175); DUCA Cristina-Mihaela: P(168,170,173175); SANDU Rebeca: P(168,170,173-175). Clasa a V-a (prof. CHIRIŢESCU
Anca). DUCA Mirela Beatrice: P(171,173,181,182), VI.108; GĂNESCU Tudor:
P(171-173,181-183); SANDU Codruţa: P(171-173,181,183); VERNER Mădălina-Georgiana: P(171,172,181-183), V.105, VI.108. Clasa a VI-a (prof. CHIRIŢESCU
Anca). BOROS Paula Mihaela: P(173,181-183), V.106, VI.108; DUCA Liliana Daniela: P(171,172,181,183), V(102,105), VI.108; PIU Debora Roxana: P(171,181183), V(105,106). Clasa a VII-a (prof. CHIRIŢESCU Anca). GAVRILAŞ Marius Alexandru: V(102,105-107),VI.108; GĂNEANU Ştefan Bogdan: V(102,105-107),
VI.108; GHIOACĂ Oana: V(102,105-107), VI.108.
Elevi rezolvitori premiaţi
Şcoala nr. 26 ”George Coşbuc”, Iaşi
1. CHIRIAC Alexandra (cl. a IV-a): 1/2009(7pb), 2/2009(7pb), 1/2010(8pb).
2. IVANOV Alexandra (cl. a IV-a): 1/2009(6pb), 2/2009(7pb), 1/2010(8pb).
3. MÂNDRU Liana (cl. a IV-a): 1/2009(7pb), 2/2009(8pb), 1/2010(8pb).
91
IMPORTANT
• În scopul unei legături rapide cu redacţia revistei, pot fi utilizate următoarele
adrese e-mail: t [email protected] şi [email protected] . Pe
această cale colaboratorii pot purta cu redacţia un dialog privitor la materialele trimise acesteia, procurarea numerelor revistei etc. Sugerăm colaboratorilor care trimit probleme originale pentru publicare să le numeroteze
şi să-şi reţină o copie xerox a lor pentru a putea purta cu uşurinţă o discuţie
prin e-mail asupra acceptării/neacceptării acestora de către redacţia revistei.
• La problemele de tip L se primesc soluţii de la orice iubitor de matematici
elementare (indiferent de preocupare profesională sau vârstă). Fiecare dintre
soluţiile acestor probleme - ce sunt publicate ı̂n revistă după un an - va fi
urmată de numele tuturor celor care au rezolvat-o.
• Adresăm cu insistenţă rugămintea ca materialele trimise revistei
să nu fie (să nu fi fost) trimise şi altor publicaţii.
• Rugăm ca materialele tehnoredactate să fie trimise pe adresa redacţiei
ı̂nsoţite de fişierele lor (de preferinţă ı̂n LATEX).
• Pentru a facilita comunicarea redacţiei cu colaboratorii ei, autorii materialelor sunt rugaţi să indice adresa e-mail.
92
Revista semestrială RECREAŢII MATEMATICE
este editată de
ASOCIAŢIA “RECREAŢII MATEMATICE”. Apare la datele de 1 martie şi
1 septembrie şi se adresează elevilor, profesorilor, studenţilor şi tuturor celor
pasionaţi de matematica elementară.
În atenţia tuturor colaboratorilor
Materialele trimise redacţiei spre publicare (note şi articole, chestiuni de
metodică, probleme propuse etc.) trebuie prezentate îngrijit, clar şi concis; ele
trebuie să prezinte interes pentru un cerc cât mai larg de cititori. Se recomandă ca
textele să nu depăşească patru pagini. Evident, ele trebuie să fie originale şi să
nu fi apărut sau să fi fost trimise spre publicare altor reviste. Rugăm ca materialele tehnoredactate să fie însoţite de fişierele lor.
Problemele destinate rubricilor: Probleme propuse şi Probleme pentru
pregătirea concursurilor vor fi redactate pe foi separate cu enunţ şi demonstraţie/rezolvare (câte una pe fiecare foaie) şi vor fi însoţite de numele autorului, şcoala şi localitatea unde lucrează/învaţă.
Redacţia va decide asupra oportunităţii publicării materialelor primite.
În atenţia elevilor
Numele elevilor ce vor trimite redacţiei soluţii corecte la problemele din
rubricile de Probleme propuse şi Probleme pentru pregatirea concursurilor
vor fi menţionate în Pagina rezolvitorilor. Se va ţine seama de regulile:
1. Pot trimite soluţii la minimum cinci probleme propuse în numărul
prezent şi cel anterior al revistei; pe o foaie va fi redactată soluţia unei singure
probleme.
2. Elevii din clasele VI-XII au dreptul să trimită soluţii la problemele
propuse pentru clasa lor, pentru orice clasă mai mare, din două clase mai mici şi
imediat anterioare. Elevii din clasa a V-a pot trimite soluţii la problemele propuse
pentru clasele a IV-a, a V-a şi orice clasă mai mare, iar elevii claselor I-IV pot
trimite soluţii la problemele propuse pentru oricare din clasele primare şi orice clasă mai mare. Orice elev poate trimite soluţii la problemele de concurs (tip G şi L).
3. Vor fi menţionate următoarele date personale: numele şi prenumele,
clasa, şcoala şi localitatea.
4. Plicul cu probleme rezolvate se va trimite prin poştă (sau va fi adus
direct) la adresa Redacţiei:
Prof. dr. Temistocle Bîrsan
Str. Aurora, nr. 3, sc. D, ap. 6,
700 474, Iaşi
Jud. IAŞI
E-mail: [email protected]
CUPRINS
Centenarul SEMINARULUI MATEMATIC "A. MYLLER" (V. OPROIU) ............... 1
SEMINARUL MATEMATIC DIN IAŞI – 100 de ani
de învăţământ matematic românesc (A. PATRAŞ) ............... 4
Amintiri de la SEMINARUL MATEMATIC (V. OPROIU) ................................................ 6
ARTICOLE ŞI NOTE
G. POPA – Rigla şi compasul .............................................................................................. 12
Gh. CIORESCU, A. SANDOVICI – O inegalitate ponderată cu medii........................... 18
F. POPOVICI – Inegalitatea lui Jensen pentru funcţii J-convexe în raport cu
medii cvasiaritmetice .......... 21
V. CHIRIAC, B. CHIRIAC – Inegalitatea H ≤ G ≤ A revizitată..................................... 25
P. MINUŢ, C. SIMIRAD – O extensiune a şirului Fibonacci......................................... 27
L. TUŢESCU – Generalizarea unei identităţi şi aplicaţii ................................................... 31
M. TETIVA – Problema G128 - comentarii ......................................................................... 33
NOTA ELEVULUI
R. CEUCĂ – O problemă de numărare ............................................................................... 35
CORESPONDENŢE
A. REISNER – Un sous-ensemble particulier de matrices carrées ..................................... 37
CUM CONCEPEM... CUM REZOLVĂM
M. TETIVA – O problemă complexă .................................................................................. 41
CHESTIUNI COMPLEMENTARE MANUALELOR
I. PĂTRAŞCU – Axe şi centre radicale ale cercurilor adjuncte unui triunghi................ 45
ŞCOLI ŞI DASCĂLI
V. PARASCHIV – Şcoala Normală "Vasile Lupu" din Iaşi – o istorie zbuciumată ...... 48
CONCURSURI ŞI EXAMENE
Concursul "Recreaţii Matematice", ed. a VII-a, 2009 ........................................................ 51
Concursul de matematică "Gaudeamus", ed. I, 2009 .......................................................... 53
PROBLEME ŞI SOLUŢII
Soluţiile problemelor propuse în nr. 1/2009.......................................................................... 55
Soluţiile problemelor pentru pregătirea concursurilor din nr. 1/2009 ................................. 71
Probleme propuse..................................................................................................................... 80
Probleme pentru pregătirea concursurilor .............................................................................. 86
Training problems for mathematical contests ....................................................................... 88
Pagina rezolvitorilor .............................................................................................................. 90
ISSN 1582 – 1765
7 lei